You are on page 1of 220

March 2010 MRCOG part 1 exam

MCQ, EMQ, Separate items to revise

• large scale study was carried out to assess the benefits of a new drug
which would prevent fractured neck of femur due to osteoporosois in
post menopausal women. the study was carried out correctly using a
treatment group and an appropriate control group. the number of
women who sustained a fracture neck of femur due to osteoporosis
was measured reliably in each group and result are

group with fracture without fracture


control n1 n3
study n2 n4
calculate the number of women who need to recieve new drug in order
to prevent one new fracture of femoral neck due to postmenopausal
osteoporosis for the items below
n1=50,n2=25,n3=9950,n4=9975
answer 400

• Successful lactation is
a maintained by oestrogen F
bmaintained by progesterone F
initiated by LH surge F
maintained by HPL F
inhibited by dopamine T

• Following are required for haemostatic clot formation


coversion of prothrombin to thrombin T
platelet phospholipid T
active conversion of plasminogen to plasmin F
fibrin degradation products F
antithrombin F

• severe oligohydramnios with limb defects


• 2.60-70% ASSOSIATED WITH CLEFT LIP/ cleft palate
• closure of neural groove - 30 days
• closure of mid gut - 14 weeks
• cobalt - 60
• tecnitium- 99

• The pelvic surface of the sacrum?

- gives origin to the piriform muscles


- gives origin to the levator ani muscles
- is broader in the male than in the female
- trnasmit dorsal rami of sacral nerves
- is in contact with the anal canal

• Prolactin.

- release is stimulated by TRH


- plasma levels are raised in the first trim of preg.
- release is increased by suckling.
- maybe produced by decidua.
- release is inhibited by metoclopromide.

• The foll disorders and org are correctly paired.

- opthalmia neonatorum: chlamydia trachomatis


- chancroid: Haemophilus ducreyi.
- sleeping sickness: Leishmania donovani.
- ringworm: Trichenella spiralis.
- non-specific urethritis: Toxoplasma gondii.

• The heart rate typically increases in response to:

- pain.
- hypoxia.
- ventilatory expiration
- increase in Intracranial pressure
- decrease baroreceptor activity

• Antibodies.

- are proteins.
- are formed in the fetus before 12 weeks of Intrauterine life.
- have an average molecular weight of around 10 000 daltons.
- of the rhesus type are genetically transmitted.
- are produced by the ribosome of plasma cells.

• The following disorders have an X linked pattern of inheritance.

- G6PD Deficeincy.
- Kleinfelter syndrome
- adrenogenital syndrome
- haemophilia B
- familial hypercholestroleamia.

• The following are derived from the urogenital sinus:

- the bladder trigone.


- the ureters.
- the female urethra.
- greater vestibular glands.
- paraurethral glands.

• In the fetal CVS


- the heart arises from endoderm
- the heart is formed by fusion of endocardial tubes.
- Cardiac pulsation is present by the 30th day after fertilization.
- oxygenated blood is tranferred to the left atrium through the foramen
ovale.
- the ductus arteriosus closes during the last 4 weeks of pregnancy.

• Arginine vasopressin
- reduces GFR.
- controls water loss in the Proximal renal tubule.
- is synthesised by the post pituitary gland.
- is released in response to rise in plasma osmolality.
- is released in response to fall in circulating plasma volume.

• Renin
- is secreted by the zona glomerulosa of the adrenal coretx.
- is a proteolytic enzyme.
- is secreted at an increased rate if the renal perfusion pressure falls.
- acts upon circulating angiotensinogen.
- is released in response to an increase in extracellular fluid volume

• Actinomyces israelii
- is a rickettsia.
- forms yellow granules in pus.
- is a commensal in the mouth.
- is a commensal in the vagina.
- is usually resistant to penicillin.

• Diseases caused by spirochaetes include.


- Weil's disease.
- lymphogranuloma venereum
- pinta
- Vincent's angina.
- bilharzias

• The following are inherited as autosomal recessive conditions:


- tuberous sclerosis.
- phenylketonuria.
- achondroplasia
- sickle cell anaemia.
- Von gierke's disease.

• Antibodies play an important part in the development of:


- phagocytosis.
- Mantoux responce.
- erythroblastosis fetalis.
- hyperemesis gravidarum
- anaphylaxis

• chemical mediators concerned in the production of an inflammatory


response include:
- 5-hydroxytryptamine
- aldosterone.
- glucocorticoids.
- bradykinin
- leukotreines

• The parathyroid gland.


- originate from the pharyngeal cleft ectoderm.
- secrete parathyroid hormone via the chief( principal) cells.
- secrete calcitonin via the oxyphil cells.
- may become hyperplastic in the presence of intestinal malabsorption.
- may develop adenomas in association with islet cell tumour of the
pancreas

• In the abdominal wall:


- the rectus abdominis muscle is attched to the crest of the pubis.
- the post border of the external oblique muscle ends in the linea
semilunaris.
- the aponeurosis of the external oblique muscle takes part in the
formation of the conjoint tendon.
- the inferior epigastric artery is a branch of the internal iliac artery.
- the conjoint tendon blends medially with the anterior layer of the
rectus sheath.

• The urogenital sinus in the female gives rise to the following:


- ureter
- paraurethral glands
- Bartholin's gland
- urachus
- Gartner's duct.

• Concerning sex hormone:


- the ovary secretes androstenedione.
- The ovary secretes testosterone
- The ovary secretes dihydrotestosterone.
- SHBG conc. are higher in women more than men
- Androgens bound to protein have high biological activity.

• Features of congenital rubella include:


- excretion of virus by the neonates.
- hepatomegaly
- excessive production of growth hormone.
- cataract
- deafness.

• Clomifene citrate:
- is an anti-androgens.
- does not stimulate ovulation directly.
- can produce visual disturbance.
- is genereally prescribed throughtout the proliferative phase of the
menstrual cycle.
- in the treatment of ovulation increases the risk of multiple pregnancy.

• The foll are cytotoxic alkylation agents:


- Cyclophosphamide.
- mercaptopurine
- chlorambucil
- fluorouracil
- methotrexate.

• If a distribution of results is markedly skewed to the left:


- the mean is the same as the 50th centile.
- the same number of values lie on either side of the median.
- the mode is equal to the median.
- the student's t test should be used to compare the distribution with
another.
- logarithmic transformation of the result will produce a distribution
closer to the normal.

• Corcerning the analysis of clinical trials:


- the 95% confidence interval indicates the range within which 19 out of
20 values will lie.
- The P value illustrates how often the result would be expected to occur
by chance.
- c)b The conventional level of statistical significance is set of P<0.005
- In a randomised trial, there must be equal numbers of results in each
arm of the study.
- A relative risk reduction of 60% is significant irrespective of the value
of P.

• The following substance are normally synthesized in the liver:


- glucagon
- vitamin A
- cholesterol.
- immunoglobulins
- prothrombin

• The pineal gland:


- is situated at the anterior end of the 3rd ventricles.
- is innervated by the parasymphathetic nervous system.
- produces melatonin.
- maybe calcified in the adults.
- is most active during daylight.

• In congenital adrenal cortical hyperplasia.


- The commonest deficiency is C18 hydroxylase.
- plasma cortisol concentration is raised.
- urinary excretion of 17 oxysteroids is elevated.
- dexamethasone will suppress the urinary excretion of 17 oxysteroids.
- there is no virilising effects.

• 28)The following are RNA containing virus:


- coxsackie
- influneza
- mumps
- herpes simplex.
- cytomegalovirus.

• Listeria monocytogenes:
- is a gram negative organism.
- is sensitive to ampicillin.
- may cause a transplacental infection.
- is sexually transmitted.
- can be cultured from a high vagina swab.

• actinomyces israelii.
- is a fungus.
- forms yellow granules in pus.
- is a mouth commensal
- occurs in association with IUCD.
- is resistant to penicillin.

• The following drugs may cause enlargement of the fetal thyroid gland:

- methyldopa
- thyroxine
- carbimazole
- propranolol
- propylthiouracil

• The following statements about anticoagulant are correct:


- Heparin inhibits the action of thrombin
- The action of heparin is antagonised by vitamin K
- Heparin increases antithrombin III activity.
- The effects of anticoagulants are decreased by metronidazole.
- Warfarin is greater than 80% protein bound in plasma

• The following drugs and side effects are associated:


- methydopa: depression
- paracetamol: thromboembolism
- indomethacin: peptic ulcer
- prednisolone: osteoporosis.
- ritodrine: hypoglycemia

• Haematopoiesis in the fetus:


- results in nucleated erythrocytes early in development.
- occurs in the yolk sac in the first month.
- does not occur in the bone marrow until term.
- is predominantly hepatic during the 4th month.
- does not require folic acid.

• Early blood borne dissemination is characterised feature of:


- carcinoma of the endometrium.
- osteosarcoma
- basal cell carcinoma
- carcinoma of the cervix
- choriocarcinoma.

• The following cells maybe phagocytic:


- neutrophils.
- kupffer cells
- monocytes
- Hofbauer cells.
- plasma cell.

• The following are premalignant conditions:


- diverticular disease of the large bowel.
- ulcerative colitis.
- pulmonary asbestosis.
- Paget's disease of the bone.
- condylomata of the vulva

• In the pituitary gland:


- the anterior lobe is smaller than the posterior lobe.
- the posterior lobe is ectodermal in origin.
- the acidophil cells produces oxytocin.
- the basophil cells produce growth hormone.
- the blood supply is derived from the internal carotis artery.

• The obturator artery:


- branches from the posterior trunk of the internal iliac artery.
- passes through the greater sciatic foramen.
- is crossed by the ureter.
- supplies the hip joint.
- may be replaced by a branch of the superior epigastric artery.

• In congenital adrenal hyperplasia:


- the commonest cause is a deficiency of 21 hydroxylase.
- the plasma cortisol conc is increased.
- there may be excessive secretion of 17 alpha hydroxyprogesterone.
- sodium retention is characteristic.
- blood cathecholamine conc are increased.

• In the fetal lung:


- bronchial cartilage formation commences at 18-24 weeks of gestation.
- type II alveolar cells first appear at 16-20 weeks gestation.
- sphingomyelin is the most common phospholipid present at term
- phospholipid release is increased by endogenous adrenaline.
- phospholipid production is decreased by exogenous corticosteroids.

• In radiotherapy
- 1 gray is equivalent to 1 joule/kg.
- the skin usually receives a greater dose of radiation than the
underlying tissues.
- the major effect of radiation energy is to damage the cytoplasm of the
cell.
- cells in tissues which are hypoxic are more vulnerable to radiation.
- Radiation induced changes in tissues may take 6 weeks to develop.

• Concerning the adrenal glands


- cortex is derived from neural crest cells.
- Zona fasiculata secretes aldosterone.
- Cortical adenomas may cause Cushing syndrome.
- Neuroblastoma arise in the medulla.
- Addison's disease may result from autoimmune destruction of the
cortex.

• The following are autosomal recessive:


- neurofibromatosis.
- cystic fibrosis.
- phenylketonuria
- polyposis coli
- sickle cell anaemia

• Uterine fibroids:
- are defines histologically as fibromyxomas.
- arise from endometrial stroma
- maybe associated with polycythamia.
- predispose to endometrial hyperplasia.
- are liable to sarcomatous change in about 5% of cases.

• growth of the foll tumors are hormone dependent:


- squamous cell carcinoma of the cervix.
- breast adenocarcinoma.
- uterine leiomyoma.
- prostatic adenocarcinoma
- testicular carcinoma

• Surfactants:
- is formed mainly in the placenta
- levels in amniotic fluid diminish after 33 weeks of gestation
- formation can be inferred from the lecithin-sphingomyelin ratio in
amniotic fluid.
- contains palmitic acid
- decreases the surface tension in pulmonary alveoli.

• Intracellular fluid differs from Extracellular fluids in that:


- it forms the major proportion of total body water.
- its volume can be measured easily.
- it has a higher concentration of potassium than of sodium.
- its volume is regulated primarily by the kidneys.
- it has a higher phosphate concentration.

• The Anal canal:


- has an upper part which is innervated by the inferior hypogastric
plexus.
- has a lower part which is supplied by the superior rectal artery
- drains lymph to the superficial inguinal nodes from its upperpart.
- has its internal sphincter innervated by the infecrior rectal nerve.
- has a superficial part of its external sphincter attached to the coccyx.

• The right ovarian artery:


- Arises from the abdominal aorta above the renal artery.
- passes posterior to the 3rd (horizontal) part of the duodenum
- passes post to the genitofemoral nerve.
- supplies the right ureter.
- anastomoses with the right uterine artery.
• Vulva supplied by:
- Internal pudendal artery.
- Inferior rectal artery.
- Genitofemoral artery
- obturator artery
- femoral artery..

• This is a question outside past papers..All r false Except internal


pudendal artery..

• The vagina:
- has an anterior wall longer than the post wall.
- contains mucus secreting glands in its epithelium.
- is related in its lower third to the bladder base.
- during reproductive life has an acid pH.
- is derived from mesonephric duct.

• The right ureter:


- is approx 50cm in length.
- ia partly covered by duodenum
- crosses the genitofemoral nerve.
- enters the bladder anteromedially.
- receives part of its blood supply from the uterine artery.

• After birth:
- allantois froms median umblical ligament
- umbilical vein forms medial umbilical ligament.
- umbilical artery forms superior vesical artery.
- ductus venosus forms the ligamentum teres.
- ductus arteriosus forms the arch of the aorta.

• In spermiogenesis:
- primary spermatocytes undergo reduction division.
- primary spermatocyte gives rise to 4 spermatids.
- whole process of spermatogenesis in man takes 6-7 days.
- grossly abnormal spermatozoa may be found in fertile semen.
- spermatids are haploid.

• In the human male, dihydrotestosterone:


- is a precursor of testosterone.
- has one-tenth of the ptency of testosterone.
- is responsible for involution of the Mullerain system.
- is responsible of the male external genitalia.
- binds to an intracellular receptor.

• In Human lactation:
- estrogens promote development of breast lobules.
- estrogen promotes milk producing effect of prolactin on the brest,
- human placental lactogen is essential for milk synthesis.
- prolactin stimualtes gonadotrophin release.
- oxytocin causes milk ejection,

• Concerning testicular hormones:


- testosterone reduces plasma LH conc.
- Inhibin stimulates LH production.
- Estrogen are formed in the testis.
- Testosterone is converted to dihyrotestosterone by 5 alpha reductase.
- Testosterone in plasma is predominantly bound to albumin.

• Epidermal growth factor:


- is mitogenic.
- synthesis is stimulated by estradiol.
- is a steroid molecule.
- is found in endometrium.
- binds to a receptor on the nuclear membrane.

• folic acid:
- deficiency causes megaloblastic bone marrow..
- is hydroxycobalamin.
- is present in green vegetables.
- is predominantly absorbed from the large intestine.
- is destroyed by boiling water.

• Doppler Ultrasound:
- is used to monitor fetal breathing.
- is used in fetal HR monitors.
- can be used to measure blood velocity in the fetus.
- measure proton relaxation times.
- requires injection of contrast agents.

• The following are structural aberration of chromosomes:


- deletions.
- inversions.
- aneuploidy.
- polyploidy.
- translocation.

• Messenger RNA

- synthesis is dependant on RNA polymerase.


- is an exact copy of sense DNA.
- contains exons.
- is measured by western analysis.
- translationoccurs in the nucleus.

• In tumors of the bones:


- primary malignancy is more common than 2ndry malignancy.
- osteoma rarely present in skull bones.
- osteosarcoma is associated with Paget's disease of bone.
- lymph node metastases are unusual.
- simple bone cysts have a strong tendency to recur.

• Concerning the rectus sheath.


- Above the costal margin rectus abdominis lies on the costal cartilages.
- Below the arcuate line the internal oblique splits to enclose rectus
abdominis.
- It contains the musculophrenic artery.
- It is innervated by the ilioinguinal nerve
- Pyramidalis is innervated by the subcostal nerve.

• The external iliac artery.


- Enters the thigh anterior to the inguinal ligament.
- at its origin is crossed by the ureter.
- at its origin is crossed by the ovarian vessels.
- Lies medial to the external iliac vein at its distal end.
- gives rise to the deep external pudendal artery.

• Action of Insulin include stimulation of:


- glycogenolysis by the liver.
- cellular uptake of amino acids.
- entry of glucose into neurons.
- d)entry of glucose into adipose tissue.
- cellular uptake of potassium.

• Metronidazole:
- is effective against Giardia lamblia.
- is effective when administered per rectum
- should not be administered intravenously.
- is usually effective against Entamoeba histolytica.
- interferes with ethanol metabolism.

• Amniotic fluid:
- at term is hyperosmolar compared to fetal plasma.
- normally contains maternal and fetal cells.
- contains a higher concentration of alpha feto protein at 16 weeks than
at term.
- contains bilirubin.
- contains phospholipids.

• Chromosomes:
- are found in the same numb in all mammalian cells.
- can be analysed more quickly from a blood sample than from an
amniotic fluid sample.
- can be reliably identified by their length.
- the Y chromosome is larger than the X chromosome.
- DNA content is doubled during the S phase of the cell cycle.

• In cystic fibrosis, abnormalities are seen in the


- pancreas
- salivary glands.
- brain
- kidneys
- ileum

• Neutrophil polymorphs at the site of inflammation are capable of the


following:
- Phagocytosis.
- production of oxygen free radicals.
- replication
- fusion to form giant cells.
- antibody production

• In the statistical analysis of any group of numericals


- the mean is always less than the mode
- standard deviation is always greater than the standard error
- the median value lies at the midpoint of the range
- the standard error of the mean is independent of the total
number of observations
- there are the same number of observations greater than and
less than the median value

• In a randomised double blind trial comparing a new drug with a


placebo
- the patient will be taking either of two drugs
- patients can choose their mode of treatment
- doctors prescribing treatment decide which patients take the
new drug
- a large trial is more likely to give a statistically significant result
than a small trial
- half of the patients will take the new drug

• In a trial of oral hypoglycaemic agents 42 patients were given drug A


and 38 drug B . blood glucose conc. were measured befor and after a
single dose of the drug. Drug B apparently caused greater fall in the
blood glucose conc. (P=0.06)
- these results reach an accepted level of statistical significants
- Non- parametric statistical analysis bshould be used if datd are
not normally distributed
- In biological terms drugs A and B have been shown to be eqully
effective
- 6% more patients responded to drug A than drug B
- Unequal numbers in the two groups invalidate the trial

• The following statistical statements are correct

- In the normal distribution the value of the mode is 1.73 * that


of the median
- In a distribution skewed to the right, the mean lies to the left to
the median
- In the series 2;7;5;2;3;2;5;8, the mode is 2
- Student's test is designed to correct for skewed distributions
- the Chi-squared test may be used when data are not normally
distributed

• pudendal nerve, nerve for perianal area, down syndrome, patau


syndrome, cobalt, technitium, dna virus, HIV virus, syphillis, heptitis b
and c virus, PG I2, ionic exchange assosiated with calcium and
potassium, positive predictive value, negative predictive value,
aspirin,mefanamic acid, insulin secretion and glucagon sec inhibited
and stimulated by what substances
• PUDENDAL NERVE ROOT VALUE S1,2,3 ANT PRIMARY RAMI
• N SUPPLY OF PERIANAL AREA
• CLOSURE OF NEURAL GROOVE ON -WK OF IUL
• LENGTH OF UMBILICAL CORD
• AUTOSOMAL RECESSIVE TAY SACH'S DISEASE
• AUTOSOMAL DOMINANT ACHONDROPLASIA
• ANTIINFLAMMATORY MUSCLE RELAXANT PROSTAGLANDIN
• STAPH AUREUS COAGULASE POSITIVE, PIGMENTED COLONY,GRAM
POSITIVE
• GENTAMYCIN BACTERICIDAL,TOXIC TO 8 TH NERVE
• DIURETIC ACTING ON DCT BENDROFLUAZIDE
• Successful lactation is
- A- maintained by oestrogen F
- B-maintained by progesterone F
- c -initiated by LH surge F
- d- maintained by HPL F
- e -inhibited by dopamine T

• The following are required for haemostatic clot formation


- a -coversion of prothrombin to thrombin T
- b -platelet phospholipid T
- c -active conversion of plasminogen to plasmin F
- d -fibrin degradation products F
- e -antithrombin F

• the following statements about vitamins are true


- vit k is water soluble
- vit D is poorly absorbed in cases of obstructive jaundice
- vit A is a fat soluble vit.
- vitamins supply the body wd energy
- vit D is bound to transport protein in the circulation.

• ANSWERS
- a.F- fat soluble
- b.T- also in cases of pancreatic disease where there is pancreatic lipase
deficiency.
- c.T-
- d.F- vitamins r organic dietry constituents necessary for life haelth n
growth that do not function by supplying the body wd energy.
- e- T- physiologically active form of vitamin D is known as calcitriol.
- calcitriol is released into the circulation, and by binding to a carrier
protein in the plasma, vitamin D binding protein (VDBP), it is
transported to various target organs.

• VITAMIN B12
- a fat soluble vit
- b-absorption takes place throughout the small intestine
- c-is essentail for the metabolism of folic acid in human body.
- d-deficiency leads to macrocytic anemia
- e-deficiency is common in strict vegetarian.

• ANSWERS
- a-F- water soluble
- b-F-absorbed mainly in lower ileum aided by gastric intrinsic factor.
- C-T
- d-T
- e-T- source is mainly from animal food stuff, vegetables alone are
inadequate source.

• FOLIC ACID
- a-water soluble
- b-requires gastric intrinsic factor
- c-is necessry for nucleic acid synthesis
- d-heat stable
- e-involved in krebs cycle

• ANSWERS
- a-T
- b-F
- c-T-tetrahydrofolate is essentaial for both purine and pyramidine
synthesis.
- d-F- the normal western diet contains 500-700 microgm/day of wwch
10-100% lost in cooking.
- e-F
• Folic acid (also known as vitamin B9 or folacin) and folate (the
naturally occurring form), as well as pteroyl-L-glutamic acid and
pteroyl-L-glutamate, are forms of the water-soluble vitamin B9.

• Folic acid is itself not biologically active, but its biological importance is
due to tetrahydrofolate and other derivatives after its conversion to
dihydrofolic acid in the liver.

• Vitamin B9 (folic acid and folate inclusive) is essential to numerous


bodily functions ranging from nucleotide biosynthesis to the
remethylation of homocysteine.

• The human body needs folate to synthesize DNA, repair DNA, and
methylate DNA as well as to act as a cofactor in biological reactions
involving folate.

• It is especially important during periods of rapid cell division and


growth.

• Both children and adults require folic acid to produce healthy red blood
cells and prevent anemia.

• Leafy vegetables such as spinach, asparagus, turnip greens, romaine


lettuces, dried or fresh beans and peas, fortified grain products (pasta,
cereal, bread), sunflower seeds and certain other fruits (orange juice,
canned pineapple juice, cantaloupe, honeydew melon, grapefruit juice,
banana, raspberry, grapefruit, strawberry) and vegetables (beets,
broccoli, corn, tomato juice, vegetable juice, brussels sprouts, bok
choy) are rich sources of folate.
• Liver and liver products also contain high amounts of folate, as does
baker's yeast.

• Some breakfast cereals (ready-to-eat and others) are fortified with


25% to 100% of the recommended dietary allowance (RDA) for folic
acid.

• Folic acid naturally found in food is susceptible to high heat, UV, and is
soluble in water.

• Folic acid is an important nutrient for women who may become


pregnant, because a woman's blood levels of folate fall during
pregnancy due to an increased maternal RBC synthesis in the first half
of the pregnancy and fetal demands in the second half.

• The first four weeks of pregnancy (when most women do not even
realize they are pregnant) require folic acid for proper development of
the brain, skull, and spinal cord.

• Serious birth defects like neural tube defects are less likely to occur
when women take 0.4 mg of folic acid daily.

• Neural tube defects (NTDs) result in malformations of the spine (spina


bifida), skull, and brain (anencephaly).

• The risk of neural tube defects is significantly reduced when


supplemental folic acid is consumed in addition to a healthy diet prior
to and during the first month following conception.

• Folate deficiency during pregnancy can increase the risk of preterm


delivery, infant low birth weight, and fetal growth retardation.
• Folate deficiency in the mother increases homocysteine level in the
blood which may lead to spontaneous abortion and pregnancy
complications such as placental abruption and preeclampsia.Folic acid
appears to reduce the risk of stroke.

• Folic acid supplements help relieve hot flushes in postmenopausal


women. Just like in estrogen hormone replacement therapy, folic acid
interacts with neurotransmitters (norepinephrine, serotonin) in the
brain to reduce hot flushes.

• IN SHORT: Folate deficiency may lead to glossitis, diarrhea,


depression, confusion, anemia, and fetal neural tube defects and brain
defects (during pregnancy).

• VITAMIN C
- a-is found only in animal food stuffs.
- b-is rapidly destroyed by heating.
- c-there are normally large stores in pancreas.
- d-impared wound healing is one of the characterstic features of severe
vit c defeciency.
- e-exess vit c can lead to formation of oxalate stones in urinary tract.

• ANSWERS
- a-F. citrus foods n leafy green veges r rich in vit c while animal sources
contain only traces.
- b-T
- c-F-eye n adrenal glands contain large quantities of vit c.
- d-T
- e-T

• Vitamin C or L-ascorbic acid is an essential nutrient for humans, in


which it functions as a vitamin.
• Ascorbate (an ion of ascorbic acid) is required for a range of essential
metabolic reactions in all animals and plants.

• Deficiency in this vitamin causes the disease scurvy in humans.

• Ascorbic acid is absorbed in the body by both active transport and


simple diffusion.
• Sodium Dependent Active Transport - Sodium-Ascorbate Co-
Transporters (SVCTs) and Hexose transporters (GLUTs) are the two
transporters required for absorption.

• Although the body's maximal store of vitamin C is largely determined


by the renal threshold for blood, there are many tissues which
maintain vitamin C concentrations far higher than in blood.
• Biological tissues that accumulate over 100 times the level in blood
plasma of vitamin C are the adrenal glands, pituitary, thymus, corpus
luteum, and retina.
• Those with 10 to 50 times the concentration present in blood plasma
include the brain, spleen, lung, testicle, lymph nodes, liver, thyroid,
small intestinal mucosa, leukocytes, pancreas, kidney and salivary
glands.

• Scurvy is an avitaminosis resulting from lack of vitamin C, since without


this vitamin, the synthesised collagen is too unstable to perform its
function.

• Scurvy leads to the formation of liver spots on the skin, spongy gums,
and bleeding from all mucous membranes.
• The spots are most abundant on the pages and legs, and a person
with the ailment looks pale, feels depressed, and is partially
immobilized.
• In advanced scurvy there are open, suppurating wounds and loss of
teeth and, eventually, death.

• The human body can store only a certain amount of vitamin C, and so
the body soon depletes itself if fresh supplies are not consumed.

• In humans, vitamin C is essential to a healthy diet as well as being a


highly effective antioxidant, acting to lessen oxidative stress; a
substrate for ascorbate peroxidase;and an enzyme cofactor for the
biosynthesis of many important biochemicals.

• Vitamin C acts as an electron donor for important enzymes:[


• Vitamin C acts as an electron donor for eight different enzymes:

• Three participate in collagen hydroxylation.


• These reactions add hydroxyl groups to the amino acids proline or
lysine in the collagen molecule via prolyl hydroxylase and lysyl
hydroxylase, both requiring vitamin C as a cofactor.
• Hydroxylation allows the collagen molecule to assume its triple helix
structure and making vitamin C essential to the development and
maintenance of scar tissue, blood vessels, and cartilage.

• are necessary for synthesis of carnitine.


• Carnitine is essential for the transport of fatty acids into mitochondria
for ATP generation.

• The remaining three have the following functions in common but do


not always do this:

• -dopamine beta hydroxylase participates in the biosynthesis of


norepinephrine from dopamine.
• -another enzyme adds amide groups to peptide hormones, greatly
increasing their stability.

• -one modulates tyrosine metabolism.

• DAILY REQUIREMENTS:

• 75 milligrams per day: the United Kingdom's Food Standards Agency

• 45 milligrams per day: the World Health Organization

• VITAMIN B.
- a.B1(thiamin)deficiency leads to impaired collagen formation
- b.B1 stores in the body are adequate for upto 9 months
- c.B2(riboflavin) concentration is higher in fetus than in mother
- d.B6(pyridoxine)requirement in pregnancy is 25mg/day
- e.niacin is synthesised in the body from tryptophan.

• ANSWERS
- a-T-
- b-F- the body contains only 30mg(average adult requirement is 1-1.5)
- c-T- as with other water soluble vit, vit B crosses the placenta by active
mechanisms, wch results in higher concentration in fetus
- d.F- the true figure is 2.5mg/day an din non pregnant adult is
2mg/day.
- e-T

• VITAMIN A
- a.requires bile for its absorption.
- b.deficiency leads to night blindness.
- c.excess leads to xerophthalmia
- d.stored in liver
- e.daily dietry requirement during pregnancy is 50mg/day

• ANSWERS
- a.F
- b.T
- c.F- xerophthalmia is due to its deficiency. hypervitaminosis A is
charachterised by anorexia , headache, hepatosplenomegaly,patchy
loss of hair and hyperostosis.
- d.T
- e.F-during pregnancy the requirement is 1000
- microgm/day.

• Vitamin A is a vitamin which is needed by the retina of the eye in the


form of a specific metabolite, the light-absorbing molecule retinal.

• This molecule is absolutely necessary for both scotopic and color


vision.

• Vitamin A also functions in a very different role, as an irreversibly


oxidized form retinoic acid, which is an important hormone-like growth
factor for epithelial and other cells.

• Vitamin A can be found in two principal forms in foods:

• -retinol
• -carotenes
• Vitamin A is found naturally in many foods:

• liver (beef, pork, chicken, turkey, fish)


• carrot
• broccoli leaf
• sweet potato
• kale
• butter
• spinach
• pumpkin
• collard greens
• cantaloupe melon
• egg
• apricot
• papaya
• mango
• pea
• broccoli

• Vitamin A plays a role in a variety of functions throughout the body,


such as:

• Vision
• Gene transcription
• Immune function
• Embryonic development and reproduction
• Bone metabolism
• Haematopoiesis
• Skin health
• Antioxidant Activity

• DEFICIENCY leads to
• -night blindness

• -xerophthalmia.
• First there is dryness of the conjunctiva (xerosis) as the normal
lacrimal and mucus secreting epithelium is replaced by a keratinized
epithelium.
• This is followed by the build-up of keratin debris in small opaque
plaques (Bitot's spots) and, eventually, erosion of the roughened
corneal surface with softening and destruction of the cornea
(keratomalacia) and total blindness.

• Other changes include impaired immunity, hypokeratosis (white lumps


at hair follicles), keratosis pilaris and squamous metaplasia of the
epithelium lining the upper respiratory passages and urinary bladder to
a keratinized epithelium.

• With relations to dentistry, a deficiency in Vitamin A leads to enamel


hypoplasia.

• Adequate supply of Vitamin A is especially important for pregnant and


breastfeeding women, since deficiencies cannot be compensated by
postnatal supplementation.

• However, excess Vitamin A, especially through vitamin


supplementation, can cause birth defects and should not exceed
recommended daily values.

• VITAMIN E
- a.present in animal food stuffs only.
- b.deficiency may cause intrauterine fetal death
- c.potentiates the action of coumarin anticoagulants
- d.used in the treatment of infertility
- e.dietry requirement is 10mg/day

• ANSWERS
- a.F-present in most foods
- b.F-proven only in animals and not in humans
- c.T
- d.F-there is no evidence that vit e increases virility, or plays any role in
treatment of infertility or recurrent abortion.
- e.T

• VIT D
- a.water soluble
- b.is stored in body fat
- c.is absorbed from large intestine
- d.deficiency leads to rickets
- e.dietry req is 10mg/day

• ANSWERS
- a.F
- b.T
- c.F-it is absorbed from small intestine
- d.T
- e.T

• VITAMIN K
- a.mainly found in green leafy veges
- b.in adults no external supplements necessary
- c.hypervitaminosis is charactererised by anemia
- d.exists in 2 forms k1 n k2
- e.is stored in large quantities in the liver

• ANSWERS
- a.T
- b.T-except in pregnant pts on anti epilactics who require vit k in last
months of pregnancy and in new borns.
- c.T
- d.T
- e-Falthough accumulates in liver initially but its hepatic concentration
declines rapidly.

• REGARDING metabolism
- a.the metabolic rate is the amount of energy liberated per unit of time
- b.anabolism is defined as the formation of substances which can store
the energy.
- c.basal metabolic rate is defined as the metabolic rate determined at
rest in a room at 12-14hrs after the last meal.
- d.the BMR of a man is about 500kcal/day
- e.the metabolic rate is decreased after consumption of a meal that is
rich in protein.

• ANSWERS
- a.T
- b.T
- c.T
- d.F-2000kcal/day
- e.F-the metabolic rate is increased after consumption of a meal rich in
protein or fat.

• ANABOLISM
- the set of metabolic pathways that construct molecules from smaller
units.
- These reactions require energy.

• BASAL METABOLIC RATE


• The primary organ responsible for regulating metabolism is the
hypothalamus.

• REGARDING METABOLISM
- a.oxidation is the comination of a substance with either oxygen or
hydrogen.
- b.Co-factors r essential for certain enzyme reactions.
- c.A co-enzyme is a protein substance wch acts as a carrier for products
of reaction.
- d.Co-enzyme A is a high energy compound wch is formed from
adenine,ribose pentotothenic acid and thioethanol amine.
- E.a calorie is defined as the amount of heat energy needed to
raise the temperature of 1gm of water by 1 degree ,from 15 to
16degreesC.
• ANSWERS
• a.F-oxidation is the combination of a substance with oxygen or loss of
hydrogen or loss of an electron. reduction is the reverse of this.
• b.T
• c.F-it is an organic non-protein substance.
• d.T
• e.T

• ENZYMES

- a.are proteins
- b.heating usually results in a complete loss of enzyme activity
- c.a change in pH has no effect on the activity of an ezyme.
- d.are present in all cell organelles.
- e.organic solvents will usually destroy an enzymes activity.

• ANSWERS
- a.T
- b.T
- c.F-a change in pH has an effect on enzyme activity.
- d.T
- e.T

• PROTEIN METABOLISM
- a.proteins contain abt 40%nitrogen
- b.chains containing>100 amino acid residues are called proteins
- c.proteins yield 4 calories /gm absorbed.
- d.during pregnancy there is a rise in the plasma concentration of
triglycerides.
- e.during pregnancy there is a rise in the plasma concentration of
albumin.

• ANSWERS
- a.F
- b.T
- c.F
- d.T
- e.F

EMQ:
• ant division of ant. remi of S2-4
• ant division of post.rami of S2-4
• genitofemoral nerve
• post division of ant. remi of S2-4
• post division of post. remi of S2-4
• sensory supply of perineal nerve
• inferior rectal nerve
• ischial tuberosity
• ischial spine

• A lady is about to deliver and you are about to give her a pudendal
block..

• Q1- what is the root value of pudendal nerve?


• Q2- what nerve supplies lower part of vagina (I think)?
• Q3- why do u give local skin infiltration before episiotomy?
• Q4- or what is the nerve supply of peranal area .?

• (2).. If we take the day of fertilization as Day’0’ then…


• Day2
• Day4
• Day 8
• Day 10
• Day12
• Day 14
• Day 18
• Day20
• Day 22
• Day 24
• Day 26
• Day 42 [
• Day 70

• Q1- Which represents 4 cell stage ?


• Q2- conceptus implants completely?
• Q3- fetal heart pulse seen on ultrasound?
• Q4- vertebra form completely?

• A Cytoplasmic transcription factor receptor


• B G protein coupled receptor on cell membrane
• C G protein coupled receptor on Golgi complex
• D Mega subunit ligand gated ion channel
• E Multisubunit antibody receptor on cell membrane
• F Multisubunit ligand gated ion channel on cell membrane
• G Nuclear protein kinase receptor
• H Nuclear transcription factor receptor
• I Protein kinase receptor on cell membrane
• J Receptor protein complex (intracytoplasmic)
• K Transcription factor receptor on cell membrane

• Select the receptor which binds the molecules referred to in the itemS
below. Each option may be used once, more than once or not at all.

- insulin .
- progesterone .
- ostradiol .
- prostaglandin .

• (4)
• PCR
• Fish test
• Chromosomal linkage analysis
• Telomer analysis
• Sum hereditary crosslinkage chromosomal option (dnt remember)
• Screen for some known gene mutation
• Sweat test
• Saliva test

• -A family with one kid presumed to have cystic fibrosis however cftr
gene mutation was not detected. Family members willing to give
samples if needed.mother wants definite diagnosis as she is 11wks and
wants to know diagnosis for her baby.parents are cousins with 1st
degree relatives having cystic fibrosis.

• Invasive diagnostics can be applied to reach final


results by?
• you have Guthrie spot of their son how will u
diagnose him for cystic fibrosis?
• families in UK with cystic fibrosis how will u find
f508?

• (5)
o Endoplasmic reticulum
o Golgi apparatus
o Lysosomes
o Microtubules
o Mitochondria
o Nucleolus
o Plasma membrane
o Ribosomes

• Select the most appropriate organelle that matches the following


descriptions

ƒ Contains enzymes capable of digesting cells and cellular


material
ƒ 'Reads' the mRNA and builds protein .
ƒ Modification of lipids and proteins with storage of
material prior to export out of the cell.

• (6)[i][b] screening test for following


ƒ syphilis treponemal antibody test
ƒ syphilis hemaglutination test
ƒ syphilis immobilization test
ƒ HB-electrophorisis
ƒ mcv
ƒ mchc
ƒ MCh
ƒ serum iron
ƒ paul-bennel test[/b][/i]

• alpha thalessemia
• folic acid deficiency
• iron deficiency anemia
• primary syphilis in early untreated period

• (7) on taking a large protein meal?


• Decrease decrease
• Increase increase
• Increase decrease
• Deacrease increase
• Nochange no change
• Nochange increase
• Nochange decrease

• Insulin & Glucagon excretion what will happened ...?

• (8)
o 3-hydroxyisovaleric acid .
o 17-alpha hydroxyprogesterone .
o 17-delta hydroxyprogesterone .
o acetic acid .
o cortisol .

• The following level of certain metabolities in amniotic fluid changes


significantly of the fetus has an inborn error of the metabolism .select
the single metabolite from list of the options above whose level in
amniotic fluid is altered by inheretiance disease in item below.
• congentiat adrenal hyperplasia

• (9)
ƒ Adrenaline
ƒ Calcitonin
• C Cholecystokinin
ƒ Cortisol
ƒ Glucagon
ƒ Insulin
ƒ Progesterone
ƒ Somatostatin
• IT- estosterone

• Select the appropriate hormone from the list for the following
structures that produce it. Each answer may be used once, more than
once, or not at all.

ƒ Adrenal Cortex .
ƒ Adrenal Medulla .
ƒ Pancreatic alpha cell .
ƒ Pancreatic D Cells .
• (10)
• AAnterior cerebral artery
• BAnterior communicating artery
• CAnterior inferior cerebellar artery
• DBasilar artery
• EInternal auditory artery
• FInternal carotid artery
• GMiddle cerebral artery
• HPosterior cerebral artery
• IPosterior communicating artery
• JSuperior cerebellar artery
• KVertebral artery
• Complete the diagram of the circle of Willis using the options given:
• this question came with diagrame in september 2007
• so you should study circle of weils very well if I get the pictures I will
post it .

• Akeratinising stratified squamous epithelium


• Bnon-keratinising stratified squamous epithelium
• Cpseudostratified columnar epithelium
• Dsimple columnar epithelium
• Esimple cuboidal epithelium
• Ftransitional epithelium

• Which of the epithelial types described above are present in the


anatomical regions below:

ƒ ectocervix .
ƒ endocervical canal .
ƒ cervical ectropion .
ƒ transformation zone of cervix .
ƒ vagina .
ƒ uterus .
ƒ Bladder .
ƒ Trachea .
ƒ Labia Majora .
ƒ Bowel .
• 11-Fallopian tube .
ƒ Vulva .
ƒ anal canal .

• Autosomal co-dominant
• Autosomal dominant
• Autosomal recessive
• Polygenic
• Single gene defect
• X linked dominant
• X linked recessive

• [Select the most likely mode of inheritance for the following patients’
conditions:

- A 27 -year old female developed gestational diabetes mellitus.


Her uncle and grandmother also had diabetes mellitus2- A
mothr is concerned regarding her baby who has developed
fractures which appear to occur with minimal trauma. He has
blue sclera.
- An 18-year old female underwent caries tooth extraction and
developed profuse bleeding. On history she revealed
menorrhagia. Her mother and her grandfather had the same
disease.
ƒ WARFARIN .
ƒ HEPARIN .
ƒ AMPICILLIN .
ƒ METHYLDOPA .
ƒ PENCILLIN .
ƒ CARBIMAZEPINE .
ƒ METRONIDAZOLE .
ƒ NON OF THE ABOVE .
ƒ ALL OF THE ABOVE .

• For each of the following choose the single most appropirate statement
from the above list of options .
- Drug contraindicated in breast feeding .
- Drug does not cross the placenta .
• A.
• Calcitonin

• B.
• Cortisol

• C.
• Glucagon

• D.
• Growth hormone

• E.
• Insulin

• F.
• Oestradiol 17-¦Â¦Â

• G.
• Oxytocin

• H.
• Parathyroid hormone

• I.
• Prolactin

• J.
• Thyroxine

• Instructions: For each action described below, choose the single most
likely causative hormone from the above list of options. Each option
may be used once, more than once, or not at all.

• Question 3: Stimulates deposition of cartilage at the ends of bones


• Question 4: Raises blood glucose levels through the breakdown of fat
and protein
• Question 5: Antagonises the effect of parathyroid hormone to minimise
bone density loss
• Question 6: Stimulates the release of milk from the breast
• Question 7: Stimulates the urinary secretion of calcium

• Options
• A.
• Common iliac artery
• B.
• External iliac artery
• C.
• Inferior epigastric artery
• D.
• Inferior vesical artery
• E.
• Internal iliac artery
• F.
• Middle rectal artery
• G.
• Ovarian artery
• H.
• Superior vesical artery
• I.
• Umbilical artery
• J.
• Uterine artery
• K.
• Vaginal artery

• Instructions: After a forceps delivery a 30-year-old primigravida


sustains a primary post partum haemorrhage of four litres. Although
the uterus appears well contracted the bleeding continues, and a
decision is made to identify and treat the bleeding point radiologically.
For each question posed below, choose the single most appropriate
option from the above list. Each option may be used once, more than
once or not at all.

• Question 8: What vessel runs up the broad ligament?


• Question 9: Which other vessel arises from the uterine artery?
• Question 10: Which other vessel anastomoses with the uterine artery?
Question 11: Which vessel does the uterine artery arise from?
• Question 12: Which vessel runs anterior and superior to the ureter?
- vitamin A
• B- vitamin E
• C- vitamin D
• vitamin B12
• vitamin B6
• vitamin C
• folic acid
• riblflavine
• vitamin K
• thiamine

- fat soluble vitamin sysnthesised in the intestinal wall from the


beta carotene .

- fat soluble vitamin synthesised by large intestinal bacteria .

- deficiency of this vitamin in childhood causes rickets .

- fat soluble vitamin synthesis by the kidney is regulated by


parathyroid hormone .

- deficiency of this vitamins typicaly occurs in women with


hyperemesis gravidarum .

- water soluble vitamin with anti-oxidant effects .

- absorption of this vitamin requires the presence of intrinsic


factor .

- deficiency of this vitamin causes megaloblastic anaemia and


neurological disorders .

- fat soluble vitamin produced in the skin by photo-activiation of


7- dehydrocholesterol .

- fat soluble vitamin with anti-oxdant effects .


- fat soluble vitamin deficiency associated with raised serum
alkaline phosphatase .

- deficiency of this vitamin associated with osteomalasia .

- maternal intake of this vitamin around the time of coception has


been shown to reduce incidance of neural tube defects .

- fat soluble vitamin , deficiency occur within few a days of


cessation of bile secretion .

- this vitamin is a cofactor in synthesis of prothrombin .


ƒ Candida spp
ƒ Herpes simplex virus
ƒ Human immunodeficiency virus
ƒ Gardnerella vaginalis
ƒ Chlamydia trachomatis
ƒ Human papilloma virus
ƒ Treponema pallidum
ƒ Neisseria gonorrhoeae
ƒ Trichomonas vaginalis

• Select the micro-organism described in the following cases:

- An obligate intracellular gram negative bacterium ....

- The cause of lymphogranuloma venereum .........

- Neutrophils containing gram negative diplococci .....


ƒ Double Y Syndrome
ƒ Down’s Syndrome
ƒ Edward’s Syndrome
ƒ Fragile X Syndrome
ƒ Klinefelter’s Syndrome
ƒ Patau’s Syndrome
ƒ Turner’s Syndrome

• Please select the most appropriate syndrome from the options below.
You may use each option once, more than once, or not at all.

- An infant girl is born with a webbed neck, hypertension and


audible ejection systolic murmur.

- A neonate boy is born with cleft lip and palate, low set ears and
polydactyly. The child survives for only 3 weeks.

- An infant boy is born with hypotonia, epicanthic folds and single


palmer creases.
ƒ Adrenal arteries
ƒ Celiac trunk
ƒ Common iliac arteries
ƒ Gonadal (ovarian or testicular) arteries
ƒ Inferior mesenteric artery
ƒ Lumbar arteries
ƒ Median sacral artery
ƒ Renal arteries
ƒ Superior mesenteric artery

• Select the most appropriate option:

- Gives rise to the left gastric, splenic and hepatic arteries.


- Gives rise to the intestinal, middle colic and right colic arteries.

- Gives rise to the left colic, sigmoid and superior rectal arteries.

- An unpaired branch that arises from the aortic bifurcation.

- Arises from the aorta between the level of L1 and L2 vertebrae,


and forms five segmental arteries that do not anastomose.
• B- Chlamydia trachomatis
• C- Herpes simplex virus
• D- Human immunodeficiency virus
• E- Gardnerella vaginalis
• F- Human papilloma virus
ƒ Neisseria gonorrhoeae
ƒ Treponema pallidum
ƒ Trichomonas vaginalis

• Select the micro-organism described in the following cases:

- A thin motile, gram negative spiral shaped bacterium ...

- An anaerobic flagellated protozoan ...

- A single strand RNA virus ..


ƒ ACTH
ƒ CRH
ƒ Dopamine
ƒ FSH
ƒ GnRH
ƒ Growth Hormone
ƒ LH
ƒ Prolactin
ƒ Somatostatin
ƒ TSH

• Please select the correct hormone from the options for each of the
descriptions below. You may use each option once, more than once, or
not at all.

- This hormone acts on cartilage and liver to release IGF-1...

- This hormone promotes iodination of tyrosine residues...

- This hypothalamic hormone inhibits the secretion of growth


hormone...

- In males, this hormone facilitates the generation spermatozoa...

- This hormone inhibits galactorrhoea...



ƒ Deep inguinal
ƒ Inferior mesenteric
ƒ Internal iliac
ƒ Para-aortic
ƒ Superficial inguinal

• Lymph from the following structures passes to the above group of


lymph nodes first. Please choose the most appropriate answer from the
list above.

- Rectal .

- Vulva .

- Hallux .

- Cervix .

- Upper anal canal .


ƒ Bias
ƒ Correlation
ƒ Error
ƒ False positive
ƒ False negative
ƒ Null hypothesis
ƒ Power
ƒ Regression
ƒ Sensitivity
ƒ Specificity

• Which of the above statistical terms is described by the following


statements:
- A random source of inaccuracy...

- A type 2 error...

- Ability to exclude a true negative...


ƒ Clostridium perfringens
ƒ clostridium difficile
ƒ Escherichia coli
ƒ Haemophilus influenzae
ƒ Staphylococcus aureus
ƒ streptococcus pyogenes
ƒ none of the above

• Select the most likely causative organism for the following infections.

- Chronic osteomyelotis after implant surgery ..

- Gas gangrene ..

- Pseudomembranous colitis ..
ƒ Mean
ƒ Median
ƒ Mid-range
ƒ Mode
ƒ Range
ƒ Spread
ƒ Standard Deviation
ƒ Standard Error

• Select the appropriate term from the list for the following definitions.
Each answer may be used once, more than once, or not at all.
- Most frequent value ..

- Spread of estimates of sample means around the true


population mean .

- The measure of spread of values around the mean .

- The mid value when all values are listed in ascending order .

- The sum of all the values divided by the number of values .


ƒ endothelial growth factor
ƒ human chorionic gonadotrophin
ƒ human placental lactogen
ƒ IGF-1
ƒ Insulin
ƒ leptin
ƒ oestrogen
ƒ progesterone
ƒ relaxin

• Select the most appropriate hormone for the following statements.

- The hormone that is the basis of most pregnancy tests.

- Hormone produced by synctiotrophoblast to regulate nutrient


storage in the final stages of pregnancy.

- Hormone that is secreted by decidual cells of the placenta .


- The hormone that is lactogenic, regulates glucose metabolism
and promotes fat breakdown .
• 1)
• ant division of ant. remi of S2-4
• ant division of post.rami of S2-4
• b)genitofemoral nerve
• post division of ant. remi of S2-4
• post division of post. remi of S2-4
• sensory supply of perineal nerve
• inferior rectal nerve
• ischial tuberosity
• h)ischial spine

• A lady is about to deliver and you are about to give her a pudendal
block..

• Q1- what is the root value of pudendal nerve? a


• Q2- what nerve supplies lower part of vagina (I think)? e/a
• Q3- why do u give local skin infiltration before episiotomy? e
• Q4- or what is the nerve supply of peranal area .?

• (2).. If we take the day of fertilization as Day’0’ then…

• Day2
• Day4
• Day 8
• Day 10
• Day12
• Day 14
• Day 18
• Day20
• Day 22
• Day 24
• Day 26
• Day 42 [
• Day 70

• Q1- Which represents 4 cell stage ? day 2


• Q2- conceptus implants completely? Day 10/12
• Q3- fetal heart pulse seen on ultrasound? Day 22
• Q4- vertebra form completely? Day 42
• PCR
• Fish test
• Chromosomal linkage analysis
• Telomer analysis
• Sum hereditary crosslinkage chromosomal option (dnt remember)
• Screen for some known gene mutation
• Sweat test
• Saliva test

• -A family with one kid presumed to have cystic fibrosis however cftr
gene mutation was not detected. Family members willing to give
samples if needed.mother wants definite diagnosis as she is 11wks and
wants to know diagnosis for her baby.parents are cousins with 1st
degree relatives having cystic fibrosis.

- Invasive diagnostics can be applied to reach final results by? ???


- you have Guthrie spot of their son how will u diagnose him for
cystic fibrosis? PCR???
- families in UK with cystic fibrosis how will u find f508? SWEAT
TEST????
• 5)
ƒ Endoplasmic reticulum
ƒ Golgi apparatus
ƒ Lysosomes
ƒ Microtubules
ƒ Mitochondria
ƒ Nucleolus
ƒ Plasma membrane
ƒ Ribosomes
• Select the most appropriate organelle that matches the following
descriptions

- Contains enzymes capable of digesting cells and cellular material


c
- 'Reads' the mRNA and builds protein . h
- Modification of lipids and proteins with storage of material prior
to export out of the cell. a

• (6) screening test for following


- syphilis treponemal antibody test
- syphilis hemaglutination test
- syphilis immobilization test
- HB-electrophorisis
- mcv
- mchc
- MCh
- serum iron
- paul-bennel test

ƒ alpha thalessemia 4
ƒ folic acid deficiency 6
ƒ iron deficiency anemia 8
ƒ primary syphilis in early untreated period 1
• on taking a large protein meal?
ƒ Decrease decrease
ƒ Increase increase
ƒ Increase decrease
ƒ Deacrease increase
ƒ Nochange no change
ƒ Nochange increase
ƒ Nochange decrease
• Insulin & Glucagon excretion what will happened ...? both will increase
in secretions…

• (8)
• 3-hydroxyisovaleric acid .
• 17-alpha hydroxyprogesterone .
• 17-delta hydroxyprogesterone .
• acetic acid .
• cortisol .

• The following level of certain metabolities in amniotic fluid changes


significantly of the fetus has an inborn error of the metabolism .select
the single metabolite from list of the options above whose level in
amniotic fluid is altered by inheretiance disease in item below.
• congentiat adrenal hyperplasia b

• (9)
ƒ Adrenaline
ƒ Calcitonin
• C Cholecystokinin
ƒ Cortisol
ƒ Glucagon
ƒ Insulin
ƒ Progesterone
ƒ Somatostatin
• IT- estosterone

• Select the appropriate hormone from the list for the following
structures that produce it. Each answer may be used once, more than
once, or not at all.
o Adrenal Cortex . d
o Adrenal Medulla . a
o Pancreatic alpha cell . e
o Pancreatic D Cells . h
• Akeratinising stratified squamous epithelium
• Bnon-keratinising stratified squamous epithelium
• Cpseudostratified columnar epithelium
• Dsimple columnar epithelium
• Esimple cuboidal epithelium
• Ftransitional epithelium

• Which of the epithelial types described above are present in the


anatomical regions below:

- ectocervix .
- endocervical canal . simple columnar
- cervical ectropion .
- transformation zone of cervix .
- vagina . A
- uterus . E
- Bladder . F
- Trachea .F
- Labia Majora .
- Bowel . simple columnar
• 11-Fallopian tube . ciliated epi
- Vulva .
- anal canal . simpe columnar

ƒ Autosomal co-dominant
ƒ Autosomal dominant
ƒ Autosomal recessive
ƒ Polygenic
ƒ Single gene defect
ƒ X linked dominant
ƒ X linked recessive

• [Select the most likely mode of inheritance for the following patients’
conditions:

- A 27 -year old female developed gestational diabetes mellitus.


Her uncle and grandmother also had diabetes mellitus D
- A mother is concerned regarding her baby who has developed
fractures which appear to occur with minimal trauma. He has
blue sclera. B
- An 18-year old female underwent caries tooth extraction and
developed profuse bleeding. On history she revealed
menorrhagia. Her mother and her grandfather had the same
disease. G
• EMQ

• 1)
ƒ PGH2
• 2)PGG2
ƒ PGE2
ƒ PGF2alpha
ƒ Arachidonic acid
ƒ Calcium
ƒ Magnesium

• Q1) Synthesis of prostaglandins by?


• Q2) the first Prostaglandin produces?
• Q3) Oxytocin uses this as its intermediate precursor?

• 2) Shapes of the pelvic


• Round shaped pelvic brim with transverse
diameter more than AP
• Round shaped pelvic brim with AP diameter more
than the transverse diameter
• C)Oval shaped pelvic brim with transverse diameter more than AP
• Oval shaped pelvic brim with AP diameter more
than the transverse diameter
• Heart shaped pelvic brim with transverse diameter
more than AP
• Heart shaped pelvic brim with AP diameter more
than the transverse diameter

• Q1) Gynecoid
• Q2) Android

• 3) DNA and RNA


• DNA—Deoxyadenine------Double standed
• Deoxythymine
• Deoxycytosine
• Deoxyguanine

• DNA—Deoxyadenosine------Single standed
• Deoxytyrosine
• Deoxycytosine
• Deoxyguanine

• RNA—Adenine-----Double stranded
• Thymine
• Cytosine
• Uracil
• ) RNA—Adenine-----Single stranded
• Thymine
• Cytosine
• Uracil
• Q1) Messenger RNA?
• Q2) DNA?
• Q3) Human papilloma virus?

Separated recalled items:

• Hormone increased in Prolactinaemia


• Hormone produced by posterior pituatary
• Baby born with right sided umbilical hernia
• Baby born with stiff limbs and mother had oligohydramniosis
• Atomic mass of technetium (99)
• Atomic mass of cobalt (58, but the nearest possible on the list 60)
• Origins of pudendal nerve- ant s2 s4
• Functions of 21 and 17a hydroxlase (convery what to what)
• POstitive predictive value and negative predictive value
• Linear regression
• Hep B and Hep C- DNA/ RNA viruses
• Lidocaine- side effects, shorter that bupivicaine
• Anatomy of pelvic bones
• Fetal skull
• Arterial supply of anus
• Lasers- ? red light faster that UV, ? cut with red light
• MRI- not radioactive, movement of H ions, resolution
• Matching disease with organism- chancre,
• Cancers with serum markers
• Amyloidosis- ?RA ?intracellular
• Bronchodilators
• Staph aureus- coagulase, toxin
• Time closure of neural tube, time mid gut move from abdo wall
• origin of pudendal nerve
• nerve supply of the skin of perinium
• the structure that can differentiate between body of uterus and cervix
in c/s
• struture must be identified during c/s to avoid injury
• embryology: by this time if the mid gut not withdrawl to abdomen the
fetus will devolp exomphilus
• by this time the an and post neroun was closed ,they give many
options
• obstructive jaunduce.
• histopathology of endometrium, hyperplasia & adenocarcinoma.
• HIV, HBV.
• selective COX 2 inhibitors & antiprogesterone.
• steroidogenesis pathways & enzymes.
• tumour markers.
• sarcoidisis.
• asbestosis.
• PUDENDAL NERVE ROOT VALUE S1,2,3 ANT PRIMARY RAMI
• N SUPPLY OF PERIANAL AREA
• LENGTH OF UMBILICAL CORD
• AUTOSOMAL RECESSIVE TAY SACH'S DISEASE
• AUTOSOMAL DOMINANT ACHONDROPLASIA
• ANTIINFLAMMATORY MUSCLE RELAXANT PROSTAGLANDIN
• STAPH AUREUS COAGULASE POSITIVE, PIGMENTED COLONY,GRAM
POSITIVE
• GENTAMYCIN BACTERICIDAL,TOXIC TO 8 TH NERVE

DIURETIC ACTING ON DCT BENDROFLUAZIDE


MCQ questions that actually appeared on MRCOG
part 1exams between 2005-2009.
1) The pelvic surface of the sacrum?
a) gives origin to the piriform muscles
b) gives origin to the levator ani muscles
c) is broader in the male than in the female
d) trnasmit dorsal rami of sacral nerves
e) is in contact with the anal canal

2)Prolactin.
a) release is stimulated by TRH
b) plasma levels are raised in the first trim of preg.
c) release is increased by suckling.
d) maybe produced by decidua.
e) release is inhibited by metoclopromide.

3) The foll disorders and org are correctly paired.


a) opthalmia neonatorum: chlamydia trachomatis
b) chancroid: Haemophilus ducreyi.
c) sleeping sickness: Leishmania donovani.
d) ringworm: Trichenella spiralis.
e) non-specific urethritis: Toxoplasma gondii.

4) The heart rate typically increases in response to:


a) pain.
b) hypoxia.
c)ventilatory expiration
d) increase in Intracranial pressure
e) decrease baroreceptor activity

5) Antibodies.
a) are proteins.
b) are formed in the fetus before 12 weeks of Intrauterine life.
c)have an average molecular weight of around 10 000 daltons.
d) of the rhesus type are genetically transmitted.
e) are produced by the ribosome of plasma cells.
6) the foll disorders have an X linked pattern of inheritance.

a) G6PD Deficeincy.
b) Kleinfelter syndrome
c) adrenogenital syndrome
d) haemophilia B
e) familial hypercholestroleamia.

7)The foll are derived from the urogenital sinus:


a) the bladder trigone.
b) the ureters.
c) the female urethra.
d)greater vestibular glands.
e) paraurethral glands.

In the fetal CVS


a) the heart arises from endoderm
b) the heart is formed by fusion of endocardial tubes.
c) Cardiac pulsation is present by the 30th day after fertilization.
d) oxygenated blood is tranferred to the left atrium through the foramen
ovale.
e) the ductus arteriosus closes during the last 4 weeks of pregnancy.

9)Arginine vasopressin
a) reduces GFR.
b) controls water loss in the Proximal renal tubule.
c) is synthesised by the post pituitary gland.
d) is released in response to rise in plasma osmolality.
e) is released in response to fall in circulating plasma volume.

10) Renin
a) is secreted by the zona glomerulosa of the adrenal coretx.
b) is a proteolytic enzyme.
c) is secreted at an increased rate if the renal perfusion pressure falls.
d) acts upon circulating angiotensinogen.
e) is released in response to an increase in extracellular fluid volume
11) Actinomyces israelii

a) is a rickettsia.
b) forms yellow granules in pus.
c) is a commensal in the mouth.
d) is a commensal in the vagina.
e) is usually resistant to penicillin.

12) Diseases caused by spirochaetes include.


a) Weil's disease.
b) lymphogranuloma venereum
c) pinta
d) Vincent's angina.
e) bilharzia

13) The foll are inherited as autosomal recessive conditions:

a) tuberous sclerosis.
b) phenylketonuria.
c) achondroplasia
d) sickle cell anaemia.
e) Von gierke's disease.

14) Antibodies play an important part in the development of:

a) phagocytosis.
b) Mantoux responce.
c) erythroblastosis fetalis.
d) hyperemesis gravidarum
e) anaphylaxis

15) chemical mediators concerned in the production of an inflammatory


response include:

a) 5-hydroxytryptamine
b) aldosterone.
c) glucocorticoids.
d) bradykinin
e) leukotreines

16) The parathyroid gland.

a) originate from the pharyngeal cleft ectoderm.


b) secrete parathyroid hormone via the chief( principal) cells.
c) secrete calcitonin via the oxyphil cells.
d) may become hyperplastic in the presence of intestinal malabsorption.
e) may develop adenomas in association with islet cell tumour of the
pancreas

17) In the abdominal wall:

a) the rectus abdominis muscle is attched to the crest of the pubis.


b) the post border of the external oblique muscle ends in the linea
semilunaris.
c) the aponeurosis of the external oblique muscle takes part in the
formation of the conjoint tendon.
d) the inferior epigastric artery is a branch of the internal iliac artery.
e) the conjoint tendon blends medially with the anterior layer of the rectus
sheath.

1 The urogenital sinus in the female gives rise to the following:

a) ureter
b) paraurethral glands
c) Bartholin's gland
d) urachus
e)Gartner's duct.

19) Concerning sex hormone:

a) the ovary secretes androstenedione.


b) The ovary secretes testosterone
c) The ovary secretes dihydrotestosterone.
d) SHBG conc. are higher in women more than men
e) Androgens bound to protein have high biological activity.

20) Features of congenital rubella include:

a) excretion of virus by the neonates.


b) hepatomegaly
c) excessive production of growth hormone.
d) cataract
e) deafness.

21) Clomifene citrate:

a) is an anti-androgens.
b) does not stimulate ovulation directly.
c) can produce visual disturbance.
d) is genereally prescribed throughtout the proliferative phase of the
menstrual cycle.
e) in the treatment of ovulation increases the risk of multiple pregnancy.

22) The foll are cytotoxic alkylation agents:

a) Cyclophosphamide.
b) mercaptopurine
c) chlorambucil
d) fluorouracil
e) methotrexate.

Most of the statistic questions posted are right. so will post only the ones
which are missing from wht i remember.

23) If a distribution of results is markedly skewed to the left:

a) the mean is the same as the 50th centile.


b) the same number of values lie on either side of the median.
c) the mode is equal to the median.
d) the student's t test should be used to compare the distribution with
another.
e) logarithmic transformation of the result will produce a distribution
closer to the normal.

24) Corcerning the analysis of clinical trials:

a) the 95% confidence interval indicates the range within which 19 out of
20 values will lie.
b) The P value illustrates how often the result would be expected to occur
by chance.
c)b The conventional level of statistical significance is set of P<0.005
d) In a randomised trial, there must be equal numbers of results in each
arm of the study.
e) A relative risk reduction of 60% is significant irrespective of the value
of P.

25) The following substance are normally synthesized in the liver:

a) glucagon
b) vitamin A
c) cholesterol.
d) immunoglobulins
e) prothrombin

26) The pineal gland:

a) is situated at the anterior end of the 3rd ventricles.


b) is innervated by the parasymphathetic nervous system.
c)produces melatonin.
d)maybe calcified in the adults.
e) is most active during daylight.

27)In congenital adrenal cortical hyperplasia.

a) The commonest deficiency is C18 hydroxylase.


b) plasma cortisol concentration is raised.
c) urinary excretion of 17 oxysteroids is elevated.
d) dexamethasone will suppress the urinary excretion of 17 oxysteroids.
e) there is no virilising effects.

28)Foll are RNA containing virus:

a) coxsackie
b) influneza
c) mumps
d) herpes simplex.
e) cytomegalovirus.

29) Listeria monocytogenes:

a) is a gram negative organism.


b) is sensitive to ampicillin.
c) may cause a transplacental infection.
d) is sexually transmitted.
e) can be cultured from a high vagina swab.

30) actinomyces israelii.

a)is a fungus.
b) forms yellow granules in pus.
c)is a mouth commensal
d) occurs in association with IUCD.
e) is resistant to penicillin.

31) The following drugs may cause enlargement of the fetal thyroid
gland:

a)methyldopa
b) thyroxine
c) carbimazole
d) propranolol
e) propylthiouracil

32) The foll statements about anticoagulant are correct:


a)Heparin inhibits the action of thrombin
b) The action of heparin is antagonised by vitamin K
c) Heparin increases antithrombin III activity.
d) The effects of anticoagulants are decreased by metronidazole.
e) Warfarin is greater than 80% protein bound in plasma

33) The following drugs and side effects are associated:

a) methydopa: depression
b) paracetamol: thromboembolism
c) indomethacin: peptic ulcer
d) prednisolone: osteoporosis.
e) ritodrine: hypoglycemia

34) Haematopoiesis in the fetus:

a) results in nucleated erythrocytes early in development.


b) occurs in the yolk sac in the first month.
c) does not occur in the bone marrow until term.
d) is predominantly hepatic during the 4th month.
e) does not require folic acid.

35)Early blood borne dissemination is characterised feature of:

a) carcinoma of the endometrium.


b) osteosarcoma
c) basal cell carcinoma
d) carcinoma of the cervix
e) choriocarcinoma.

36) The following cells maybe phagocytic:

a) neutrophils.
b) kupffer cells
c) monocytes
d) Hofbauer cells.
e) plasma cell.

37) The following are premalignant conditions:

a) diverticular disease of the large bowel.


b) ulcerative colitis.
c) pulmonary asbestosis.
d) Paget's disease of the bone.
e) condylomata of the vulva

3 In the pituitary gland:

a) the anterior lobe is smaller than the posterior lobe.


b) the posterior lobe is ectodermal in origin.
c) the acidophil cells produces oxytocin.
d) the basophil cells produce growth hormone.
e) the blood supply is derived from the internal carotis artery.

39)The obturator artery:

a)branches from the posterior trunk of the internal iliac artery.


b) passes through the greater sciatic foramen.
c) is crossed by the ureter.
d)supplies the hip joint.
e) may be replaced by a branch of the superior epigastric artery.

40) In congenital adrenal hyperplasia:

a) the commonest cause is a deficiency of 21 hydroxylase.


b) the plasma cortisol conc is increased.
c) there may be excessive secretion of 17 alpha hydroxyprogesterone.
d) sodium retention is characteristic.
e)blood cathecholamine conc are increased.

41) In the fetal lung:

a)bronchial cartilage formation commences at 18-24 weeks of gestation.


b) type II alveolar cells first appear at 16-20 weeks gestation.
c) sphingomyelin is the most common phospholipid present at term
d) phospholipid release is increased by endogenous adrenaline.
e) phospholipid production is decreased by exogenous corticosteroids.

42) In radiotherapy

a) 1 gray is equivalent to 1 joule/kg.


b) the skin usually receives a greater dose of radiation than the underlying
tissues.
c) the major effect of radiation energy is to damage the cytoplasm of the
cell.
d)cells in tissues which are hypoxic are more vulnerable to radiation.
e)Radiation induced changes in tissues may take 6 weeks to develop.

43) Concerning the adrenal glands

a) cortex is derived from neural crest cells.


b) Zona fasiculata secretes aldosterone.
c) Cortical adenomas may cause Cushing syndrome.
d) Neuroblastoma arise in the medulla.
e) Addison's disease may result from autoimmune destruction of the
cortex.

44)The following are autosomal recessive:

a) neurofibromatosis.
b) cystic fibrosis.
c) phenylketonuria
d) polyposis coli
e) sickle cell anaemia

45) Uterine fibroids:

a) are defines histologically as fibromyxomas.


b) arise from endometrial stroma
c) maybe associated with polycythamia.
d) predispose to endometrial hyperplasia.
e) are liable to sarcomatous change in about 5% of cases.

46) growth of the foll tumors are hormone dependent:

a) squamous cell carcinoma of the cervix.


b) breast adenocarcinoma.
c) uterine leiomyoma.
d) prostatic adenocarcinoma
e) testicular carcinoma

47)Surfactants:

a) is formed mainly in the placenta


b) levels in amniotic fluid diminish after 33 weeks of gestation
c) formation can be inferred from the lecithin-sphingomyelin ratio in
amniotic fluid.
d) contains palmitic acid
e) decreases the surface tension in pulmonary alveoli.

4 Intracellular fluid differs from Extracellular fluids in that:

a) it forms the major proportion of total body water.


b) its volume can be measured easily.
c) it has a higher concentration of potassium than of sodium.
d) its volume is regulated primarily by the kidneys.
e) it has a higher phosphate concentration.

49) The Anal canal:

a) has an upper part which is innervated by the inferior hypogastric


plexus.
b) has a lower part which is supplied by the superior rectal artery
c) drains lymph to the superficial inguinal nodes from its upperpart.
d) has its internal sphincter innervated by the infecrior rectal nerve.
e) has a superficial part of its external sphincter attached to the coccyx.
50) The right ovarian artery:

a) Arises from the abdominal aorta above the renal artery.


b) passes posterior to the 3rd (horizontal) part of the duodenum
c) passes post to the genitofemoral nerve.
d) supplies the right ureter.
e) anastomoses with the right uterine artery.

51) Vulva supplied by:

a) Internal pudendal artery.


b) Inferior rectal artery.
c) Genitofemoral artery
d) obturator artery
e) femoral artery..

This is a question outside past papers..All r false Except internal pudendal


artery..

52) The vagina:

a) has an anterior wall longer than the post wall.


b) contains mucus secreting glands in its epithelium.
c) is related in its lower third to the bladder base.
d) during reproductive life has an acid pH.
e) is derived from mesonephric duct.

53) The right ureter:

a) is approx 50cm in length.


b) ia partly covered by duodenum
c) crosses the genitofemoral nerve.
d) enters the bladder anteromedially.
e) receives part of its blood supply from the uterine artery.

54) After birth:


a) allantois froms median umblical ligament
b) umbilical vein forms medial umbilical ligament.
c) umbilical artery forms superior vesical artery.
d) ductus venosus forms the ligamentum teres.
e) ductus arteriosus forms the arch of the aorta.

55)In spermiogenesis:

a) primary spermatocytes undergo reduction division.


b) primary spermatocyte gives rise to 4 spermatids.
c) whole process of spermatogenesis in man takes 6-7 days.
d) grossly abnormal spermatozoa may be found in fertile semen.
e) spermatids are haploid.

56) In the human male, dihydrotestosterone:

a) is a precursor of testosterone.
b) has one-tenth of the ptency of testosterone.
c) is responsible for involution of the Mullerain system.
d) is responsible of the male external genitalia.
e) binds to an intracellular receptor.

57) In Human lactation:

a) estrogens promote development of breast lobules.


b) estrogen promotes milk producing effect of prolactin on the brest,
c) human placental lactogen is essential for milk synthesis.
d) prolactin stimualtes gonadotrophin release.
e) oxytocin causes milk ejection,

58)Concerning testicular hormones:

a) testosterone reduces plasma LH conc.


b) Inhibin stimulates LH production.
c) Estrogen are formed in the testis.
d) Testosterone is converted to dihyrotestosterone by 5 alpha reductase.
e) Testosterone in plasma is predominantly bound to albumin.
59) Epidermal growth factor:

a) is mitogenic.
b) synthesis is stimulated by estradiol.
c) is a steroid molecule.
d) is found in endometrium.
e) binds to a receptor on the nuclear membrane.

60) folic acid:

a) deficiency causes megaloblastic bone marrow..


b) is hydroxycobalamin.
c) is present in green vegetables.
d) is predominantly absorbed from the large intestine.
e) is destroyed by boiling water.

61) Doppler Ultrasound:

a) is used to monitor fetal breathing.


b) is used in fetal HR monitors.
c) can be used to measure blood velocity in the fetus.
d) measure proton relaxation times.
e) requires injection of contrast agents.

62) The following are structural abberation of chromosomes:

a) deletions.
b) inversions.
c) aneuploidy.
d) polyploidy.
e) translocation.

63) Messenger RNA

a) synthesis is dependant on RNA polymerase.


b) is an exact copy of sense DNA.
c) contains exons.
d) is measured by western analysis.
e) translationoccurs in the nucleus.

64) In tumors of the bones:

a) primary malignancy is more common than 2ndry malignancy.


b) osteoma rarely present in skull bones.
c) osteosarcoma is associated with Paget's disease of bone.
d) lymph node metastases are unusual.
e) simple bone cysts have a strong tendency to recur.

Questions about

• endometrial hyperplasia
• side effect of drug (carboplatin)-(taxal)
• arias stella if its specific in pregnancy
• chlamydia
• varicella
• hpv ,wart
• hcg concentration in pregnancy
• citric acid cycle diagram
• hormones diagram levels of, fsh,lh,estrogen,testosterone
• by transvaginal u/s yolk sac appear in which week
• genetic about pcr, fish test
• laser
• totipotent cell,tumour like mass
• voiding presure
• ca requirement in pregnancy
• test used to diagnose folic acid deficency
• thalasthmia
• cervical ectropion cells are columnar cell
• mri contraindicated in early pregnancy
• stellate instability in dna
• transverse abdominus ms attached to lumbar transverse process
• cut s1 lead to autonomic bladder
• cancer which are hormone dependent
-breast
-prostate
-thyroid
• guthre test
• cystic fibrosis and diagnosis
-swet test -karyotyping-fish test
• disinfection and sterilization
• complication of contraceptive pills
• innervation of all pelvic organs.
• lining of ureter
• erythropoeitin and renin q from john duthie
• complications of cisplatin paclitaxel
• statistics -std error calculation mean in a normal distribution
• occipito frontal diameter
• hcg titre at detection by tv u/s 10 at 3 days after fertilization
• crl in scan is 60mm what is gestational age ,crl at term ?
• parietal suture is between
• kuffer cells are –phagocytic
• estradiol receptors
• progestrone receptors -we know that these are intra nuclear but
there were confusing options with intranuclear , one option had
intranuclear kinase .
• anticonvulsant with description of phenytoin
• vitamin deficiency causing hyperemesis
• macrocytic anemia
• xerophtalmia
• cystic fibrosis
• complement causes
• b thalasemia detection
• folate def detection
• basal cell ca - local malignant i think
• tumor like –hamaroma
• cystic fibrosis guthic spot test
• receptors in apoptosis caspa
• arias stella reaction
• blood picture in PET and in preg is inflam like
• in statistics mostly repeated questions..example given..calculate
sensitivity,specificity,+ve predictive value..in right sided skewed
distribution curve median will be on which side of mean?
• microsatellites alleles
- cd4
-nk cells
• down syndrome
• pulmonary embolism

• apoptosis or endometrial receptors.


• anal canal
• vagina
• ovary
• Phenylketonuria
• inhibin
• passive transport
• erythropoietin
• renin
• hcg levels
• syphylis
• rubella
• hormone dependent malignancies(testicular carc._sertoli leydig-
clear cellcarc.ofkidney-thyroid carc.)
• pigd
• congenital heart
• ring Y chromosome
• passive diffussion not depend on .
• concentration gradient
• molecular size
• following tissue are capable of regeneration

spinal cord
liver
epidermis
myocardium
bone marrow
• double blind trial
• level of HCG in the urine pregnancy test
• APOPTOSIS
• methods of disinfection and
• BOWIE DICK test
• progesterone receptors
• syphilis , toxoplasma , hpv , immune responses
• down syndrome
associated with duodenal atresia true
associated with ambig genetalia false
only maternal chromosome F
only paternal chromosome F
• oogonia
• miosis starts at puberty
• miosis before mitosis
• during S phase chromosomes are doubled.
• to calculate sensitivity and specificity.
• standard deviation
• standard error of the mean.
• mean, mode median in normal distribution and skewed data.
• T test
• MRI... type of radiation,
causes effect in pregnancy
women should be placed left tilted during scan.
• dose of radiation
. 1 gray = joules/ kg
. absorbed dose.
• dopplers USG

EMQ questions that actually appeared on MRCOG
part 1exams between 2007-2009.

EMQ about

• vitamins (hyperemesis), vit B6 and (xerophthalmia).


• paclitaxel carboplatin side effects
• complex partial seiizure gum hypertrophy,acne ,facial
coarsening,vit K TO BE given..?drug used
• oestrogen progesterone receptor site
• vitamins-xerophthalmia..morning sickness..macrocytic anaemia.
• rubella incubation period,specific immunity within 15 days..
• HEPATOCELLULAR carcinoma,..??
• lining epithelium of ureter,..?
• mantoux rection..
• ovulatory DUB
• aneurysm- 10% due 2 inflam.,syphilitic aneu,thoracic
vessels,marfan recessive.
• if a mother has a child with cystic fibrosis ,she is pregnant and will
do amniocentesis
a. which test to be done (every possibility mentioned)
b. all relatives will volunteer to do gutic spot test ,which test you
will carry on the blood .
• crown \heel lenghth at birth
crown/rump length 6 cm =? Weeks
• type of hpv causing benign wart
type of hpv causing cancer cx
• .B cell
.plasma cell
.dendritic cell
.mast cells
. t cell

QUESTION
. ANTIBODY PRODUCTION?
. ANTIGEN PRESENTATION?
• Options
1. oval inlet transversal diameter longer than anterposterior .
2. oval inlet anterposterior dia longer than trans.
3. heart shape inlet anterposterior dia. longer than trans.
4. heart shape inlet trans. diam. longer than anterposterior
5. rounded inlet .. T>A.
6. rounded inlet ..A > T
Qus.
a. Gynaecoid pelvis.
b. Android pelvis .

------------------------------------------------------------

a. Adenine monophosphate
Guanine monophosphate
Cytosine monophosphate duple helix
Thymine monophosphate

b. Adenine monophosphate
Guanine monophosphate
Cytosine monophosphate duple helix
Uracil monophosphate

c. Adenine monophosphate
Guanine monophosphate
Cytosine monophosphate single helix
Uracil monophosphate

d. Adenine monophosphate
Guanine monophosphate
Cytosine monophosphate single helix
Thymine monophosphate

e. Adenine monophosphate
Guanine monophosphate
Cytosine monophosphate duple helix
Thymine monophosphate

f. Diadenine monophosphate
Diguanine monophosphate
Dicytosine monophosphate duple helix
Diuracil monophosphate

c. DiAdenine monophosphate
DIGuanine monophosphate
DICytosine monophosphate single helix
DIUracil monophosphate

d. DIAdenine monophosphate
DIGuanine monophosphate
DICytosine monophosphate single helix
DIThymine monophosphate

e. Adenine monophosphate
Guanine monophosphate
Cytosine monophosphate duple helix
Thymine monophosphate

b. Adenine monophosphate
Guanine monophosphate
Cytosine monophosphate duple helix
Uracil monophosphate
c. Adenine monophosphate
Guanine monophosphate
Cytosine monophosphate single helix
Uracil monophosphate

d. Adenine monophosphate
Guanine monophosphate
Cytosine monophosphate single helix
Thymine monophosphate

1. DNA.
2. mRNA
3. Nucliac acid o HPV.
• Options

Somatotroph, acidophil
Somatotroph, basophil
lactotroph, acidophil
lactotroph, basophil
corticotroph, acidophil
corticotroph, basophil
gonadotroph, acidophil
gonadotroph, basophil
thyrotroph, acidophil
thyrotroph, basophil

Q1. Growth hormone


Q2. Prolactin

number 1:

a. pelvic brim oval in shape, transverse diameter more than anteropost


diameter.
b. pelvic brim oval in shape, anteropost diameter more than transverse
diameter.
c. pelvic brim heart shaped, transverse diameter more than anteropost
diameter.
d. pelvic briim heart shaped, anteropost diameter more than transverse
diameter.
e. pelvic brim circular, transverse diameter more than anteropost
diameter.
f. pelvic brim circular, anteroposr diameter more than transverse.

1) gynecoid pelvis.
2) android pelvis.

number 2:

a.double stranded, adenosine monophosphate, guanine monophosphate,


cytosine monophosphate, uracil monophosphate.
b. double stranded, adenosine monoph, guanine monph, cytosine
monoph, thymine monoph.
c. single stranded, adenine monoph, guanine monoph, cytosine monoph,
thymine monoph.
d. single stranded, adenine monoph, guanine monoph, cytosine monoph,
uracil monoph.
e. single stranded, deoxyadenine monoph, deoxyguanine monoph,
deoxycytosine monoph, deoxyuracil monoph.
f. single stranded, deoxyadenine monoph, deoxyguanine monoph,
deoxycytosine monoph, deoxythymine monoph.
g. double stranded, deoxyadenine monoph, deoxyguanine monoph,
deoxycytosine monoph, deoxyuracil monoph
h. double stranded, deoxyadenine monoph, deoxyguanine monoph,
deoxycytosine monoph, deoxythymine monoph,

1) DNA.
2) RNA.
3) genome of HPV.

number 3:

a. edwards syndrome.
b. exomphalus.
c. gastroschiasis.
d. patau's syndrome.
e. down's syndrome.
f. kleinfeltr's syndrome.

1) abnormality in the anterior abdominal wall, usually to the right of the


umbilicus, other genetic abnormalities rarely associated.
2) abnormality in chromosome 18.

number 4:

a. azithromycin.
b. amoxycillin.
c. cefuroxime.
d. benzylpenicillin.
e. metronidazole.

1) best treatment of acute upper UTI in third trimester.


2)treatment of chlamydia in a non-pregnant woman.

number 5:

a. L1,2,3
b. L1
c. L2,3,4
d. L1,2
e. L3,4,5

1) ilioinguinal nerve.
2) genitofemoral nerve.

number 6:

a. streptococus, aerobic, G+ve.


b. streptococus, anaerobic, G+ve.
c. staphylococus, aerobic, G+ve.
d. staphlococus, anaeobic, G-ve.
e. pseudomonas, aerobic, G+ve.
f. pseudomonas, aerobic G-ve.
g.pseudomonas, anaerobic, G-ve.

1) toxic shock syndrome.


2) hospital cross infection.

number 7:

a. CMV.
b. listeria monocytogens.
c. treponame pallidum.
d. HIV.
e. human leucocytic virus.
f. staphlococus aureus.
g. streptococus.
h. varicella zoster.

1) a pregnant woman developed a "flue-like" illness with fever and


general malaise, her baby was born with hepatosplenomegaly and
jaundice.
2) a woman has a history of 2 previous stillbirths, is now pregnant and at
37 weeks complained of fever and rash, her son was born healthy but at
age of one year he had abnormal incisors and later deafness.

number 8:

a. hepatitis C, DNA.
b. hepatitis C, RNA.
c. herpes simplex.
d. HPV.
e. i cant recall the other options.

1) associated with high incidence of hepatocellular carcinoma.


2) papular skin rash.

number 9:
a. estrogen.
b. progesterone.
c. DHEA-sulphate.
d. cortisol.
e. cortisone.

1) hormone maintains uterine quisence throughout pregnancy.


2) hormone produced by the placenta and fetal adrenal gland, promotes
fetal lung maturation.

number 10:

a. allantois.
b. cloaca.
c. mesonephros.
d. yolk sac.

1) rectum develops from the posterior portion if this structure.


2) germ cells arise from it.

number 11:

a. B cell
b. T CD4 cell
c. T CD8 cell
d. dendritic dells.

1) major antigan presenting cell.


2) could not recall it.

EMQ

1)
1) PGH2
2)PGG2
3) PGE2
4) PGF2alpha
5) Arachidonic acid
6) Calcium
7) Magnesium

Q1) Synthesis of prostaglandins


Q2) the first Prostaglandin produces.
Q3) Oxytocin uses this as its intermediate precursor

2) Shapes of the pelvic

A) Round shaped pelvic brim with transverse diameter more than AP


B) Round shaped pelvic brim with AP diameter more than the transverse
diameter
C)Oval shaped pelvic brim with transverse diameter more than AP
D) Oval shaped pelvic brim with AP diameter more than the transverse
diameter
E) Heart shaped pelvic brim with transverse diameter more than AP
F) Heart shaped pelvic brim with AP diameter more than the transverse
diameter

Q1) Gynecoid
Q2) Android

3) DNA and RNA


A) DNA—Deoxyadenine------Double standed
Deoxythymine
Deoxycytosine
Deoxyguanine

B) DNA—Deoxyadenosine------Single standed
Deoxytyrosine
Deoxycytosine
Deoxyguanine

C) RNA—Adenine-----Double stranded
Thymine
Cytosine
Uracil
D) ) RNA—Adenine-----Single stranded
Thymine
Cytosine
Uracil

Q1) Messenger RNA


Q2) DNA
Q3) Human papilloma virus

4)
1) L1
2) L1, L2
3) L1, L2, L3
4) L1, L2, L3, L4
5) L2, L3, L4
6) L3, L4

Q1) Ilioinguinal
Q2) Genitofemoral.

) Given in a single dose


A) AZITHROMYCIN
B) Amoxicillin
C) Cephalosporin
D) Vancomycin
E) Gentamycin
F) Benzylpenicillin

Q1) A female with a chlamydial infection, and non-pregnant, which is the


best medication?
Q2) A female presents with acute upper UTI?

6)Infections
A) Staph
B) Strept
C) Pseudomonas aeruginosa

Q1) most common cause wound infection in hospital


Q2) hospital acquired infection

7) Statistics
Another question on calculation for specificity and sensitivity.. People
preparing for March 2010, please learn the formulas well.

8-THE FOLLOWING STATEMENTS ABOUT THE ADRENAL


GLAND ARE CORRECT
a-they lie anterior to the diaphragm ...........True.
b-the left adrenal gland lies behind the pancreas.....True.
C-lymphatic drainage is to the superficial inguinal nodes.....False
d-the adrenal cortex contains chromaffin cells .........False.
e-the adrenal medulla is derived from mesoderm ....False

the ovary
a- is attached to the ant surface of the broad ligament
b- lies on the genitofemoral nerve
c- lies in the angle between the ureter and the external iliac vessel
d- - has visceral afferent fibres from the pelvic splanchnic nerve
e- has lymphatic drainage to the superficial inguinal lymph nodes

concerning the uterus


a- it is formed from the mesonephric duct
b- it has a lymphatic drainage in part to the inguinal glands
c- the uterine artery passes below the ureter
d- the uterine vein communicate with the vesical plexus of veins
e-pain sensation from the body of uterus is carried by the pelvic
splanchnic nerve

about this stem am not sure if it was like that in the exam

Obturator artery
a- branches from the posterior trunk of the internal iliac artery
b- passes through the greater sciatic foramen
c- is crossed by the ureter
d- supplies the hip joint
e- may be replaced by a branch of the superior epigastric artery

The external iliac artery

a-enters the thigh anterior to the inguinal ligament


b- at its origin is crossed by ureter
c- at its origin is crossed by ovarian vessels
d-lies medial to the external iliac vein at its distal end
e-give rise to the deep external pudendal artery .

The pelvic splanchnic nerves..

A- Are derived from the posterior rami of the sacral spinal nerves
B- Supply afferent fibres
C- Unite with branches of the synpathetic pelvic plexus
D- Supply the ascending colon with motor fibres
E- Supply the uterus with parasympathetic fibres

In the anterior abdominal wall...

A- Rectal muscle is intersected tranversely by three bands


B- The posterior rectus sheath below the arcuate line consists of
transversalis fascia only
C- Above the costal margin the posterior rectus sheath is deficient
D- The superior epigastric artery arises from the internal thoracic artery.
E- The inferior epigastric artery arises from the femoral artery

The pelvic surface of the sacrum.

A- Gives origin to the piriforms muscle


B- Gives origin to the levator ani muscle
C- Is broader in the male than in the female
D- Transmits the dorsal remi of sacral nerves
E- Is in contact with the anal canal.
The rectum

A- Is supplied in part by the inferior rectal artery


B- Is innervated by the inferior rectal nerve
C- Is lined by stratified squamous epithelium
D- Has its lymphatic drainage to the superficial inguinal nodes
E- Possesse a complete layer of longitudinal muscle

The Spleen

A- has a notched posterior border


B- lies in front of the costo-diaphragmatic recess
C- is in contact with the body of the pancrease
D- lies under the cover of the 9th to the 11th ribs
E- is innervated from the renal plexus

THE VULVA

a-internal pudendal nerve


b-anterior cutaneous of thigh
c-inferior rectal
d-illioinguinal
e-obturator

the pineal gland

a-is situated at the anterior end of the third ventricle


b-is innervated by parasympathetic nervous system
c-produce melatonin
d-may be calcified in the adult
e-is most active during daylight

screening test for following


1- syphilis treponemal antibody test
2- syphilis hemaglutination test
3- syphilis immobilization test
4- HB-electrophorisis
5- mcv
6- mchc
7- MCh
8- serum iron
9- paul-bennel test

Q1) Thalassemia
Q2) Syphilis

Sep. 2009
Options
1. oval inlet transversal diameter longer than anterposterior .
2. oval inlet anterposterior dia longer than trans.
3. heart shape inlet anterposterior dia. longer than trans.
4. heart shape inlet trans. diam. longer than anterposterior
5. rounded inlet .. T>A.
6. rounded inlet ..A > T
Qus.
a. Gynaecoid pelvis.
b. Android pelvis .

------------------------------------------------------------

a. Adenine monophosphate
Guanine monophosphate
Cytosine monophosphate duple helix
Thymine monophosphate

b. Adenine monophosphate
Guanine monophosphate
Cytosine monophosphate duple helix
Uracil monophosphate

c. Adenine monophosphate
Guanine monophosphate
Cytosine monophosphate single helix
Uracil monophosphate

d. Adenine monophosphate
Guanine monophosphate
Cytosine monophosphate single helix
Thymine monophosphate

e. Diadenine monophosphate
Diguanine monophosphate
Dicytosine monophosphate duple helix
Diuracil monophosphate

f. Diadenine monophosphate
Diguanine monophosphate
Dicytosine monophosphate duple helix
Dithymine monophosphate

g. DiAdenine monophosphate
DIGuanine monophosphate
DICytosine monophosphate single helix
DIUracil monophosphate

h. DIAdenine monophosphate
DIGuanine monophosphate
DICytosine monophosphate single helix
DIThymine monophosphate

1. DNA.
2. mRNA
3. Nucliac acid o HPV.

STRUCTURE OF BASES AND NOT THE NUCLEOTIDE OR


NUCLEOSIDES

gynecoid and android pelvis and the DNA

1)
a) ant division of ant. remi of S2-4
b) ant division of post.rami of S2-4
b)genitofemoral nerve
c) post division of ant. remi of S2-4
d) post division of post. remi of S2-4
e) sensory supply of perineal nerve
f) inferior rectal nerve
g) ischial tuberosity
h)ischial spine

A lady is about to deliver and you are about to give her a pudendal
block..

Q1- what is the root value of pudendal nerve?


Q2- what nerve supplies lower part of vagina (I think)?
Q3- why do u give local skin infiltration before episiotomy?
Q4- or what is the nerve supply of peranal area .?

(2).. If we take the day of fertilization as Day’0’ then…

Day2
Day4
Day 8
Day 10
Day12
Day 14
Day 18
Day20
Day 22
Day 24
Day 26
Day 42 [
Day 70

Q1- Which represents 4 cell stage ?


Q2- conceptus implants completely?
Q3- fetal heart pulse seen on ultrasound?
Q4- vertebra form completely?

(3)
A Cytoplasmic transcription factor receptor
B G protein coupled receptor on cell membrane
C G protein coupled receptor on Golgi complex
D Mega subunit ligand gated ion channel
E Multisubunit antibody receptor on cell membrane
F Multisubunit ligand gated ion channel on cell membrane
G Nuclear protein kinase receptor
H Nuclear transcription factor receptor
I Protein kinase receptor on cell membrane
J Receptor protein complex (intracytoplasmic)
K Transcription factor receptor on cell membrane

Select the receptor which binds the molecules referred to in the


itemS below. Each option may be used once, more than once or not
at all.

1- insulin .
2- progesterone .
3- ostradiol .
4- prostaglandin .

(4)
PCR
Fish test
Chromosomal linkage analysis
Telomer analysis
Sum hereditary crosslinkage chromosomal option (dnt remember)
Screen for some known gene mutation
Sweat test
Saliva test

-A family with one kid presumed to have cystic fibrosis however


cftr gene mutation was not detected. Family members willing to
give samples if needed.mother wants definite diagnosis as she is
11wks and wants to know diagnosis for her baby.parents are
cousins with 1st degree relatives having cystic fibrosis.

1- Invasive diagnostics can be applied to reach final results by?


2- you have Guthrie spot of their son how will u diagnose him for
cystic fibrosis?
3- families in UK with cystic fibrosis how will u find f508?

(5)
A- Endoplasmic reticulum
B- Golgi apparatus
C- Lysosomes
D- Microtubules
E- Mitochondria
F- Nucleolus
G- Plasma membrane
H- Ribosomes

Select the most appropriate organelle that matches the following


descriptions

1- Contains enzymes capable of digesting cells and cellular


material
2- 'Reads' the mRNA and builds protein .
3- Modification of lipids and proteins with storage of material prior
to export out of the cell.

(6)[i][b] screening test for following


1- syphilis treponemal antibody test
2- syphilis hemaglutination test
3- syphilis immobilization test
4- HB-electrophorisis
5- mcv
6- mchc
7- MCh
8- serum iron
9- paul-bennel test[/b][/i]

A- alpha thalessemia
B- folic acid deficiency
C- iron deficiency anemia
D- primary syphilis in early untreated period

(7) on taking a large protein meal?


A- Decrease decrease
B- Increase increase
C- Increase decrease
D- Deacrease increase
E- Nochange no change
F- Nochange increase
G- Nochange decrease

Insulin & Glucagon excretion what will happened ...?

(8)
a- 3-hydroxyisovaleric acid .
b- 17-alpha hydroxyprogesterone .
c- 17-delta hydroxyprogesterone .
d- acetic acid .
e- cortisol .

The following level of certain metabolities in amniotic fluid


changes significantly of the fetus has an inborn error of the
metabolism .select the single metabolite from list of the options
above whose level in amniotic fluid is altered by inheretiance
disease in item below.
congentiat adrenal hyperplasia

(9)
A- Adrenaline
B- Calcitonin
C Cholecystokinin
D- Cortisol
E- Glucagon
F- Insulin
G- Progesterone
H- Somatostatin
IT- estosterone

Select the appropriate hormone from the list for the following
structures that produce it. Each answer may be used once, more
than once, or not at all.

1- Adrenal Cortex .
2- Adrenal Medulla .
3- Pancreatic alpha cell .
4- Pancreatic D Cells .

(10)
AAnterior cerebral artery
BAnterior communicating artery
CAnterior inferior cerebellar artery
DBasilar artery
EInternal auditory artery
FInternal carotid artery
GMiddle cerebral artery
HPosterior cerebral artery
IPosterior communicating artery
JSuperior cerebellar artery
KVertebral artery
Complete the diagram of the circle of Willis using the options
given:
this question came with diagrame in september 2007
so you should study circle of weils very well if I get the pictures I
will post it .

Akeratinising stratified squamous epithelium


Bnon-keratinising stratified squamous epithelium
Cpseudostratified columnar epithelium
Dsimple columnar epithelium
Esimple cuboidal epithelium
Ftransitional epithelium

Which of the epithelial types described above are present in the


anatomical regions below:

1- ectocervix .
2- endocervical canal .
3- cervical ectropion .
4- transformation zone of cervix .
5- vagina .
6- uterus .
7- Bladder .
8- Trachea .
9- Labia Majora .
10- Bowel .
11-Fallopian tube .
12- Vulva .
13- anal canal .

A- Autosomal co-dominant
B- Autosomal dominant
C- Autosomal recessive
D- Polygenic
E- Single gene defect
F- X linked dominant
G- X linked recessive

[Select the most likely mode of inheritance for the following patients’
conditions:

1- A 27 -year old female developed gestational diabetes mellitus. Her


uncle and grandmother also had diabetes mellitus2- A mothr is concerned
regarding her baby who has developed fractures which appear to occur
with minimal trauma. He has blue sclera.
3- An 18-year old female underwent caries tooth extraction and
developed profuse bleeding. On history she revealed menorrhagia. Her
mother and her grandfather had the same disease.

A- WARFARIN .
B- HEPARIN .
C- AMPICILLIN .
D- METHYLDOPA .
E- PENCILLIN .
F- CARBIMAZEPINE .
G- METRONIDAZOLE .
H- NON OF THE ABOVE .
I- ALL OF THE ABOVE .

For each of the following choose the single most appropirate statement
from the above list of options .
1- Drug contraindicated in breast feeding .
2- Drug does not cross the placenta .

A.
Calcitonin
B.
Cortisol

C.
Glucagon

D.
Growth hormone

E.
Insulin

F.
Oestradiol 17-¦Â¦Â

G.
Oxytocin

H.
Parathyroid hormone

I.
Prolactin

J.
Thyroxine

Instructions: For each action described below, choose the single


most likely causative hormone from the above list of options. Each
option may be used once, more than once, or not at all.

Question 3: Stimulates deposition of cartilage at the ends of bones


Question 4: Raises blood glucose levels through the breakdown of
fat and protein
Question 5: Antagonises the effect of parathyroid hormone to
minimise bone density loss
Question 6: Stimulates the release of milk from the breast
Question 7: Stimulates the urinary secretion of calcium

Options
A.
Common iliac artery
B.
External iliac artery
C.
Inferior epigastric artery
D.
Inferior vesical artery
E.
Internal iliac artery
F.
Middle rectal artery
G.
Ovarian artery
H.
Superior vesical artery
I.
Umbilical artery
J.
Uterine artery
K.
Vaginal artery

Instructions: After a forceps delivery a 30-year-old primigravida sustains


a primary post partum haemorrhage of four litres. Although the uterus
appears well contracted the bleeding continues, and a decision is made to
identify and treat the bleeding point radiologically. For each question
posed below, choose the single most appropriate option from the above
list. Each option may be used once, more than once or not at all.
Question 8: What vessel runs up the broad ligament?
Question 9: Which other vessel arises from the uterine artery?
Question 10: Which other vessel anastomoses with the uterine artery?
Question 11: Which vessel does the uterine artery arise from?
Question 12: Which vessel runs anterior and superior to the ureter?

A- vitamin A
B- vitamin E
C- vitamin D
D- vitamin B12
E- vitamin B6
F- vitamin C
G- folic acid
H- riblflavine
I- vitamin K
J- thiamine

1- fat soluble vitamin sysnthesised in the intestinal wall


from the beta carotene .

2- fat soluble vitamin synthesised by large intestinal


bacteria .

3- deficiency of this vitamin in childhood causes rickets .

4- fat soluble vitamin synthesis by the kidney is regulated


by parathyroid hormone .

5- deficiency of this vitamins typicaly occurs in women


with hyperemesis gravidarum .

6- water soluble vitamin with anti-oxidant effects .

7- absorption of this vitamin requires the presence of


intrinsic factor .

8- deficiency of this vitamin causes megaloblastic anaemia


and neurological disorders .

9- fat soluble vitamin produced in the skin by photo-


activiation of 7- dehydrocholesterol .

10- fat soluble vitamin with anti-oxdant effects .

11- fat soluble vitamin deficiency associated with raised


serum alkaline phosphatase .

12- deficiency of this vitamin associated with osteomalasia


.

13- maternal intake of this vitamin around the time of


coception has been shown to reduce incidance of neural
tube defects .

14- fat soluble vitamin , deficiency occur within few a days


of cessation of bile secretion .

15- this vitamin is a cofactor in synthesis of prothrombin .


A- Candida spp
B- Herpes simplex virus
C- Human immunodeficiency virus
D- Gardnerella vaginalis
E- Chlamydia trachomatis
F- Human papilloma virus
G- Treponema pallidum
H- Neisseria gonorrhoeae
I- Trichomonas vaginalis

Select the micro-organism described in the


following cases:
1- An obligate intracellular gram negative
bacterium ....

2- The cause of lymphogranuloma venereum


.........

3- Neutrophils containing gram negative


diplococci .....
A- Double Y Syndrome
B- Down’s Syndrome
C- Edward’s Syndrome
D- Fragile X Syndrome
E- Klinefelter’s Syndrome
F- Patau’s Syndrome
G- Turner’s Syndrome

Please select the most appropriate syndrome


from the options below. You may use each
option once, more than once, or not at all.

1- An infant girl is born with a webbed neck,


hypertension and audible ejection systolic
murmur.

2- A neonate boy is born with cleft lip and


palate, low set ears and polydactyly. The child
survives for only 3 weeks.

3- An infant boy is born with hypotonia,


epicanthic folds and single palmer creases.
A- Adrenal arteries
B- Celiac trunk
C- Common iliac arteries
D- Gonadal (ovarian or testicular) arteries
E- Inferior mesenteric artery
F- Lumbar arteries
G- Median sacral artery
H- Renal arteries
I- Superior mesenteric artery

Select the most appropriate option:

1- Gives rise to the left gastric, splenic and hepatic


arteries.

2- Gives rise to the intestinal, middle colic and right


colic arteries.

3- Gives rise to the left colic, sigmoid and superior


rectal arteries.

4- An unpaired branch that arises from the aortic


bifurcation.

5- Arises from the aorta between the level of L1 and


L2 vertebrae, and forms five segmental arteries that
do not anastomose.
A- Candida spp
B- Chlamydia trachomatis
C- Herpes simplex virus
D- Human immunodeficiency virus
E- Gardnerella vaginalis
F- Human papilloma virus
G- Neisseria gonorrhoeae
H- Treponema pallidum
I- Trichomonas vaginalis

Select the micro-organism described in the following


cases:

1- A thin motile, gram negative spiral shaped bacterium


...
2- An anaerobic flagellated protozoan ...

3- A single strand RNA virus ..


A- ACTH
B- CRH
C- Dopamine
D- FSH
E- GnRH
F- Growth Hormone
G- LH
H- Prolactin
I- Somatostatin
J- TSH

Please select the correct hormone from the options for each of
the descriptions below. You may use each option once, more
than once, or not at all.

1- This hormone acts on cartilage and liver to release IGF-1...

2- This hormone promotes iodination of tyrosine residues...

3- This hypothalamic hormone inhibits the secretion of growth


hormone...

4- In males, this hormone facilitates the generation


spermatozoa...

5- This hormone inhibits galactorrhoea...


B- A- Deep inguinal
B- Inferior mesenteric
C- Internal iliac
D- Para-aortic
E- Superficial inguinal

Lymph from the following structures passes to the above group


of lymph nodes first. Please choose the most appropriate answer
from the list above.

1- Rectal .

2- Vulva .

3- Hallux .

4- Cervix .

5- Upper anal canal .


A- Bias
B- Correlation
C- Error
D- False positive
E- False negative
F- Null hypothesis
G- Power
H- Regression
I- Sensitivity
J- Specificity

Which of the above statistical terms is described by the following


statements:

1- A random source of inaccuracy...

2- A type 2 error...

3- Ability to exclude a true negative...


A- Clostridium perfringens
B- clostridium difficile
C- Escherichia coli
D- Haemophilus influenzae
E- Staphylococcus aureus
F- streptococcus pyogenes
G- none of the above
Select the most likely causative organism
for the following infections.

1- Chronic osteomyelotis after implant


surgery ..

2- Gas gangrene ..

3- Pseudomembranous colitis ..
A- Mean
B- Median
C- Mid-range
D- Mode
E- Range
F- Spread
G- Standard Deviation
H- Standard Error

Select the appropriate term from the list for the following
definitions. Each answer may be used once, more than once, or
not at all.

1- Most frequent value ..

2- Spread of estimates of sample means around the true


population mean .

3- The measure of spread of values around the mean .

4- The mid value when all values are listed in ascending order .

5- The sum of all the values divided by the number of values .


A- endothelial growth factor
B- human chorionic gonadotrophin
C- human placental lactogen
D- IGF-1
E- Insulin
F- leptin
G- oestrogen
H- progesterone
I- relaxin

Select the most appropriate hormone for


the following statements.

1- The hormone that is the basis of most


pregnancy tests.

2- Hormone produced by
synctiotrophoblast to regulate nutrient
storage in the final stages of pregnancy.

3- Hormone that is secreted by decidual


cells of the placenta .

4- The hormone that is lactogenic,


regulates glucose metabolism and
promotes fat breakdown .

PCR
Fish test
Chromosomal linkage analysis
Telomer analysis
Sum hereditary crosslinkage chromosomal option (dnt remember)
Screen for some known gene mutation
Sweat test
Saliva test

-A family with one kid presumed to have cystic fibrosis however cftr
gene mutation was not detected. Family members willing to give samples
if needed.mother wants definite diagnosis as she is 11wks and wants to
know diagnosis for her baby.parents are cousins with 1st degree relatives
having cystic fibrosis.
1- Invasive diagnostics can be applied to reach final results by? ???
2- you have Guthrie spot of their son how will u diagnose him for cystic
fibrosis? PCR???
3- families in UK with cystic fibrosis how will u find f508? SWEAT
TEST????

5)
A- Endoplasmic reticulum
B- Golgi apparatus
C- Lysosomes
D- Microtubules
E- Mitochondria
F- Nucleolus
G- Plasma membrane
H- Ribosomes

Select the most appropriate organelle that matches the following


descriptions

1- Contains enzymes capable of digesting cells and cellular material c


2- 'Reads' the mRNA and builds protein . h
3- Modification of lipids and proteins with storage of material prior to
export out of the cell. a

(6) screening test for following


1- syphilis treponemal antibody test
2- syphilis hemaglutination test
3- syphilis immobilization test
4- HB-electrophorisis
5- mcv
6- mchc
7- MCh
8- serum iron
9- paul-bennel test
A- alpha thalessemia 4
B- folic acid deficiency 6
C- iron deficiency anemia 8
D- primary syphilis in early untreated period 1

(7) on taking a large protein meal?


A- Decrease decrease
B- Increase increase
C- Increase decrease
D- Deacrease increase
E- Nochange no change
F- Nochange increase
G- Nochange decrease

Insulin & Glucagon excretion what will happened ...? both will increase
in secretions…

(8)
a- 3-hydroxyisovaleric acid .
b- 17-alpha hydroxyprogesterone .
c- 17-delta hydroxyprogesterone .
d- acetic acid .
e- cortisol .

The following level of certain metabolities in amniotic fluid changes


significantly of the fetus has an inborn error of the metabolism .select the
single metabolite from list of the options above whose level in amniotic
fluid is altered by inheretiance disease in item below.
congentiat adrenal hyperplasia b

(9)
A- Adrenaline
B- Calcitonin
C Cholecystokinin
D- Cortisol
E- Glucagon
F- Insulin
G- Progesterone
H- Somatostatin
IT- estosterone

Select the appropriate hormone from the list for the following structures
that produce it. Each answer may be used once, more than once, or not at
all.

1- Adrenal Cortex . d
2- Adrenal Medulla . a
3- Pancreatic alpha cell . e
4- Pancreatic D Cells . h

A- WARFARIN .
B- HEPARIN .
C- AMPICILLIN .
D- METHYLDOPA .
E- PENCILLIN .
F- CARBIMAZEPINE .
G- METRONIDAZOLE .
H- NON OF THE ABOVE .
I- ALL OF THE ABOVE .

For each of the following choose the single most


appropirate statement from the above list of options .
1- Drug contraindicated in breast feeding . H
2- Drug does not cross the placenta B
A.
Calcitonin

B.
Cortisol
C.
Glucagon

D.
Growth hormone

E.
Insulin

F.
Oestradiol 17-¦Â¦Â

G.
Oxytocin

H.
Parathyroid hormone

I.
Prolactin

J.
Thyroxine

Instructions: For each action described below, choose the


single most likely causative hormone from the above list
of options. Each option may be used once, more than
once, or not at all.

Question 3: Stimulates deposition of cartilage at the ends


of bones GH
Question 4: Raises blood glucose levels through the
breakdown of fat and protein Insulin
Question 5: Antagonizes the effect of parathyroid
hormone to minimize bone density loss calcitonin
Question 6: Stimulates the release of milk from the breast
Oxytocin
Question 7: Stimulates the urinary secretion of calcium
calcitonin..?

B. Options
A.
Common iliac artery
B.
External iliac artery
C.
Inferior epigastric artery
D.
Inferior vesical artery
E.
Internal iliac artery
F.
Middle rectal artery
G.
Ovarian artery
H.
Superior vesical artery
I.
Umbilical artery
J.
Uterine artery
K.
Vaginal artery

Instructions: After a forceps delivery a 30-year-old


primigravida sustains a primary post partum haemorrhage
of four litres. Although the uterus appears well contracted
the bleeding continues, and a decision is made to identify
and treat the bleeding point radiologically. For each
question posed below, choose the single most appropriate
option from the above list. Each option may be used once,
more than once or not at all.

Question 8: What vessel runs up the broad ligament? J


Question 9: Which other vessel arises from the uterine
artery? K
Question 10: Which other vessel anastomoses with the
uterine artery? G
Question 11: Which vessel does the uterine artery arise
from? E
Question 12: Which vessel runs anterior and superior to
the ureter? J
C. Options :

A- vitamin A
B- vitamin E
C- vitamin D
D- vitamin B12
E- vitamin B6
F- vitamin C
G- folic acid
H- riblflavine
I- vitamin K
J- thiamine

1- fat soluble vitamin sysnthesised in the intestinal wall


from the beta carotene . VIT A

2- fat soluble vitamin synthesised by large intestinal


bacteria . VIT K

3- deficiency of this vitamin in childhood causes rickets .


VIT D

4- fat soluble vitamin synthesis by the kidney is regulated


by parathyroid hormone . VIT D

5- deficiency of this vitamins typicaly occurs in women


with hyperemesis gravidarum . THIAMINE

6- water soluble vitamin with anti-oxidant effects . VIT C

7- absorption of this vitamin requires the presence of


intrinsic factor . VITB 12

8- deficiency of this vitamin causes megaloblastic


anaemia and neurological disorders . B 12

9- fat soluble vitamin produced in the skin by photo-


activiation of 7- dehydrocholesterol . VIT D

10- fat soluble vitamin with anti-oxdant effects . VIT E

11- fat soluble vitamin deficiency associated with raised


serum alkaline phosphatase . ???

12- deficiency of this vitamin associated with


osteomalasia VIT D.

13- maternal intake of this vitamin around the time of


coception has been shown to reduce incidance of neural
tube defects . FOLIC ACID

14- fat soluble vitamin , deficiency occur within few a


days of cessation of bile secretion . VIT K

15- this vitamin is a cofactor in synthesis of prothrombin


VIT K

D. A- Candida spp
B- Herpes simplex virus
C- Human immunodeficiency virus
D- Gardnerella vaginalis
E- Chlamydia trachomatis
F- Human papilloma virus
G- Treponema pallidum
H- Neisseria gonorrhoeae
I- Trichomonas vaginalis

Select the micro-organism described in the following cases:

1- An obligate intracellular gram negative bacterium ...T VAGINALIS

2- The cause of lymphogranuloma venereum ........C TRACHOMATIS.

3- Neutrophils containing gram negative diplococci NESISSRIA

- Double Y Syndrome
B- Down’s Syndrome
C- Edward’s Syndrome
D- Fragile X Syndrome
E- Klinefelter’s Syndrome
F- Patau’s Syndrome
G- Turner’s Syndrome

Please select the most appropriate syndrome from the options below. You
may use each option once, more than once, or not at all.

1- An infant girl is born with a webbed neck, hypertension and audible


ejection systolic murmur.
TURNERS
2- A neonate boy is born with cleft lip and palate, low set ears and
polydactyly. The child survives for only 3 weeks. EDWRDS

3- An infant boy is born with hypotonia, epicanthic folds and single


palmer creases DOWN
- A- Adrenal arteries
B- Celiac trunk
C- Common iliac arteries
D- Gonadal (ovarian or testicular) arteries
E- Inferior mesenteric artery
F- Lumbar arteries
G- Median sacral artery
H- Renal arteries
I- Superior mesenteric artery

Select the most appropriate option:

1- Gives rise to the left gastric, splenic and hepatic arteries. COELAIC

2- Gives rise to the intestinal, middle colic and right colic arteries. SUP
MESENTERIC

3- Gives rise to the left colic, sigmoid and superior rectal arteries. INF
MESESNTERIC

4- An unpaired branch that arises from the aortic bifurcation. MEDIAN


SACRAL

5- Arises from the aorta between the level of L1 and L2 vertebrae, and
forms five segmental arteries that do not anastomoses LUMBAR

For the pregnancies at risk of the following conditions (1-8) , select the
most appropriate prenatat diagnostic investigation from the alternatives
(a-e).

conditions:
----------------
1- spina bifida.
2- duchenne muscular dystrophy.
3- cystic fibrosis.
4- congenital heart disease.
5- sever osteogenesis imperfecta.
6- fetal rubella .
7- anonychia .
8- down syndrome

Alternatives:
--------------------
a- karyotyping of the fetal amniocytes .
b- fetoscopy .
c- DNA analysis of chorionvillus sample .
d- ultrasonography .
e- fetal blood sampling .

Which of the following viruses are associated with the following


conditions

1- Herpes simplex type 2 .


2- Human papillomavirus types 16 and 18 .
3- Hepatitis E .
4- Echoviruses .
5- Cytomegalovirus .

a- cervical intraepithelial neoplasis (CIN) .


b- severe and often fatal maternal infection in pregnancy in developing
counteries .
c- outbreaks of infection in nurseries for newborn infants .
d- neonatal infection acquired via fetal scalp monitoring .
e- persistent and generalised fetal infection .

For each complication (a-d) select the drug (i-v) most likely to be the
cause :

a- Cystitis .
b- Cardiac toxicity .
c- Pulmonary fibrosis .
d- renal toxicity .[/i]

[i]i- Vincristine .
ii- Bleomycin .
iii- cis platinum .
iv- Cyclophosphamide .
v- Doxorubicin .

For each drug (A-B) select the appropriate description(i-v):

A- 5-Fluorouracil (5FU) .
B- Cyclophosphamide .

i- Pyridine of purine analogue .


ii- Alkylating agent .
iii- Folic acid analogue .
iv- Plant alkaloid .
v- Antibiotic .

Match the following (A - E) to five of the statements below..

DURING THE MENSTRUAL CYCLE ;


A- follicle stimulating hormone .
B- oestradiol .
C- progesterone .
D- the first meitotic division .

1- is completed following the LH surge .


2- is produced by the adrenal gland .
3- is completed during the neonatal period .
4- is inhibited by oestradiol .
5- decereases mid - cycle .
6- is a precursor of oestradiol .
7- is inhibited by GnRH .
8- is produced throughout the cycle .
9- is produced in the secretory phase .
10- is secreted by the hypthalamus.

Match five of the following hormones / steriods to the statements below :

a- Dihydroepiandrostenedione .
b- FSH .
c- GnRH .
d- Insulin .
e- LH .
f- Oestradiol .
g- Progesterone .
h- Prolactin .
I- Testosterone .
j- Thyroid stimulating hormone (TSH) .

1- is produced in a pulsatile fashion from the hypothalamus .


2- is increased in the postpartum period .
3- is produced predominantly by the thecal cells in women .
4- is the pituitary hormone that is stimulated by low oestradiol
concentrations .
5- is produced predominantly by the adrenal gland in women

Match the following (A - E) to five of the statements below..


DURING THE MENSTRUAL CYCLE ;

A- follicle stimulating hormone .-------------------------------------------4


B- oestradiol .---------------------------------------------------------------8
C- progesterone .-----------------------------------------------------------9
D- the first meitotic division .----------------------------------------------1
E- testosterone .------------------------------------------------------------6
1- is completed following the LH surge .
2- is produced by the adrenal gland .
3- is completed during the neonatal period .
4- is inhibited by oestradiol .
5- decereases mid - cycle .
6- is a precursor of oestradiol .
7- is inhibited by GnRH .
8- is produced throughout the cycle .
9- is produced in the secretory phase .
10- is secreted by the hypthalamus.

Testosterone is produced by the thecal cells and converted to ostradiol by


aromatase. Oestradiol is secreted throughout the menstrual cycle initially
by granulosa cells in the developing follicle and then by the corpus
luteum ,Progesterone is produced by the corpus luteum , changing the
endometrium from proliferative to secretory . The LH surge triggers the
final stage of the first meiotic division whilst fertilisation causes the
second meiotic division with the extrusion of the polar body by uneven
division of the cytoplasm GnRH is secreted by the hypothalamus in a
pulsatile manner and stimulates the production and release of LH and
FSH in the anterior pitutary gland . Androgen precursors are secreted by
the adrenal but oestradiol is only produced by the overy ....

Sep. 2009
Options
1. oval inlet transversal diameter longer than anterposterior .
2. oval inlet anterposterior dia longer than trans.
3. heart shape inlet anterposterior dia. longer than trans.
4. heart shape inlet trans. diam. longer than anterposterior
5. rounded inlet .. T>A.
6. rounded inlet ..A > T
Qus.
a. Gynaecoid pelvis.
b. Android pelvis .
------------------------------------------------------------

a. Adenine monophosphate
Guanine monophosphate
Cytosine monophosphate duple helix
Thymine monophosphate

b. Adenine monophosphate
Guanine monophosphate
Cytosine monophosphate duple helix
Uracil monophosphate

c. Adenine monophosphate
Guanine monophosphate
Cytosine monophosphate single helix
Uracil monophosphate

d. Adenine monophosphate
Guanine monophosphate
Cytosine monophosphate single helix
Thymine monophosphate

e. Diadenine monophosphate
Diguanine monophosphate
Dicytosine monophosphate duple helix
Diuracil monophosphate

f. Diadenine monophosphate
Diguanine monophosphate
Dicytosine monophosphate duple helix
Dithymine monophosphate

g. DiAdenine monophosphate
DIGuanine monophosphate
DICytosine monophosphate single helix
DIUracil monophosphate
h. DIAdenine monophosphate
DIGuanine monophosphate
DICytosine monophosphate single helix
DIThymine monophosphate

1. DNA.
2. mRNA
3. Nucliac acid o HPV.


MRCOG part 1 RECALLS
SEPTEMBER 2008
Paper 1 & 2 EMQs
In a country wide survey, 10 general practices were picked at random and
5% of patients were selected at random from each practice:
Ta) the sample of patients is a true random sample
Tb) all practices had an equal chance of selection
Tc) all patients had an equal chance of selection
Fd) two siblings could not have been selected
Te) inferences about all patients in the country may be drawn from the
sample A lady is about to deliver and you are about to give her a
pudendal block..

a) ant division of ant. remi of S2-4


b) ant division of post.rami of S2-4
b)genitofemoral nerve
c) post division of ant. remi of S2-4
d) post division of post. remi of S2-4
e) sensory supply of perineal nerve
f) inferior rectal nerve
g) ischial tuberosity
h)ischial spine

Q1 -wht is the root value of pudendal nerve?


Q2- wht nerve supplies lower part of vagina (I think)?
Q3 –why do u give local skin infiltration before episiotomy?

If we take the day of fertilization as Day’0’ then…

Day2
Day4
Day 8
Day 10
Day12
Day 14
Day 18
Day20
Day 22
Day 24
Day 26
Day 42
Day 56
Day 70

Q4-Which represents 4 cell stage ?


Q5- conceptus implants completely?
Q6-feable hrt pulse seen on ultrasound?
Q7- vertebra form completely?

Table: on different values given for deaths due to maternal mortality with
direct and indirect causes as well as # of days from delivery to the cause
of death

Deaths due to
placenta previa and hypovolemia x @ e.g 20 days,10 days,8days etc
suicide(with previous hx of psychiatric illness)
crash victim
diabetic neuropathy
two more causes given with values

Different numbers were given in options

Q8-the number of late maternal deaths due to indirect causes?


Q9-number of maternal deaths due to direct causes?

Calculate Numbers needed to treat from a drug trial on osteoporosis: (Q


frm EMQ book)

Options
25
50
100
250
300
400
1000
others

Q10-N1=10,n2=5,n3=9990,n4=9995
Q11-N1=50,N2=25,n3=9950,N4=9975

FSH
Oxytocin
GH
Prolactin
Rennin
ACTH
etc

Q12-which is a protein with alpha and beta subunits ?


Q13 – which stimulates mineralocorticoid activity/relsease?
Q14- which hormone is produced by posterior pituatory?

Folic acid
Magnesium
Calcium
Vit b1-B12 given

Q15- which vitamin is required in 400mcgs at preconception?


Q16-which vitamins absorption is hampered by oxalic acid & phytates?
Insulin & Glucagon excretion

Decrease decrease
Increase increase
Increase decrease
Deacrease increase
Nochange no change
Nochange increase
Nochange decrease
Etc

Q-17-on taking a large protein meal?

Folowing are risk factors for cancer

Q-18- HCC – ans Hep B

Q19- Skin cancer-ans UV radiation

Q20- Cervical/uterine cancer-ans HPV

Q21- Breast cancer-ans BRCA1 mutation

(Repeats from march 2008)

Hormone receptors for following are located in

a) protein kinase nuclease receptor


b) nucleae transcription receptor
c) G-linked cell membrane receptor
d) G-linked golgi apparatus receptor
e) Protein complex intra cytoplasmic receptor
f) Multiligand receptor in cell membrane etc
g) Protein Tyrosine kinase activity receptor on cell membrane

Q22- Insulin
Q23-PG E2
Q24-Estrogen
Q25-Progesterone

Q26-A family with one kid presumed to have cystic fibrosis however cftr
gene mutation was not detected. Family members willing to give samples
if needed.mother wants definite diagnosis as she is 11wks and wants to
know diagnosis for her baby.parents are cousins with 1st degree relatives
having cystic fibrosis. Invasive diagnostics can be applied to reach final
results by?
Q27-you have Guthrie spot of their son how will u diagnose him for
cystic fibrosis?
Q28- families in uk with cystic fibrosis how will u find f508?
Options:

Pcr
Fish test
Chromosomal linkage analysis
Telomer analysis
Sum hereditary crosslinkage chromosomal option (dnt remember)
Screen for some known gene mutation
Sweat test
Saliva test

screening test for following

Q29-alpha thalessemia
Q30-folic acid deficiency
Q31-iron deficiency anemia
Q32-(one more i think!)

options

hbelectrophorisis
mcv
mchc
MCh
serum iron
paul-bennel test
etc

Q33-which is the commonest cause of non-dysjunction at meiosis?


trisomy 21
Q-34 which defect is associated with 60-70% cleft palate and cleft palate?
trisomy 13 or 18 or KF syndrome

all about cystic fibrosis

screening tests for a-thallesemia trait


iron deficiency amemia,folic acid anemia and....EMQ

RECEPTORS FOR
ESTROGEN,PROGESTERONE,PROSTAGLANDIN,INSULIN ....EMQ

NERVE SUPPLYING PERIANAL SKIN,PUDENDAL NERVE


ORIGIN,LA DURING EPISIOTOMY IS GIVEN FOR?????......EMQ

EMQ receptors for insulin


prostaglandins
estogen
progesteron

primary syphilis in early untreated period

options
syphilis treponemal antibody test
syphilis hemaglutination test
syphilis immobilization test

emq traetment of chlamedia


..................bacterial vaginosis

emq pre malignant conditions


emq statistics calculate n to be traetad
emq diagnosis of
iron defecincy anaemia
folic acid ..................
early untrated syphylis
emq congenital adrenal h icreased ? SUBSTANCE
emq embryology 4 cells ? day
vertebra formation

mcq about DIGITALIS, cyclosporine .mitochondrial dna ,cd8 cells , the


uterer(2 mcq) ,3 repeated statistics mcqs,
EMQ 36 WS PRIMI GRAVIDA had flu like sympt , the baby was born
jaundiced , hepatosplenomegaly ,skIn rash , brady cardia
,,,,choose the cuase (all viruses ,bacteria,listeria,plasmodium )??????

EMQ ...why we do local infiltration + pudendal block

EMQ ...cause of post operative infection ??pseudomonas

EMQ .....WHICH vit or mineral absorbtion is affected by phytate (no iron


in the choices

MCQ

citric acid cycle


fatty acid oxidation
???? genetic imprinting
?????stylomastoid foramen
spinal cord
in smooth muscle cells /regeneration possible /actin &myosin
present/lengh of fibers
repeated 2 mcqs u/s and radiation effect
apoptosis
2 mcqs about hyperplasia
pelvic splanchnic nerve

Paper 1 & 2 EMQs

A lady is about to deliver and you are about to give her a pudendal block..

a) ant division of ant. remi of S2-4


b) ant division of post.rami of S2-4
b)genitofemoral nerve
c) post division of ant. remi of S2-4
d) post division of post. remi of S2-4
e) sensory supply of perineal nerve
f) inferior rectal nerve
g) ischial tuberosity
h)ischial spine

Q1 -wht is the root value of pudendal nerve?


Q2- wht nerve supplies lower part of vagina (I think)?
Q3 –why do u give local skin infiltration before episiotomy?

If we take the day of fertilization as Day’0’ then…

Day2
Day4
Day 8
Day 10
Day12
Day 14
Day 18
Day20
Day 22
Day 24
Day 26
Day 42
Day 56
Day 70

Q4-Which represents 4 cell stage ?


Q5- conceptus implants completely?
Q6-feable hrt pulse seen on ultrasound?
Q7- vertebra form completely?

Table: on different values given for deaths due to maternal mortality with
direct and indirect causes as well as # of days from delivery to the cause
of death

Deaths due to
placenta previa and hypovolemia x @ e.g 20 days,10 days,8days etc
suicide(with previous hx of psychiatric illness)
crash victim
diabetic neuropathy
two more causes given with values

Different numbers were given in options

Q8-the number of late maternal deaths due to indirect causes?


Q9-number of maternal deaths due to direct causes?

Calculate Numbers needed to treat from a drug trial on osteoporosis: (Q


frm EMQ book)

Options

25
50
100
250
300
400
1000
others

Q10-N1=10,n2=5,n3=9990,n4=9995
Q11-N1=50,N2=25,n3=9950,N4=9975

FSH
Oxytocin
GH
Prolactin
Rennin
ACTH
etc

Q12-which is a protein with alpha and beta subunits ?


Q13 – which stimulates mineralocorticoid activity/relsease?
Q14- which hormone is produced by posterior pituatory?

Folic acid
Magnesium
Calcium
Vit b1-B12 given
Q15- which vitamin is required in 400mcgs at preconception?
Q16-which vitamins absorption is hampered by oxalic acid & phytates?
Insulin & Glucagon excretion

Decrease decrease
Increase increase
Increase decrease
Deacrease increase
Nochange no change
Nochange increase
Nochange decrease
Etc

Q-17-on taking a large protein meal?

Folowing are risk factors for cancer

Q-18- HCC – ans Hep B

Q19- Skin cancer-ans UV radiation

Q20- Cervical/uterine cancer-ans HPV

Q21- Breast cancer-ans BRCA1 mutation

(Repeats from march 2008)

Hormone receptors for following are located in

a) protein kinase nuclease receptor


b) nucleae transcription receptor
c) G-linked cell membrane receptor
d) G-linked golgi apparatus receptor
e) Protein complex intra cytoplasmic receptor
f) Multiligand receptor in cell membrane etc
g) Protein Tyrosine kinase activity receptor on cell membrane

Q22- Insulin
Q23-PG E2
Q24-Estrogen
Q25-Progesterone
Q26-A family with one kid presumed to have cystic fibrosis however cftr
gene mutation was not detected. Family members willing to give samples
if needed.mother wants definite diagnosis as she is 11wks and wants to
know diagnosis for her baby.parents are cousins with 1st degree relatives
having cystic fibrosis. Invasive diagnostics can be applied to reach final
results by?
Q27-you have Guthrie spot of their son how will u diagnose him for
cystic fibrosis?
Q28- families in uk with cystic fibrosis how will u find f508?
Options:

Pcr
Fish test
Chromosomal linkage analysis
Telomer analysis
Sum hereditary crosslinkage chromosomal option (dnt remember)
Screen for some known gene mutation
Sweat test
Saliva test

EMQ 1 ST PAPER
WHICH NERVE SUPPLIES PERIANAL SKIN

EMQ 2 CYSTIC FIBROSIS


a - invasive testing will be done ( she is preg 11 ws) which test

b- their child blood is ready for guathri test ,which test you will do

c- the relatives are willing to be tested , which test will reveal the 405
mutation for families in great braiten

any one have suggetions

more options were given for these qs


karyotyping
comparative genomic hybridization
flourecent ????

MCQ
propranolol
smooth endoplasmic reticulum
anal canal
smooth muscles (2 mcq ,paper1 ,2 )
primitive streak
fetal circulation
??microtubules
digitalis
mcq ; the following are bacteria cryptococcus ,candida,histoplasmosis,
mcq : the embryological origin of peritoneum,....
mcq mitosis miosis
mcq : the follwing are structral abnormalities of chromosomes
mcq : ?..................increase with preg
mcq : drug interaction methotrexate , sulphonamides

mcq :arterial and mean blood pressure


mcq : causes of alkalosis
mcq : the folloing are types of cells with their secretion
? endothelial cells heparin
?mast cells igE
? plasma celle
(i am not sure of the exact coupling)

mcq at 20 ws gestation rbcs are made in liver


lymph nodes
spleen
bone marrow
mcq about pudendal nerve
mcq ischiorectal fossa
mcq ureter in abdomen
mcq ureter in pelvis
mcq foramen of winslow (in detailes)
mcq kallman syndrome in men
mcq hcg
mcq trophoplasts
mcq ? all types of cells in islets of langerhans and their products
mcq the following are premalignant conditions: paget disease ...(repeat
mcq endotoxins or exotoxins ( i am not sure)
MCQ CORPUS LUTEUM
mianly endothelial clls
secerets inhibin
function decrease after 12 ws

MCQ INHIBIN

MARCH 2008
cystic fibrosis guthic spot test
receptors in apoptosis caspa
arias stella reaction
blood picture in PET and in preg is inflam like

1.EMQ paclitaxel carboplatin side effects


2.EMQ complex partial seiizure gum hypertrophy,acne ,facial
coarsening,vit K TO BE given..?drug used
3.EMQ oestrogen progesterone receptor site
4.EMQ on vitamins-xerophthalmia..morning sickness..macrocytic
anaemia.
5.MCQ rubella incubation period,specific immunity within 15 days..
6.EMQ HEPATOCELLULAR carcinoma,..??
7.EMQ lining epithelium of ureter,..??
8. EMQ montoux rection..
9.MCQ ovulatory DUB
10.MCQ concerning aneurysm- 10% due 2 inflam.,syphilitic
aneu,thoracic vessels,marfan recessive
if a mother has a child with cystic fibrosis ,she is preg and will do
amniocentesis
a. which test to be done (every possibility mentioned)
b. all relatives will volunteer to do gutic spot test ,whichtest you will
carry on the blood

-graph with hormones of the endometrial cycle


-
-citric acid cycle (graph)

microsatellites alleles
- cd4
-nk cells
-down syndrome
-pulmonary embolism
anal canal
-vagina
-ovary

phenylketonuria
inhibin
passive transport
erythropoietin
renin
hcg levels

syphylis
rubella
chlamydia
hormone dependent malignancies(testicular carc._sertoli leydig-clear
cellcarc.ofkidney-thyroid carc.)
pigd

missed abortion

MCQ congenital heart


emq crown \heel lenghth at birth
crown/rump length 6 cm =? weeks
about 10 questions statistics
plrese does anyone remember the q about ring Y chromosome
q about sattellite alleles ??

passive diffussion not depend on .

concenteration gradiant
molecular size

2following tissue are capable of regeneration

spinal cord
liver
epedermis
myocardium
bone marrow

3
double blind trial

delliosion of HCG in the urine pregnancy test

about the q APOPTOSIS and capsase ,q methods of disinfection and


BOWIE DICK test where is source to study these subjects?
q progesterone receptors

q about progesterone receptors

syphilis , toxoplasma , hpv , immune responses

EMQ type of hpv causing benign wart


type of hpv causing cancer cx

MCQ down syndrome


associated with duodenal atresia true
associated with ambig genetalia false
only maternal chromosome F
only paternal chromosome F

MCQ oogonia
miosis starts at puberty
miosis before mitosis
during S phase chromosomes are doubled

the most diagnostic test to be done on amniocentesis


gautic spot test what test you do with the blood sample

emq about vitamins (hyperemesis) i wrote vit B6


and (xerophthalmia)
q about endometrial hyperplasia
q about side effect of drug (carboplatin)-(taxal)
q about arias stella
if its specific in pregnancy
q about chlamydia
q about varicella
q about hpv ,wart
answers 16 or 18 or 6,11
q about hcg conc in pregnancy
citric acid cycle diagram
hormones diagram
fsh,lh,estrogen,testosteron
q about:by transvaginal u/s yolk sac apear in which week
many questions in genetic about pcr,fish test
q about laser
q in pathology
about totipotent cell,tumour like mass
q about voiding presure
q about ca requirement in pregnancy
q about test used to diagnose folic acid deficency
q about thalasthmia
q about cervical ectropion
cells i choose columner cell
- mri contraindicated in early pregnancy
q about stallete instability in dna
- transverse abdominus ms attached to lumbar transverse process
-cut s1 lead to autonomic bladder
q about cancer which are hormone dependent
-breast
-prostate
-thyroid
q about guthre test
q about cystic fibrosis and diagnosis
-swet test -karyotyping-fish test
q about disinfection and sterelization
q about complication of contraceptive pills

emqs
1lining of ureter
2.erythropoeitin and
renin q from john duthie
3complications of cisplatin
4 paclitaxel
5statistics -std error calculation
7 mean in a normal distribution
8occipito frontal diametr
9hcg titre at detection by tvs
10 at 3 days aftr fertilization
11crl in scan is 60mm what is gestational age
12crl at term ?
13 parietal sutur is between
14kuffer cells are -phagocytic

15 estradiol receptors
16 progestrone receptors -we know that thes are intra nuclear but there
were 2 confusin options with intranuclear plz refer this in depth , one
option had intranuclear kinase or somethin
16anticonvulsant with description of phenytoin
17vitamin deficiency causin hyperemesis
18 - macrocytic anemia
19 -xerophtalmia
20 cystic fibrosis 3 emqs
24 citric acid cyle diagrm , enzymes were options
complement causes
26 b thalasemia detection
27 folate def detection
29 basal cell ca - local malignant i think
30tumor like –hamaroma

March 2007
Copper IUD mode of action:
sperm motility
microthrombi

MRI:
ionising radiation?
no adverse effect on fetus?
detect fetal cardiac abn?

USG:
probe: array of magnets?
doppler to detect fetal heart movement?

facial hair increased by:


oestrogen?
testerone?
spirolactone?

characterics:
pass thro' origin?
slope = 0?
pass thro the mean?
unaffected by changing scale?
unaffected by changing dependent variable?

Copper containing IUDs:


F should be changed every year
F have a high incidence of actinomycoces colonization than plastic
devices
T Cause a relative increased in ectopic pregnancy
T have been implicated as a cause of fatal infection in pregnancies
F do not cause menorrhagia

Comments:
Inert devices can be left in place until the menopause, but copper devices
need renewal every 3-5 years, depending on the make, because of the
gradual absorption of copper. Copper IUCDs produce local
concentrations of copper salts which apparently give some protection
against bacterial contamination. Pelvic infection with actinomyces
organisms is most likely with a plastic device that has been in situ for
some years. While the rate of intrauterine pregnancy is reduced, that of
ectopics is not. Hence, there is a relative increase in ectopic pregnancy
after IUCD insertion. If an IUCD is left in place there is a slight risk of
intrauterine infection, preterm labour and antepartum haemorrhage, but
most pregnancies are uncomplicated and the device is delivered with the
placenta. Increased menstrual loss may be caused by increased
fibrinolytic activity which occurs round the IUCD. The progestogen
intrauterine system (IUS) reduces menstrual flow and often
dysmenorrhoea.

Which of the following is/are true concerning MRI?


A It involves ionizing radiation (False)
B Has no recognised side effects on the foetus (True)
C The pregnant mother should be turned to her left side during scanning
(True)
D Tissue with high hydrogen concentrations are difficult to distinguish
False)
E Blood vessels appear white on scanning (False)

Comments:
Water is a molecule composed of hydrogen and oxygen atoms. The nuclei
of the hydrogen atoms are able to act as microscopic compass needles.
When the body is exposed to a strong magnetic field, the nuclei of the
hydrogen atoms are directed into order - stand "to attention". When
submitted to pulses of radio waves, the energy content of the nuclei
changes. After the pulse, a resonance wave is emitted when the nuclei
return to their previous state. The small differences in the oscillations of
the nuclei are detected. By advanced computer processing, it is possible
to build up a three-dimensional image that reflects the chemical structure
of the tissue, including differences in the water content and in movements
of the water molecules. This results in a very detailed image of tissues
and organs in the investigated area of the body. In this manner,
pathological changes can be
documented.[/color:febed2ebf4][/b:febed2ebf4]-------------------------------
---------------------------------------

Many stats about sensitivity, positive predictive value, accuracy, even the
linear regression

Acute UTI in pregnancy


rarely symptomatic ?F

Copper IUD mode of action:


?T sperm motility
?T microthrombi
?T intervent ovary movement

MRI:
ionising radiation F
no adverse effect on fetus T
detect fetal cardiac abn ?F ( I think it will be very small)

USG:
probe: array of magnets? ?F
doppler to detect fetal heart movement T

facial hair increased by:


oestrogen?F
testerone?T
spirolactone F ( I think it is used for treating hirsutism)

Mycobacteria
?T strict ?? *(nor sure which word they use ) aerobic

-------------------------------------------------------------------
48---if a distribution of results is markedly skewed to the left(sep2000)
a-the mean is same as 50th centile— f
b-the same no of values lie on either side of the median- t
c-the mode is equal to the median- f
d-the students test should be used to compare this distribution with
another- f
e-logrithmic transformation of the results will produce a distribution close
to normal-t

-concerning the ablity of a test to predict disease


a-sensitivity is the ablity to predict those with disease correctly T
b--sensitivity is the same as positive predictive value F
c-the confidence interval must cross 1 to prove significance t (NOT A
MUST as CI at 1 is perfect )
d-an odds ratio of 1;3 implies the risk of 33% f
e--an odds ratio of 2 indicates a halving of risk F
------------------------------------------------------------------------------------

1-SUCCESSFUL LACTATION IS
a-maintained by estrogen --
b-maintained by progesterone --
c-initiated by prolactin surge--
d-maintained by human placental lactogen
e-inhibited by dopamine –

2-THE RELEASE OF CATECHOLAMINES FROM THE ADRENAL


MEDULLA INCREASES
a-during sleep—
b-when the nerves to adrenal glands are stimulated—
c-when the blood sugar rises—
d- immediately following a myocardial infarction—
e- in the presence of pheochromocytoma—T
F-DURING ACUTE HAEMORRHAGE—

3- In the pathogenesis of thrombosis


a-prostacyclin induces platelet aggregation—
b-plateletes synthesis thromboxane A2--
c-thromboxane A2 induces vasoconstriction--
d-contact with subendothelial collagen causes platelete aggregation--
e-thrombin inhibits platelete aggregation—

--4- CONVERSION OF GLUCOSE TO LACTIC ACID


a-occurs in single enzymatic reaction—
b-is the only pathway for synthesis of ATP in red blood cells—
C-is a reversible process in skeletal muscle
d- is inhibited by high cellular conc of ATP-
e-occurs in skeletal muscle when availability of oxygen is limited-

5-GLUCOCORTICOIDS
a- promotes hepatic gluconeogenesis-
b- suppress uptake of glucose by muscle-
c—promotes protein breakdown-
d—promotes fat breakdown-
e-increase glycolysis in adipose tissue-

6-Actinomycete israelii
1-Is a fungus—
2- forms yellow granules in pus –
3-is a commensal in mouth--
4- is a commensal in vagina--
5- is usually resistant to penicillin—

7-Following substances increase the serum uric acid concentration(


a-colchicine--
b-chlorothiazide--
c-allopurinol--
d-probenecid--
e-phenylbutazone—

7--THE FOLLOWING ARE INHERITED AS AUTOSOMAL


RECESSIVE CONDITIONS

A. tuberous sclerosis.
b-phenylketonuria--
C. ahondroplasia.--
D. sickle cell anaemia-
E. Von Giek l disease.—

8-. Genes on sex chromosomes are responsible for the inheritance of (


A. glucose-&pbosphate dehydrogenase defiency.
B. defective colour vision.—
C. hairy ear rims. –
D. homocystinuria.
E. Hurler syndrome.

9-in human a haploid no of chromosomes is found in)

a-red blood cells-


b-blastocysts-
c-primary oocytes-.
d-the first polar body-
e-spermatozoa-

10-osteoprosis is associated with


a-an increase in uncalcified bone matrix(osteoid tissue
b-prolonged oestrogen therapy—
c-a normal histological bone structure-
d-bone fracture—
e-irregularity of epiphyseal plates—

11-THE FOLLOWING HAVE an anti-emetic effect


a-hyoscine bromide—
b-morphine sulphate—
c-chlorpropamide--
d-promethazine hydrochloride—
e-perphenazine—

12CLOMIFENE CITRATE
a- is an antiandrogen—
b-does not stimulate ovulation directly—
c-can produce visual disturbances--
d-is generally prescribed throughout the proliferative phase of the
menstrual cycle—
e-in the treatment of anovulation increases the risk of multiple
pregnancy—

13--if a distribution of results is markedly skewed to the left


a-the mean is same as 50th centile
b-the same no of values lie on either siDe of the median
c-the mode is equal to the median-
d-the students test should be used to compare this distribution with
another-
e-logrithmic transformation of the results will produce a distribution close
to normal-

15. when a man has hemophilia


a. 50% of his daughters would not expected to be carriers –
b. 25% of his sons expected to be carriers
c. Good medical control of blood definition reduce the risk of this
condition in his children
d. His new born child is likely to require an urgent blood transfusion -
e. His sister has 50% probability of being a carrier

16- THE INTERSTITIAL CELL (LEYDIG) OF THE TESTIS


a--secrete seminal fluid—
b-are stimulated by LH –
c-secrete androgen binding protein—
d-secrete fructose –
e-produce testosterone T in response to LH --

17--CONCERNING VIRUSES
a-the core of every virus contain RNA—
b-they usually produce intracellelur toxin causing cell death

c-antibodies are directed against capsular protein—


d- they can be grown in intact cells--
e-interferone are synthetic antiviral substances—

18-THE FOLLOWING ARE CYTOTOXIC ALKYLATING AGENTS(


a-cyclophosphamide—
b-mercaptopurine--
c-chlorambucil--
d-fluorouracil--
e-methotrexate-

19-2,3 DIPHOSPHOGLYCERATE
a-is present in higher conc in maternal erythrocytes than fetal
erythrocytes--
b-binds more avidly to haemoglobin A than to haemoglobin E—

c-increases the affinity of haemoglobin for oxygen—


d-is a phospholipid
e- is synthesised by the pentose phosphate pathway

18-HYPERKALEMIA IS A CHARACTERSTIC FINDING IN


TINDALL
a-primary aldosteronism
b-treatment with Spironolactone-
c-hyperparathyroidism-
d-ACTH secreting tumours of bronchus-
e-renal failure-

19- CONCERNING INHERITABLE DISEASES


A-huntigdon ,s chorea is transmitted by a dominant gene—
b-phenylketonuria is transmitted by a recessive gene—
c-haemophillia ia an autosomal dominant condition--
d- Von Willebrand disease is a sex linked condition—
e-cystic fibrosis is transmitted by an x linked recessive gene--
20-Early blood borne dissemination is a characteristic feature of:
a) carcinoma of the endometrium –
b) osteosarcoma –
c) basal cell carcinoma –
d) carcinoma of the cervix-
e) choriocarcinoma-

21- in tissue pigmentation the following are associated


a-kernicterus and conjugated billirubin
b-addison disease and increased cutaneous melanin
c-melanosis coli and bile pigments-
d—wilson disease and copper deposition in the cornea-
e-corpus leuteum and carotenoids-

22-acquired diverticular disease of the colon


a-is present in at least 15% of Caucasian over the age of 50 years-
b-is due to congenital abnormality of the bowel wall
c-is associated with increased intraluminal pressure—
d-is associated with muscular thickening—
e-may result in intestinal obstruction—

23 stored blood which is to be used for transfusion—


a-kept at -4 degree
b-must be used in 1 week –
c-is tested for compliment content before transfusion --
d-may be used for platelet replacement –
e-contains an acid anticoagulant –

24- in uncomplicated homozygous beta thalasemia there is


a-hypochromasia--
b-a reduction in haemoglobin A—
c-an increase in haemoglobin F—
d- RED CELL SICKLING
e-presence of megaloblasts in bone marrow-

32-The following are capable of cellular regeneration


a)spinal cord
b)liver parenchyma –
c)gut epithelium –
d)kidney ???
e)bone marrow—

33-during the development of female reproductive system


a-primordial germ cells arise in yolk sac--
b-ovarian development is dependent upon oestrogen activity--
c-the paramesonephric duct give rise to the cervix--
d-the greater vestibular glands arise from the urogenital sinus--
e-differentiation of external genitalia is dependent upon ovarian activity--

36- IN CONGENITAL ADRENAL CORTICAL HYPERPLASIA


a-commonest deficiency is C21
b- plasma cortisol conc is raised –
c-there may be excessive secretion of 17 hydroxy progesterone--
d- sodium retension is characterstic –
e-blood catecholamine conc are increased –

37-- SCHISTOSOMA HAEMATOBIUM


a- is a snail –
b- is prevalent in china—
c- infestation may affect the uterine cervix
d- give rise to chronic granulomatous lesions--
e- infestation predisposes to carcinoma—

38- KETONE BODIES


.
a-can be utilised by theadult brain—.
B-Include acetone -
c-are water soluble --
d- are synthesised in skeletal muscles—
e-can be utilised during starvation—

39-IRON IONS
a-diffuse passively into erythropoitic cells-
b-bind to transferin-
c-are taken up by hepatocytes-
d- are necessary for cytochrome synthesis-
e-are absorbed predominantly by the ileum-

40-- --the conjugation of billirubin


a-takes place in hepatocytes-
b-is catalysed by UDP glucuronyl transferase-
c-is inhibited by phenobarbitone-
d-renders it water soluble-
e-is impaired in acute billiary obstruction-

41- in radiotherapy(

a-1Gray is eq to 1joule per kg


b- the skin usually recieve a greater dose of radiation than underlying
tissue-
c-the major effect of radiation energy is to damage the cytoplasm of the
cell-
d-cells in tissue which are hypoxic are more vulnerable to radiation-
e-radiation induced changes in tissues may take 6 weeks to develop-

-42--osteomalacia is characterised by
a-mineralization of the periostium –
b-deposition of uncalcifiedbone matrix --
c-normal osteobastic activity --
d-increase capillary fragility -
e-normal calcification of bone—

43-progesterone
a-is a C21 compound
b-synthesised by the ovary before ovulation-
c-increases ventilation-
d- raises BMR-
e-binds to corticotrophin –binding globulin-

44-EPIDERMAL GROWTH FACTOR


A- is mitogenic-.
b- synthesis is stimulated by oestradiol-
c-is a steroid molecule-F ...
d- is found in endometrium--
e- binds to receptor on the nuclear membrane-

45-the following are structural aberrations of chromosomes:


a. deletions --
b. inversions—
c. aneuploidy –
d. . polyploidy-
e translocations—

46-- Messenger RNA


a-Synthesis is dependent on RNA polymerase –
b-is an exact copy of sense DNA—
c-Contains exons –
d-Is measured by Western blotting –
e-translation occurs in the nucleus

47-- BETA SYMPATHOMIMETIC DRUGS MAY—


A- caue bronchospasm—
b-reduce frequency of uterine contraction—
c-cause heart block—
d-reduce diastolic blood pressure—

e- increase blood glucose conc

48- The following statistical statements are correct-


a-in the normal distribution,the value of mode is 1.73 times the median –
b-in a distribution skew to the right,the mean lies to the left of median-
c-in the series- 2;7;5;2;3;2;5;8, the mode is 2 -
d-student’s ‘t’ test is designed to correct for skew distribution –
e-the chi squared testmay be used when data are not normally distributed-
-

50-Contraindicated in breast feeding—

A-POPs
B-Bromocriptine-
C-chloroqunine-
d-??
e-warfarine—F

51-Contraindicated with kidney impairment


A-Dopamine -
B-Gentamycine –
C-Cephahexin
D-Cis platin

52- the following are characters of hypoxic cell death:


A-Apoptosis… --
B-Phagocytosis—…
C-Pyknosis… --
D-Poikilocytosis…
E-Release of phospholipids—

52- the following are characters of hypoxic cell death:

A-Apoptosis… --
B-endocytosis
C-Pyknosis… --
D-Poikilocytosis…
E-koliosytosis

prostagladins-
a-lipooxygenase pathway
b-thromboxane causes vasoconstriction
c-increased in the myometruim
d-?
e-?

linear regression:
a-starts from the origin
b-passes thru the mean-
c-values vary
d-ranking is done???

following test r used to compare 2 drugs a and b


a-student t test
b-meta analysis
c-wiloxone paired test
withney u paired test
e?

toxic shock syndrome


associated with the use of tampoons
b-due to toxigenic starinsof strep
c-infrequently reported outside n america
d-a consequence of previous antibioitic therapy
e-confined to seualy active women

the following can regenerate


a-kidneyepitheluim?
b-pheriheral axons-
c-liver parenchyme
d-bone marrow
e-bone

radiation sensitive
a-bone
b-bone marrow
c-epith of gut
d-skin
e-?

Mycobacteruim
a- are alchochol acid fast
b-does not form spores
c-d-a facultative anaerobes
d-responsible for leprosy
e-pathogenic inhumans all strains

the germination of tetanus spores in a wound is inhibited by

a-tissue trauma
b-oxygen
c-inject of toxoid
d-injection of antitoxin
e-removal of devitalised tissue

Aldosterone
a-reduces Na resorption in PCT
b-reduces Na absorption in descen loop of henle
c-Increase Na absortion in DCT
Increase K loss from the tubule
e increases Na absortion in collecting tubules

the following r conclusive evidence of pregnancy in uterine


cureetings???? can, t remmber
a-decidua compacta-
b-Arias -stella changes in endometrail glad-
c-spiral arterioles
d-plasma cell infiltration
e chorionic villi
White cell migration from the bld vessels in areas of inflammation
involves--- march 1997/2 21
a-call migration between the endothelial cells
b-a pssive loss of fluid bld elements
c-cell migration independeant of endothelial cell motion
d-initail emigration of polymorhs neutrophils
e-more polymorphs than monocytes after 2 day=

the vulva is supplied by


a-ant cutaneu erve of the thigh
b-femoral br/o genito femoral nerve
c-?
d?
e?

glucocorticoids
a-promote hepatic gluconeogenesis
b-suppress uptake of glucose by muscles
c-promote protein break down
d-promote fat reakdown
e-increase glycolysis

halothane(refer sept 1997)42


a-cardic arrythmias
b-explosive mixtures with air
c-liver damage

53- U/S:

A-Pulstile
B-Increase body temp 1 degree after 15 min scanning..
C-High frequency penetrates deeper TISSUE,-
D-Can distinguish between 2 points closer than 0.5 mm
E-Best echoes are produded by beam at right angle to the structures.

54-Disinfectant solutions may become contaminated with:


1-Enterobacter species –
2-Streptococcus species -
3-Escherichia coli -
4-Pseudomonas aeruginosa -
5-Staphyloccus pyogenes-
55-Nitric oxide
a) is synthesised in the endothelium –
b) has a short half life --
c) causes smooth muscle contraction
d) increases during pregnancy -
e) combines with oxygen to produce L-arginine—

but few question got modifications eg;


nitrous oxide synthesised by macrophages
naloxone antiemitic effect
drugs contraindicated in breast feeding
drugs causing renal impairment
ultasound
mri
statistics quit difficult all definitions
rna
pre implantation diagnosis

FIBRINOGEN (1
2) HEPATOGLOGIN
3) FERRITIN
4) ...

predominantly produced by UG system


- is a kind of lukotriene
- is phospholipid

which reactant protein will increase in injury

--------------------------------------------------------------------------------

1) FIBRINOGEN
2) HEPATOGLOGIN
3) FERRITIN
4) albumin
5) ?

Cell cycle
-prophase I and II
-mitosis I and II

Vulva nerve supply

Urinary bladder - lining/embrology

vagina - embrology

Fetal testis

IL I

MCH I

Function of Neutrophil polymorphs

Fetal HbF

PG

Umblical cord – embrology

prostaglandins
half lifa 30 min f

il1
produced by macrophages

whartons jelly originate from extra embryonic endodem

detrusor muscle is of mesodermal origin

acute phase proteins are transferrin


alfa feto protein

preimplantation diagnosis
possible 2 detect the sex of the embryo
uses pcr 4 chromosomal deletions
can be used 4 detection of single gene disoderes

facial hair causes


testosterone
mestranol
spironolactone

real time bias


length time bias
negative predictive value

fibrocystic disease of thr breast...in paper 2 , breast anatomy and breast


from endocrine function (in paper 1..)...ie 3 qs we about the breast
levator ani its attachnment and its coverings
endometrial carcinoma
adenomyosis
about measuring the arterial blood pressure and its methods
interleukin 2
prenatal implantation genetic diagnosis
menstural cycle in paper 1 and 2

MARCH 2006
The germination of tetanus spores in a wound is inhibited by
a)tissue trauma
b)oxygen
c)injection of anti-toxin
d)injection of toxoid
e)removal of devitalised tissue

The following values fall within the normal range for the adult female
bladder
a)residual volume of 100ml
b)voiding capacity of 250ml
c)bladder capacity of 900ml
d)intravesical pressure rise of less than 10cm H2O during early filling
e)maximum urine flow rate of 60ml per second
In the small intestine, the following substances are absorbed by active
processes
a)water
b)sodium
c)vitamin K
d)amino acids
e)chloride

White cell migration from blood vessels in areas of inflammation


involves
a)cell migration occurring between endothelial cells
b)a passive loss of fluid blood elements
c)cell migration independent of endothelial cell motion
d)initial emigration of polymorphonuclear neutrophils
e)more polymorphs that monocytes after 2 days

Antibodies (!!)
a)are soluble proteins
b)are formed in the fetus before 12 weeks of intrauterine life
c)have an average molecular weight of around 10000 daltons
d)of the rhesus type are genetically transmitted
e)are produced by the ribosomes of plasma cells

In DNA
a)a codon is a sequence of three bases
b)all codons have an identified function
c)there is a greater variety of amino acids than there are different codons
d)replication can be initiated at several different points along a
chromosome
e)complementary pairing precedes messenger mRNA synthesis

The inferior vena cava


a)is formed at the level of the fifth lumbar vertebra
b)commences posterior to the right external iliac artery
c)receives the left ovarian vein
d)receives the right ovarian vein
e)pierces the central tendon of the diaphragm

In the normal human pelvis


a)the promontory of the sacrum is in the upper anterior border of the first
sacral vertebra
b)the anterior surface of the sacrum has five paired foramina
c)the joint between the two pubic bones is a synovial joint
d)the acetabular fossa is wholly formed from parts of the pubic and
ischial bones
e)the transverse diameter of the brim is greater than the anteroposterior
diameter

Arginine vasopressin
a)reduces the glomerular filtration rate
b)controls water loss in the proximal renal tubule
c)is synthesised by the posterior pituitary gland
d)is released in response to a rise in plasma osmolality
e)is released in response to a fall in circulating plasma volume

Concerning ovarian function


a)progesterone is the major steroid of the developing follicle
b)granulosa cells secrete oestradiol
c)oestradiol is derived from androgen precursors
d)insulin-like growth factor (IGF-1) is not secreted by the ovary
e)circulating inhibin concentrations are a marker of granulosa cell
function

Actinomyces israelii
a)is a rickettsia
b)forms yellow granules in pus
c)is a commensal in the mouth
d)is a commensal in the vagina
e)is usually resistant to penicillin

Halothane produces
a)cardiac arrhythmias
b)explosive mixtures with air
c)liver damage if given repeatedly
d)myometrial relaxation
e)bronchial irritation

The therapeutic effect of the first drug is enhanced by the second drug
a)phenytoin: ethinyloestradiol
b)bromocryptine: metoclopramide
c)penicillin: probenicid
d)ritodrine: dexamethasone
e)warfarin: phenobarbitone

Potassium
a)is mainly intracellular
b)plasma levels vary in proportion to intracellular levels
c)plasma levels are decreased in Addison’s disease
d)plasma levels are increased in diabetic ketoacidosis
e)deficiency occurs with prolonged vomiting

Concerning carbohydrates
a)sucrose is a disaccharide of glucose and fructose
b)cereal grains contain less than 40% starch
c)cellulose is a fructose polysaccharide
d)a normal diet contains less than 60g of carbohydrate daily
e)dietary carbohydrate is oxidised in the body to carbon dioxide and
water

Steroid hormones
a)all contain 20 carbon atoms
b)can be produced by structures of urogenital ridge origin
c)are mostly activated in the liver
d)are predominantly excreted unchanged in the urine
e)mainly circulate unbound to carrier proteins

Ventilation is increased due to stimulation of central receptors by


a)nikethamide
b)hypoxia
c)doxapram
d)phenobarbitone
e)salbutamol

Tetrahydrofolic acid
a)is involved in purine synthesis
b)is a precursor of folic acid
c)is a coenzyme in amino acid synthesis
d)catalyses the conversion of glucose to glucose-6-phosphate
e)activity is inhibited by Methotrexate

Mitochondrial DNA
a)is located in the nucleus
b)inheritance is patrilineal
c)is present in two copies per cell
d)mutation causes cystic fibrosis
e)is involved in the control of oxidative phosphorylation

The following conditions may lead to hydronephrosis


a)mercury poisoning
b)cervical carcinoma
c)renal calculi
d)renal vein thrombosis
e)posterior urethral valves

In uncomplicated homozygous beta thalassaemia there is


a)hypochromasia
b)a reduction in haemoglobin A2
c)an increase in haemoglobin F
d)no depletion of iron stores
e)the presence of megaloblasts in bone marrow

The following statements relate to lung function in normal pregnancy


a)vital capacity is increased by about 50%
b)tidal volume is increased
c)the subcostal angle increases
d)the residual volume is reduced
e)the respiratory rate is increased

In normal pregnancy, uterine blood flow


a)is about 50ml/minute at term
b)is maintained throughout the cardiac cycle to the choriodecidual space
c)is reduced by prostacyclin
d)is increased during uterine contractions
e)represents about 10% of the cardiac output by the end of the first
trimester

During normal pregnancy


a)arterial pCO2 decreases
b)the blood hydrogen ion concentration decreases
c)plasma bicarbonate concentrations decrease
d)urine pH falls
e)lactic acid production is increased

The anal canal


a)has an upper part which is innervated by the inferior hypogastric plexus
b)has a lower part which is supplied by the superior rectal artery
c)drains lymph to the superficial inguinal nodes from its upper part
d)has its internal sphincter innervated by the inferior rectal nerve
e)has a superficial part of its external sphincter attached to the coccyx.

Human placental lactogen


a)is a single chain polypeptide
b)reaches the same concentration in fetal and maternal blood at term
c)may be secreted by the decidua
d)is detectable only after the 25th week of pregnancy
e)is an insulin antagonist

Human chorionic gonadotrophin !!


a)is a glycoprotein
b)is detectable 48 hours after fertilisation occurs
c)secretion peaks at 20 weeks of gestation
d)is synthesised by the corpus luteum of pregnancy
e)binds to luteinizing hormone receptors

Cholecalciferol (vitamin D)
a)promotes the absorption
b)is 25-hydroxylated in the liver
c)is synthesised in the SKIN
d)is 1-hydroxylated in the kidney
e)is most active in the 1,25-dihydroxyl form

Unconjugated bilirubin !!
a)is normally present in the plasma in lower concentration than
conjugated bilirubin
b)circulates in the plasma bound to albumin
c)is not excreted in the urine
d)does not cross the blood-brain barrier
e)crosses the placenta

The following are capable of cellular regeneration:


a)spinal cord
b)liver
c)epidermis
d)myocardium
e)bone marrow

The Leydig cells of the testis !!


a)secrete seminal fluid
b)are stimulated by luteinising hormone
c)are active in intrauterine life
d)secrete fructose
e)produce androstenedione

Angiotensin II !!
a)is the most potent vasoconstrictor
b)reduces aldosterone production
c)is mainly found in the lungs
d)is a decapeptide
e)is produced when the extracellular fluid volume is reduced

Cyproterone acetate
a)is an oestrogen
b)is used for the treatment of amenorrhoea
c)binds to androgen receptors
d)increases libido
e)inhibits spermatogenesis

Lignocaine used as a local anaesthetic !!


a)causes tachycardia if given as a systemic injection
b)has a longer lasting action than bupivicaine
c)is used in combination with adrenaline for ring block
d)causes vasoconstriction
e)is a weak base

Plasma osmolarity in the human


a)is normally about 290 milliosmoles per kg in the nonpregnant state
b)increases during the first trimester of pregnancy
c)is closely controlled by plasma protein concentration
d)is regulated by arginine vasopressin
e)regulates the sensation of thirst

In the statistical analysis of any group of numerical observations


a)the mean is always less than the mode
b)the median value always lies at the mid-point of the range
c)standard deviation is always greater than the standard error of the mean
d)the standard error of the mean is independent of the total number of
observations
e)there are the same number of observations greater than and less than the
median value

The pelvic splanchnic nerves


a)are derived from the posterior rami of the sacral spinal nerves
b)supply afferent fibres
c)mix with branches of the sympathetic pelvic plexus
d)supply the ascending colon with motor fibres
e)supply the uterus with parasympathetic fibres
Exotoxins
a)are derived from gram-negative bacteria
b)have specific action
c)are more toxic than endotoxins
d)are neutralised by their homologous antitoxin
e)can be converted to toxoid

Candida albicans
a)is gram positive
b)is an anaerobic organism
c)is associated with diabetes mellitus
d)is motile
e)is inhibited by oral tetracycline therapy

Transferrin
a)is one third saturated with iron
b)is increased in pregnancy
c)binds to 10mg of iron per gram
d)levels in the neonate are low
e)is actively transported

In Crohn’s disease there is


a) non casesous granuloma formation
b) formation of deep fissures
c) formation of crypt abscesses
d) the presence of mucosal polyps
e) a recognised association with occurence in the vulva

The rectum
a) usually commences at the level of S3
b) has a mesentery in its proximal third
c) drains lymph to the pre-aortic nodes
d) has a parasympathetic nerve supply derived entirely form the S3 spinal
segment
e) is about 25cm in length

The female urethra (!!)


a) traverses the perineal membrane
b) is lined throughout by urothelium
c) has a muscle layer continuous with that of the bladder
d) has an external sphincter supplied by the obturator nerve
e) corresponds developmentally to the membranous urethra in the male

During menstruation, endometrial haemostasis depends upon


a) a platelet and fibrin plug formation after the first 20 hours of bleeding
b) oestrogen mediated vasoconstriction
c) prostaglandin mediated spiral vessel constriction
d) production of plasminogen activating factor by the glandular epithelial
cells

In HIV, seroconversion illness after a needlestick injury


a)occurs within 1 to 4 weeks following exposure
b)infectivity decreases after seroconversion
c)the CD4 count is a useful adjunct in diagnosis
d)measurement of HIV RNA viral load is most useful in diagnosis

The following tumours arise in the ovary:


a) nephroblastoma
b) cystadenoma
c) granulosa cell tumour
d) neuroblastoma
e) teratoma

Early blood borne dissemination is a characteristic feature of:


a) carcinoma of the endometrium
b) osteosarcoma
c) basal cell carcinoma
d) carcinoma of the cervix
e) choriocarcinoma

Breast milk compared to cow's milk (!!)


a) has more protein
b) has more sodium
c) contains more carbohydrate
d) has more casein
e) contains more calories

The following are consequences of pulmonary embolism:


a) pulmonary infarction
b) fibrinous pleurisy
c) right ventricular hypertrophy
d) sudden death
e) haemoptysis
In a country wide survey, 10 general practices were picked at random and
5% of patients were selected at random from each practice:
a) the sample of patients is a true random sample
b) all practices had an equal chance of selection
c) all patients had an equal chance of selection
d) two siblings could not have been selected
e) inferences about all patients in the country may be drawn from the
sample

Nitric oxide
a) is synthesised in the endothelium
b) has a short half life
c) causes smooth muscle contraction
d) increases during pregnancy
e) combines with oxygen to produce L-arginine

Carbohydrates
a) yield 17kj/g of energy
b) are a major source of energy for the brain
c) are a a major component of the diet
d) give a higher energy yield compared to fats

Head mesenchyme gives rise to the following skull bones


a)Parietal
b)Mandible
c)Maxillary
d)Frontal
e)Ethmoid

Pre-implantation genetic diagnosis


a) can be used to identify single gene defects
b) is used in ICSI only
c) testing for trisomy 21 should be offered in a case of ovum donation
with a 45 year old recipient and 23 year old donor
d) is routinely used in IVF in selection of embryos
e) is governed by the regulations of the Human, Embryology and
Fertilisation Authority

The following are gram positive organisms


a)Brucella
b)Listeria
c)Staphylococcus
d)Vibrio cholera
Endometriotic deposits
a) only occur in the pelvis
b) consist of deposits of endometrial stromal tissue without glands
c) do not occur in postmenopausal women
d) are hormone sensitive

Amyloidosis
a)Can be inherited
b)Is associated with bronchiectasis
c)Is a cause of cardiomyopathy

Phenylketonuria
a)can only be diagnosed in the adult
b)is diagnosed by high levels of phenylpyruvate in the urine
c)treatment is with diet restriction

DUB (Dysfunctional uterine bleeding)


a) is common at the extremes of life
b) may be caused by coagulation disorders

Chylomicrons
a) are not normally present in the fasting state
b) after hydrolysis of lipoproteins, mainly consist of phospholipid

Other questions that I cannot remember in such detail were about:

Insulin (*half life of 30min)


Pituitary gland Anatomy (*contains pars tuberalis)
Many Pathology questions (equivalents ie *ovarian dysgerminoma =
seminoma?, *endodermal sinus tumour = yolk sac tumour?)
Embryology (notochord, primitive streak, mesonephros derivatives)
Ionising radiation (radiosensitivity ie *liver /intestinal epithelium - which
is more radiosensitive; *is radiation exposure of dental x-ray equivalent
to 1/5 that from a transatlantic flight)

difference b/w human milk and cows milk


casien
lactose
sodium
ascorbic acid
water
phenyl ketoneuria
inherited disorder
cause by defficiency of phenyl alanine
dietry restriction of tyrosine helpful
diagnosed by measuring phenyl ketone level in urine
essential amino acids
are not synthesized by body sufficiently
all ketogenic
can produce energy dont remember exact wording
radiation
mostly harmful in 8-12 wks gestation
xrays r ionizing radiation
contraindicated in pregnancy
transatlantic flight hv higher dose of radiation than xray
MRI
repeat q from sept paper
chylomicrins
responsible for turbulence of plsma?dont remem
metabolized in adipose tissue -true 80% 20% in liver
after hydrolysis by lipoprotien mainly phospholipids
r absorbed form intestinal luminal cells to ----?
ovarian tumors
dysgerminoma and endodermal sinus tumor hv same origin
choriocharcinoma can arise
cocp increase risk of ovarian cancers -this qs repeated again in other qs
with different wording
b cells
arise from plasma cells
in pregnancy humoral immunity is depressed early in pregnany
tumor necrotic factor alpha raised in parturition
pulmonary embolism
cause s
sudden death
fibrinous plurisy
rt ventricular hypertrohy
haemoptysis
repeated from sept 05
transferrin -
endo metrium
hiv seroconversion
carbohydrate
preimplantation diagnosis repeat from march 2005
progesteron
following rise in leuteal phase
basal body temp
progesterone
LH--PLZ SOME ONE ADD I DONT REMEMBER
NOTOCHORD
R SOLID CELLS
GIVE RISE TO brain --?
SKULL MESENCHYME GIVE RISE
TEMPORAL BONE
FRONTAL
PARIETAL

Q-Disinfectant solutions may become contaminated with:


1-Enterobacter species
2-Streptococcus species
3-Escherichia coli
4-Pseudomonas aeruginosa
5-Staphyloccus pyogenes

Q-Examples of active transport across a membrane include the passage


of:
1-Potassium ions into the neurons
2-Water into the proximal renal tubular cell
3-Glucose from the proximal renal tubular cell
4-Fatty acids into the mucosal cells of the intestine
5-Hydrogen ions from the gastric oxyntic cells.

ENDOMETRIOSIS
has endometrial tissue with no glands
not occurs after menopause

The following values fall within the normal range for the adult female
bladder
Fa)residual volume of 100ml
Tb)voiding capacity of 250ml
Fc)bladder capacity of 900ml
Td)intravesical pressure rise of less than 10cm H2O during early filling
Fe)maximum urine flow rate of 60ml per second
The germination of tetanus spores in a wound is inhibited by
Fa)tissue trauma
Tb)oxygen
Fc)injection of anti-toxin
Td)injection of toxoid
Te)removal of devitalised tissue

In DNA
Ta)a codon is a sequence of three bases
Tb)all codons have an identified function
Fc)there is a greater variety of amino acids than there are different codons
Td)replication can be initiated at several different points along a
chromosome
Fe)complementary pairing precedes messenger mRNA synthesis

Concerning ovarian function


Fa)progesterone is the major steroid of the developing follicle
Tb)granulosa cells secrete oestradiol
Tc)oestradiol is derived from androgen precursors
Td)insulin-like growth factor (IGF-1) is not secreted by the ovary
Fe)circulating inhibin concentrations are a marker of granulosa cell
function

Actinomyces israelii
Fa)is a rickettsia
Tb)forms yellow granules in pus
Tc)is a commensal in the mouth
Fd)is a commensal in the vagina
Fe)is usually resistant to penicillin

In the small intestine, the following substances are absorbed by active


processes
F a)water
T b)sodium
F c)vitamin K
T d)amino acids
T e)chloride

White cell migration from blood vessels in areas of inflammation


involves
T a)cell migration occurring between endothelial cells
F b)a passive loss of fluid blood elements
T c)cell migration independent of endothelial cell motion
T d)initial emigration of polymorphonuclear neutrophils
F e)more polymorphs that monocytes after 2 days

Which are formed from intramembranous ossification?


T Temporal , occipital, parietal
F. Sphenoid, Ethmoid

Antibodies (!!)
Ta)are soluble proteins -- Ab are glycoprotein. Are they soluble? not sure
[color=red:c2901df9c1][b:c2901df9c1]F[/b:c2901df9c1][/color:c2901df9
c1]b)are formed in the fetus before 12 weeks of intrauterine life
Fc)have an average molecular weight of around 10000 daltons
Fd)of the rhesus type are genetically transmitted
Te)are produced by the ribosomes of plasma cells

Correction
The germination of tetanus spores in a wound is inhibited by
Fa)tissue trauma
Tb)oxygen
[color=red:1e03073b8c][b:1e03073b8c]T[/b:1e03073b8c][/color:1e0307
3b8c]c)injection of anti-toxin
Td)injection of toxoid
Te)removal of devitalised tissue

Insulin (*half life of 30min) F


Pituitary gland Anatomy (*contains pars tuberalis)F

Q-Disinfectant solutions may become contaminated with:


T1-Enterobacter species
F2-Streptococcus species
F3-Escherichia coli
T4-Pseudomonas aeruginosa
F5-Staphyloccus pyogenes

The inferior vena cava


Ta)is formed at the level of the fifth lumbar vertebra
Fb)commences posterior to the right external iliac artery
Fc)receives the left ovarian vein
Td)receives the right ovarian vein
Te)pierces the central tendon of the diaphragm

In the normal human pelvis


Ta)the promontory of the sacrum is in the upper anterior border of the
first sacral vertebra
Fb)the anterior surface of the sacrum has five paired foramina
Fc)the joint between the two pubic bones is a synovial joint
[b:715c81bcd5]d)the acetabular fossa is wholly formed from parts of the
pubic and ischial bones [/b:715c81bcd5]
Te)the transverse diameter of the brim is greater than the anteroposterior
diameter

Arginine vasopressin
Fa)reduces the glomerular filtration rate
Fb)controls water loss in the proximal renal tubule
Fc)is synthesised by the posterior pituitary gland
Td)is released in response to a rise in plasma osmolality
Te)is released in response to a fall in circulating plasma volume

Concerning ovarian function


Fa)progesterone is the major steroid of the developing follicle
Tb)granulosa cells secrete oestradiol
Tc)oestradiol is derived from androgen precursors
Td)insulin-like growth factor (IGF-1) is not secreted by the ovary
Fe)circulating inhibin concentrations are a marker of granulosa cell
function

Actinomyces israelii
Fa)is a rickettsia
Tb)forms yellow granules in pus
Tc)is a commensal in the mouth
Fd)is a commensal in the vagina
Fe)is usually resistant to penicillin

Halothane produces
Ta)cardiac arrhythmias
Fb)explosive mixtures with air
Tc)liver damage if given repeatedly
Td)myometrial relaxation
Fe)bronchial irritation

The therapeutic effect of the first drug is enhanced by the second drug
Fa)phenytoin: ethinyloestradiol
Fb)bromocryptine: metoclopramide
Tc)penicillin: probenicid
Td)ritodrine: dexamethasone
Fe)warfarin: phenobarbitone

Potassium
Ta)is mainly intracellular
Fb)plasma levels vary in proportion to intracellular levels
Fc)plasma levels are decreased in Addison’s disease
Td)plasma levels are increased in diabetic ketoacidosis
Te)deficiency occurs with prolonged vomiting

Concerning carbohydrates
Ta)sucrose is a disaccharide of glucose and fructose
[color=red:715c81bcd5][b:715c81bcd5]F[/b:715c81bcd5][/color:715c81b
cd5]b)cereal grains contain less than 40% starch
[color=red:715c81bcd5][b:715c81bcd5]F[/b:715c81bcd5][/color:715c81b
cd5]c)cellulose is a fructose polysaccharide
Fd)a normal diet contains less than 60g of carbohydrate daily
Te)dietary carbohydrate is oxidised in the body to carbon dioxide and
water

Steroid hormones
Fa)all contain 20 carbon atoms
Tb)can be produced by structures of urogenital ridge origin
Fc)are mostly activated in the liver
Fd)are predominantly excreted unchanged in the urine
Fe)mainly circulate unbound to carrier proteins

Ventilation is increased due to stimulation of central receptors by


Ta)nikethamide
Tb)hypoxia
Tc)doxapram
Fd)phenobarbitone
Fe)salbutamol

Tetrahydrofolic acid
Ta)is involved in purine synthesis
Fb)is a precursor of folic acid
Fc)is a coenzyme in amino acid synthesis
Fd)catalyses the conversion of glucose to glucose-6-phosphate
Te)activity is inhibited by Methotrexate

Mitochondrial DNA
Ta)is located in the nucleus
Fb)inheritance is patrilineal
Fc)is present in two copies per cell
Fd)mutation causes cystic fibrosis
Te)is involved in the control of oxidative phosphorylation

The following conditions may lead to hydronephrosis


?Fa)mercury poisoning
Tb)cervical carcinoma
Tc)renal calculi
?Fd)renal vein thrombosis
Te)posterior urethral valves

In uncomplicated homozygous beta thalassaemia there is


Ta)hypochromasia
Fb)a reduction in haemoglobin A2
Tc)an increase in haemoglobin F
Td)no depletion of iron stores
Fe)the presence of megaloblasts in bone marrow

The following statements relate to lung function in normal pregnancy


Fa)vital capacity is increased by about 50% (ans from busyspr)
Tb)tidal volume is increased
Tc)the subcostal angle increases
Td)the residual volume is reduced
Fe)the respiratory rate is increased

[color=darkblue:715c81bcd5]In normal pregnancy, uterine blood flow


Fa)is about 50ml/minute at term
Tb)is maintained throughout the cardiac cycle to the choriodecidual space

Fc)is reduced by prostacyclin


Fd)is increased during uterine contractions
Te)represents about 10% of the cardiac output by the end of the first
trimester [/color:715c81bcd5]

P.200 sep 2001 paper 2 Q 46


During normal pregnancy
Ta)arterial pCO2 decreases
Fb)the blood hydrogen ion concentration decreases
Tc)plasma bicarbonate concentrations decrease
Fd)urine pH falls
?Te)lactic acid production is increased

The anal canal


Ta)has an upper part which is innervated by the inferior hypogastric
plexus
Fb)has a lower part which is supplied by the superior rectal artery
Fc)drains lymph to the superficial inguinal nodes from its upper part
Fd)has its internal sphincter innervated by the inferior rectal nerve
Te)has a superficial part of its external sphincter attached to the coccyx.

Human placental lactogen


?T[b:715c81bcd5]a)is a single chain polypeptide [/b:715c81bcd5]
Fb)reaches the same concentration in fetal and maternal blood at term
Tc)may be secreted by the decidua
?F[b:715c81bcd5]d)is detectable only after the 25th week of
pregnancy[/b:715c81bcd5]
?T[b:715c81bcd5]Te)is an insulin antagonist [/b:715c81bcd5]
Human chorionic gonadotrophin !!
Ta)is a glycoprotein
?T[b:715c81bcd5]b)is detectable 48 hours after fertilisation occurs
[/b:715c81bcd5]
Fc)secretion peaks at 20 weeks of gestation
Fd)is synthesised by the corpus luteum of pregnancy
Te)binds to luteinizing hormone receptors

Cholecalciferol (vitamin D)
Ta)promotes the absorption
Tb)is 25-hydroxylated in the liver
Tc)is synthesised in the SKIN
Td)is 1-hydroxylated in the kidney
Te)is most active in the 1,25-dihydroxyl form

Unconjugated bilirubin !!
?Fa)is normally present in the plasma in lower concentration than
conjugated bilirubin
?Tb)circulates in the plasma bound to albumin
Fc)is not excreted in the urine
Fd)does not cross the blood-brain barrier
?Te)crosses the placenta

The following are capable of cellular regeneration:


Fa)spinal cord
Tb)liver
Tc)epidermis
Fd)myocardium
Te)bone marrow

The Leydig cells of the testis !!


Fa)secrete seminal fluid
Tb)are stimulated by luteinising hormone
Tc)are active in intrauterine life
Fd)secrete fructose
?Fe)produce androstenedione
Angiotensin II !!
Ta)is the most potent vasoconstrictor
Fb)reduces aldosterone production
Tc)is mainly found in the lungs
Fd)is a decapeptide -- angiotensin I is a decapeptide
Te)is produced when the extracellular fluid volume is reduced

Cyproterone acetate
Fa)is an oestrogen [color=blue:715c81bcd5]--is an antiandrogenic
progesterone[/color:715c81bcd5]
Fb)is used for the treatment of amenorrhoea [color=blue:715c81bcd5]--
treat hirsutism [/color:715c81bcd5]
Tc)binds to androgen receptors
Fd)increases libido
Te)inhibits spermatogenesis

Lignocaine used as a local anaesthetic


Ta)causes tachycardia if given as a systemic injection
Fb)has a longer lasting action than bupivicaine
Fc)is used in combination with adrenaline for ring block
Fd)causes vasoconstriction
Te)is a weak base

Plasma osmolarity in the human


Ta)is normally about 290 milliosmoles per kg in the nonpregnant state
Fb)increases during the first trimester of pregnancy
?Fc)is closely controlled by plasma protein concentration
Td)is regulated by arginine vasopressin
Te)regulates the sensation of thirst

In the statistical analysis of any group of numerical observations


Fa)the mean is always less than the mode
Tb)the median value always lies at the mid-point of the range
Tc)standard deviation is always greater than the standard error of the
mean
Fd)the standard error of the mean is independent of the total number of
observations
Te)there are the same number of observations greater than and less than
the median value

The pelvic splanchnic nerves


Fa)are derived from the posterior rami of the sacral spinal nerves
Fb)supply afferent fibres
Tc)mix with branches of the sympathetic pelvic plexus
Fd)supply the ascending colon with motor fibres
Te)supply the uterus with parasympathetic fibres

Exotoxins
Ta)are derived from gram-negative bacteria
[color=red:715c81bcd5][b:715c81bcd5]T[/b:715c81bcd5][/color:715c81
bcd5]b)have specific action
Tc)are more toxic than endotoxins
Td)are neutralised by their homologous antitoxin
Te)can be converted to toxoid

Candida albicans
Ta)is gram positive
Fb)is an anaerobic organism
Tc)is associated with diabetes mellitus
Fd)is motile
Fe)is inhibited by oral tetracycline therapy
T treat with miconazole
T commonsal in intestine

Transferrin
Ta)is one third saturated with iron
c)binds to 10mg of iron per gram
d)levels in the neonate are low
e)is actively transported
Increased in pregnancy t
In Crohn’s disease there is
?Ta) non casesous granuloma formation
Tb) formation of deep fissures
Tc) formation of crypt abscesses
Td) the presence of mucosal polyps
e) a recognised association with occurence in the vulva

The rectum
Ta) usually commences at the level of S3
Fb) has a mesentery in its proximal third
[b:715c81bcd5][color=red:715c81bcd5]T[/color:715c81bcd5][/b:715c81
bcd5]c) drains lymph to the pre-aortic nodes
[color=red:715c81bcd5][b:715c81bcd5]T[/b:715c81bcd5][/color:715c81
bcd5]d) has a parasympathetic nerve supply derived entirely form the S3
spinal segment
[color=red:715c81bcd5][b:715c81bcd5]F[/b:715c81bcd5][/color:715c81b
cd5]e) is about 25cm in length

[b:715c81bcd5]The female urethra (!!)


a) traverses the perineal membrane
b) is lined throughout by urothelium
c) has a muscle layer continuous with that of the bladder
d) has an external sphincter supplied by the obturator nerve
e) corresponds developmentally to the membranous urethra in the male
[/b:715c81bcd5]

[b:715c81bcd5]During menstruation, endometrial haemostasis depends


upon
a) a platelet and fibrin plug formation after the first 20 hours of bleeding
b) oestrogen mediated vasoconstriction
Tc) prostaglandin mediated spiral vessel constriction
d) production of plasminogen activating factor by the glandular epithelial
cells [/b:715c81bcd5]
During menstruation endometrial hemostasis depends upon

Platelet and fibrin plug formation after first 20 hrs of bleeding


Production of PAF by glandular epithelial cells
Estrogen mediated hemostasis
PG mediated hemostasis

In HIV, seroconversion illness after a needlestick injury


Ta)occurs within 1 to 4 weeks following exposure
[color=darkblue:715c81bcd5][b:715c81bcd5]?T[/b:715c81bcd5][/color:7
15c81bcd5]b)infectivity decreases after seroconversion
Tc)the CD4 count is a useful adjunct in diagnosis
Td)measurement of HIV RNA viral load is most useful in diagnosis
HIV seroconversion
Illness occurs witin 1-4 wks......T...........SEROCONVERSION OCCURS
IN FIRST FEW WEEKS......FOLLOWED BY SIGNS OF ILLNESS
After illness infectivity decreases
.........[b:715c81bcd5][color=darkblue:715c81bcd5].F.........[/color:715c81
bcd5][/b:715c81bcd5]I THINK SO BECAUSE THOUGH ITS A
LATENT PHASE AND NO OVERT SYMPTOMS OCCUR WHILE
THE VIRUS MULTIPLIES, I THINK INFECTIVITY REMAINS JUST
THE SAME.........
The best available diagnostic test is HIV viral load
assay........[color=red:715c81bcd5][b:715c81bcd5]T[/b:715c81bcd5][/col
or:715c81bcd5].........FROM AN OBSTETRIC POINT OF
VIEW.....VIRAL LOAD IS THE ONLY USEFUL WAY OF
PREDICTING DISEASE......BECAUSE SEROLOGY IS UNRELIABLE
IN ITS RESULTS DUE TO MATERNAL ANTIBODIES......WHICH
MIGHT TAKE UPTO 18MONTHS TO DIE DOWN...
Without any intervention during pregnancy the mother to child
transmission rate is 80%
.........[color=red:715c81bcd5][b:715c81bcd5]F[/b:715c81bcd5][/color:71
5c81bcd5].....ITS 15-45%......LATTER IN UNTREATED CASES......
Mode of delivery by LSCS decreases the transmission to
baby......[color=red:715c81bcd5][b:715c81bcd5]T[/b:715c81bcd5][/color
:715c81bcd5].....CAN BRING THE TRANSMISS DOWN TO 2%
Short course of anti viral Rx during labor decreases transmission to
baby.........[b:715c81bcd5][color=red:715c81bcd5]T[/color:715c81bcd5][/
b:715c81bcd5]
BF is safe
........[b:715c81bcd5][color=red:715c81bcd5]F[/color:715c81bcd5][/b:71
5c81bcd5].......MOST IMP METHOD OF TRASMISS IF OTHER
PREVENTIVE MEASURES HAVE BEEN UNDERTAKEN....

The following tumours arise in the ovary:


Fa) nephroblastoma
Tb) cystadenoma
Tc) granulosa cell tumour
Fd) neuroblastoma
Te) teratoma

Early blood borne dissemination is a characteristic feature of:


Fa) carcinoma of the endometrium
[color=red:715c81bcd5]T[/color:715c81bcd5]b) osteosarcoma
Fc) basal cell carcinoma
Fd) carcinoma of the cervix
Te) choriocarcinoma

Breast milk compared to cow's milk (!!)


a) has more protein
Tb) has more sodium
c) contains more carbohydrate
d) has more casein
e) contains more calories
difference b/w human milk and cows milk
Tlactose
Tsodium
ascorbic acid
Twater

PS:human milk contains more water

The following are consequences of pulmonary embolism:


Ta) pulmonary infarction
Tb) fibrinous pleurisy
?Fc) right ventricular hypertrophy
Td) sudden death
Te) haemoptysis

[color=blue:715c81bcd5]In a country wide survey, 10 general practices


were picked at random and 5% of patients were selected at random from
each practice:
Ta) the sample of patients is a true random sample
Tb) all practices had an equal chance of selection
Tc) all patients had an equal chance of selection
Fd) two siblings could not have been selected
Te) inferences about all patients in the country may be drawn from the
sample [/color:715c81bcd5] ans from onexamination

Nitric oxide
Ta) is synthesised in the endothelium
Tb) has a short half life
Fc) causes smooth muscle contraction
Td)production is increases during pregnancy
[b:715c81bcd5]e) combines with oxygen to produce L-arginine
[/b:715c81bcd5]
T generated by arginine (from onexamination)

Carbohydrates
a) yield 17kj/g of energy
b) are a major source of energy for the brain
?Tc) are a a major component of the diet
Fd) give a higher energy yield compared to fats

Head mesenchyme gives rise to the following skull bones


a)Parietal
b)Mandible
c)Maxillary
d)Frontal
e)Ethmoid

Pre-implantation genetic diagnosis


Ta) can be used to identify single gene defects
Fb) is used in ICSI only
Fc) testing for trisomy 21 should be offered in a case of ovum donation
with a 45 year old recipient and 23 year old donor
Fd) is routinely used in IVF in selection of embryos
?T[b:715c81bcd5]e) is governed by the regulations of the Human,
Embryology and Fertilisation Authority[/b:715c81bcd5]

The following are gram positive organisms


?Fa)Brucella
Tb)Listeria
Tc)Staphylococcus
?Fd)Vibrio cholera

Endometriotic deposits
Fa) only occur in the pelvis
?Tb) consist of deposits of endometrial stromal tissue without glands
?Fc) do not occur in postmenopausal women
?Td) are hormone sensitive

Amyloidosis
Ta)Can be inherited
Tb)Is associated with bronchiectasis
Tc)Is a cause of cardiomyopathy
(ref: Familial Amyloidosis

Familial amyloidosis, or ATTR, is a rare form of inherited amyloidosis.


The amyloid deposits in familial amyloidosis are composed of the protein
transthyretin, or TTR, which is made in the liver. Familial amyloidosis is
inherited an [color=darkblue:715c81bcd5]autosomal dominant
[/color:715c81bcd5]in genetics terminology)
http://www.ncbi.nlm.nih.gov/entrez/query.fcgi?cmd=Retrieve&db=PubM
ed&list_uids=12472203&dopt=Abstract
http://www.emedicine.com/med/topic3365.htm
When the heart is involved heavily but the nerves are not, the disease is
called familial amyloid cardiomyopathy (FAC).
Phenylketonuria
Fa)can only be diagnosed in the adult
Tb)is diagnosed by high levels of phenylpyruvate in the urine
?Tc)treatment is with diet restriction
?Tinherited disorder
Fcause by defficiency of phenyl alanine ( defficiency of phenylalanine
hydroxylase)
Fdietry restriction of tyrosine helpful (restriction of phenylalanine)
?Fdiagnosed by measuring phenyl ketone level in urine

DUB (Dysfunctional uterine bleeding)


Ta) is common at the extremes of life
Tb) may be caused by coagulation disorders

Insulin (*half life of 30min) T


Pituitary gland Anatomy (*contains pars tuberalis) F

Many Pathology questions


(equivalents ie *ovarian dysgerminoma = seminoma?,
*endodermal sinus tumour = yolk sac tumour?)

Embryology (notochord, primitive streak, mesonephros derivatives)

NOTOCHORD
R SOLID structure as it forms --T
GIVE RISE TO brain -F
F is formed from primitive streak (ans from mrcogexam)
[color=blue:715c81bcd5][b:715c81bcd5]F fuses temporally with the
endoblast of yolk sac[/b:715c81bcd5][/color:715c81bcd5]

Ionising radiation
(?T radiosensitivity ie *liver /intestinal epithelium - which is more
radiosensitive;
*is radiation exposure of dental x-ray equivalent to 1/5 that from a
transatlantic flight)
essential amino acids
Fare not synthesized by body sufficiently
?Tall ketogenic
?Tcan produce energy dont remember exact wording
[b:715c81bcd5]Are ONLY used for the synthesis or important body
proteins.
Canot be catabolised for energy.[/b:715c81bcd5]

radiation
?Tmostly harmful in 8-12 wks gestation
Txrays r ionizing radiation
Tcontraindicated in pregnancy
[b:715c81bcd5]transatlantic flight hv higher dose of radiation than xray
[/b:715c81bcd5]

Biophysics:
Radiation effect more in hypoxic tissue..false
Bone, peripheral nerve and muscle are senstive to radiation..false
Intestine and bone marrow are sestive to radiation..true

chylomicrins
Ta) are not normally present in the fasting state
?T[b:715c81bcd5]responsible for turbulence of
plsma?[/b:715c81bcd5]dont remem
[color=darkblue:715c81bcd5]metabolized in adipose tissue -
[b:715c81bcd5]true [/b:715c81bcd5]80% 20% in liver
[/color:715c81bcd5]
Fafter hydrolysis by lipoprotien mainly phospholipids
?F r absorbed form intestinal luminal cells to ----?
(ref: Monoglycerides and free fatty acids associate with bile salts and
lecithin to form micelles *
The core of the micelle is composed of cholesterol and fat soluble
vitamins *
Monoglycerides, cholesterol and free fatty acids are absorbed by passive
diffusion across the duodenal and jejunal mucosa while the bile salts
remain in the lumen and are absorbed in the terminal ileum
Chylomicrons have a core of triglycerides and cholesterol and a coat of
protein and phospholipids

ovarian tumors
Tchoriocharcinoma can arise

cocp
F increase risk of ovarian cancers

Pill failure:
tetracycline…T
Rifmpicin…true
digoxin....F
phenobarbitone....T

Contraindicated in breast feeding:


cascara.....T
phenophthalien...F
norethisterone.....F
chloramphenicol...T
sulphonamides......T

b cells
Farise from plasma cells
in pregnancy humoral immunity is depressed early in pregnany
Ftumor necrotic factor

alpha raised in parturition

pulmonary embolism cause s :


Tsudden death
Tfibrinous plurisy
Frt ventricular hypertrohy
Thaemoptysis

following rise in leuteal phase


Tbasal body temp
Tprogesterone

SKULL MESENCHYME GIVE RISE


TEMPORAL BONE
FRONTAL
PARIETAL

Q-Examples of active transport across a membrane include the passage


of:
?T1-Potassium ions into the neurons
F2-Water into the proximal renal tubular cell --F
?T3-Glucose from the proximal renal tubular cell
[color=red:715c81bcd5][b:715c81bcd5]F[/b:715c81bcd5][/color:715c81b
cd5]4-Fatty acids into the mucosal cells of the intestine
5-Hydrogen ions from the gastric oxyntic cells.
(ref: glucose absorption in small intestine n renal tubules is active,all
other places is facilitated using GLUT.
also Na/K pump is active.
Na absorption is active in renal tubules.
Monoglycerides, cholesterol and free fatty acids are absorbed by passive
diffusion across the duodenal and jejunal mucosa while the bile salts
remain in the lumen and are absorbed in the terminal ileum )

ENDOMETRIOSIS
has endometrial tissue with no glands
?Tnot occurs after menopause[b:715c81bcd5][/b:715c81bcd5]

Regarding data
a The coefficient of variation expresses the Standard deviation as a
proportion of the mean

b Standard error of mean indicates how close a sample is to the


population mean

c A probability of one means that an event has one in ten chance of


happening

d Logarithmictransfoormation is a method of normalizing data that are


not normally distributed

e ANOVA is used for normally distributed data

Indomethacin can block ovulaion

Pulsatile release of LH occurs in first weel of life

RBCs possess the antigens of ABO system in fetus

MRI involves the administration of radio labelled compound

Doppler effect is used to detect movement

Proton is injected to improve contrast during MRI

ventilation and stimulation of central chemoreceptors...someone said true


for hypoxia. From my knowledge, the central chemoreceptors respond to
pH and CO2, not O2, rather the PERIPHERAL aortic and carotid bodies
respond to O2 and thus hypoxia.

sept.2005
Sigmoid colon
Attached to pelvic wall
Lies lateral to psoas major
Ends at S3

Rectum
Starts at S3
No appendices epiploae
Middle rectal artery is the major artery suuplying it
12 cm long

T pallidum
Spiral shape
Causes yaws in children
Stained with silver impregnation

HIV seroconversion
Illness occurs witin 1-4 wks
After illness infectivity decreases
The best available diagnostic test is HIV viral load assay
Without any intervention during pregnancy the mother to child
transmission rate is 80%
Mode of delivery by LSCS decreases the transmission to baby
Short course of anti viral Rx during labor decreases transmission to baby
BF is safe

Percutaneous mode of infection


H simplex
Rabies
Hepatitis B
Epstein barr virus

Hep A mode of infection is through


Blood transfusion
Shared needles
Faeco oral route
Insect bites
Air borne
Match the disease with their respective causative org
Bone marrow suppression – myco bacterium avium
Red cell aplasia – parvovirus

The foll are pre malignant


CIN 3
Vulval lichen sclerosis
Endometrioses
Fibro epithelial polyp
Atypical endometrial hyperplasia

Bacterial vaginosis
Microscopic finding “clue cells”
pH less than 4.5
best Rx with ampicillin
sensitive to clindamycin
may be due to gardnerella vaginalis

vaginal infecton
incidence of n gonorrhoea resistance to penicillin is 1%
incidence of n gonorrhoea resistance to 3 rd gen ceph 5%
azithromycin DOC for trichomoniasis
crytococcus neoformans causes meningitis

C trachomatis
Strains L123 cause LGV
Reticulate body is the infectious form

Transferrin
Increased in pregnancy
One third saturated with Fe
Binds to 10 mg of iron per gm
Levels in neonate are low

Antibodies
Are soluble proteins
Formed before 12 wks in fetus
ABO antigens are present in fetus

Foll Hb chains are present in intrauterine life


Alpha 2 beta2
Alpha 2 gamma 2
Gamma 4
Alpha 2 epsilon 2
Zeta 2 epsilon2

Pulm embolism
Spiral CT scan is indicated in suspected cases
DVT always causes pulm embolism
X ray angiography is diagnostic
Spiral CT involves dignoses based on the perfusion copared to air in
lungs
ECG is reliable for diagnosis

MRI
Involves administerind of radio contrast agents
Protons give better imaging
Damage to fetus is due to tissue heating

Xray
Contraindicated in pregnancy
Low rays are used for breast imaging

Conventional USG
Increased freq gives higher resolution
Increased freq gives better attenuation
TVS uses higher freq compared to conventional USG
Doppler USG measures movement
Gives accurate measurement of bld flow in fetal vessels
Doppler effect based on blood flow within vessels
Calcification occurs in
Renal calculi
Hyaline degeneration of fibroid
Parathyroid adenomas
Secondary bone deposits in prostratic CA

Foll originate from endothelial cells


Nitric oxide
Protein C
Fibronectin
Plasminogen activator

Thyroid
100-200 micro gms are trapped daily
increases in size in pregnancy
trapsinorganic iodine
colloid is stored outside epithelial cells
TBG is increased n pregnancy
5% is bound to CBG

ketone bodies
formed from acetone
formed from acetyl CoA
not formed during starvation
not utilised by brain

Mgso4
Potentiates action of non depolarising agents
Check respiratory status
Causes brisk patellar reflexes
In toxicity Rx IV calcium sulphate

CIN
CIN 3 does not involve breach in basement epithelial layer
Arias stella reaction mimicks clear cell CA
Tissue biopsy is required for diagnosis
Incidence is decreased in renal transplant patients

GH
Stmulated by glucose infusion
Increased in sleep
Opposes insulin action
Causes positive nitrogen balance
Decreased in pregnancy

Foll form boundaries of ovarian fossa


Ureter
Ext iliac vn
Int iliac art
Int pudendal art
Obliterated umb art

Penem antibiotics
Active in beta lactam ring
Drug of choice for ps aeroginoa

Vagina
Derived from mullerian ducts
Seperated from anal canal by anal body

Bladder
Pelvic splanchnic stimulation contracts the trigone
Ext urethral sphincter innervated by pudendal nv
Mucosa of urethra is innervated by pudendal nv
Transaction of spinal cord above S2 causes autonomous bladder
Afferent innervation of glans of clitoris is by ilioinguinal nv

Hypo k+ is found in
Prolonged vomiting
In diabetis ketoacidosis
Addison’s disease

Foll are below inguinal ligament


Ilio inguinal nv
Lateral branch of genito femoral nv
Subcostal nv
Femoral art

Inf hypogastric (pelvic) plexus


Supplies pain fibres to body of ut
Continues as vesical plexus
Lies at base of braod lig

Sciatic nv
S3&4 contribute towards it
Lies behind quadratus femoris
Tibial branch supplies short head of biceps femoris
Injection on the outer & upper quadrant of the buttock may cause damage
to the nv

Undescended testis
More common in premature than term infants
Commonly assoc with gonadal neoplasia
Asoc with 1% of absent testis
More common on left side

Embryo
Parameso duct lies medial to mesonephros
Mesonephros forms urine
TVS detects fetal heart beat 21 days after implantation
Erthropoiesis is entirely medullary

UG sinus
Divides the cloaca coronally
Urachus connects bladder to umbilicus

Corpus luteum
Is major source of relaxin
Contains endothelial cells
Hysterectomy prolonges the life period
17 hydroxy prog is the main prog secreted
steroids prevent recruitment of new follicles

fertilisation
acrosome fuses with zona pellucida
polar body has same karyotype as ovum
fetilised egg moves slower than unfertilised egg in fallopian tube
inner ell mass lysis deciduas during implantation

histo pathologic examination of tissue of conception can diagnose


CIN 3
Cx incompetence
Ectopic preg
Gest Trophblastic Di

Ring inversion of Y chromosome found the foll will be done


Maternal chromosome is tested
Paternal chromosome is tested
Scan to look for male genitalia
Anomaly scan and amniocentesis done for fetus
Do karyotype of all previous children of couple

Right ureter
Below sigmoid coon
Supplied by vaginal art
Lined throughout by urothelium
Contain glands

Oral absorption of drug depends


More availablibility in ionised form than non ionised
Increased lipid solubility increases penetration in CNS
Enters intestinal cells by simle diffusion
Protein binding increases excretion
Lipid form increases uptake

Fetal circulation
Portal vn drains in IVC
Ductus arteriosus takes blood to pulm art
R atria and L atria communicate thro’ Foramen Ovale

Voiding in females
Occurs above rate of 20 ml/sec
When vesical press is more than 45 mm H2O
Initial contraction of trigone

Enzyme activity is modified by


Dephosphorylation
Allosteric esters
Binding to plasma prot
Proteolytic cleaving of inactive precursors
Gene transcription

Functional cyst occurs in


Stein levinthal synd
PID
Multiple sclerosis
Gest tropho di
Rx with clomiphene

Amniotic fluid
Hypotonic compared to fetal plasma
Absored in lungs
Unrelated to swallowing
Increased until term in fetus
Contains more HCO3 compared to fetal blood

Action of complement
Increase in permeability
Migration of polymorphs

Genetics
South blot detects DNA
DNA is exact copy of RNA
DNA is transcribed by RNA polymerase
Ribosomes contain RNA
Transcription occurs at any direction along the chain
DNA replication is called transformation
DNA has transcription restriction enzyme
Histones do not contain DNA

Insulin
Binding to tyrosine receptor kinase increases activity
Causes glucose transporter movement to plasma membrane
Increases glycogen synthetase activity
Increases fat deposition
Increase glucose utilisation by CNS

Heparin
T ½ is 1.5 hrs
LMW heparin is administered 4 hrly
Given in pregnancy causes intracranial haemorrhage in fetus
Longer duration of action in S/C compared to IV administration

Foll have imp natural reservoir other than human


Listeria
Saolmonella typhimurium
Brucella

Crohn’s disease
Non caseous granulomatous infection
Forms deep fissures
Forms crypt abscesses
Assoc with mucosal polyps
Recognised to occur in vulva
Assoc with intestinal wall thickening

Absorption in kdny
Glucose occurs in loop of henle
Water is actively absorbed in proximal conv tubules
Estrogen
17 beta estradiol is formed by aromatisation of testos
increases LH receptors
secreted by theca cells
granulosa cells secrete inhibin
humoral immunity is depressed in pregnancy
T & B cells are derived from bone marrow

teratoma
usually benign
can occur is assoc with germ cell tumors
secretes hormones

during menstruation
platelet and fibrin plug is formed after 1st 20 hrs of bleeding
hemostasis is thro’ estr mediated vaso constriction
hemostasis is thro PG mediated spiral vessel constriction

During ovulation
Basal vacuoles occur in mid secretory phase
Gland stromal mitosis maximum in mid secretory phase
PG in follicle fluid during ovulation
Increase of estr and prog receptors in proliferaive phase
LDL receptors are increased in proliferative phase
Indomethacin inhibits ovulation
Matrixmetalloproteinases(MMC) is inhibited by progestrones
Cloiphene can cause deficient secretory phase

Glycogen
Synthesis occurs in muscle
Is stored along with water

Carbohydrates
Yield energy ?17 J/Kg
Major source of energy for brain
Major component of diet
Higher energy compared with fats

Foll decrease the efficacy of OCP’s


Isoniazid
Carbamaepine
Phenytoin
Rifampicin

Female pelvis compared to male


Sacrum more curved
Greater sc notch is larger
Sub pubic angle is more
Distance betw pubic sym to acetabulum is less than the diam of
acetabulum

Vascular derived epithelial growth factor


Causes vasodilatation of placental vessels
Causes angiogenesis in placenta
Is an angigogenic factor
Increases permeability of endothelial cells

And always save the best for last ……STATISTICS

Definition
LBW – less than 1,500 gms
PMR – deaths in first 1 week per 1,000 births
Perinatal mortality – deaths after 22wks ( more than 154 days) until 1 wk
after birth
Maternal mortality rate

Tests used
ANOVA
Std error of mean means diffce betw sample mean from population mean
Probablity of 1 means 1 in 10 chance for event to occur

Examples of ?continuous variables are


Bld glucose
Haemoglobin
Gender
Height
Blood pressure

Sigmoid colon
Attached to pelvic wall......T
Lies lateral to psoas major ........F
Ends at S3 .........T

SIGMOID COLON...STARTS AT PELVIC BRIM.....ENDS AT


S3.......IS 45 CMS LONG.....LIES ON THE PERITONEAL SURFACE
OF BLADDER AND UTERUS.....SO THE PSOAS IS A DEEPER
STRUCTURE.....

Rectum
Starts at S3 ........T
No appendices epiploae.......T
Middle rectal artery is the major artery suuplying it........F......ITS THE
SUPERIOR RECTAL ARTERY
12 cm long .............T

T pallidum
Spiral shape........T
Causes yaws in children..........T
Stained with silver impregnation.........F.......
THE ONLY TESTS FOR MICROSCOPY ARE DARK GROUND
IMMUNOFLOURSCENCE................OTHERWISE SEROLOGY IS
THE OTHER DIAGNOSTIC AID....

HIV seroconversion
Illness occurs witin 1-4 wks......T...........SEROCONVERSION OCCURS
IN FIRST FEW WEEKS......FOLLOWED BY SIGNS OF ILLNESS
After illness infectivity decreases ..........F.........I THINK SO BECAUSE
THOUGH ITS A LATENT PHASE AND NO OVERT SYMPTOMS
OCCUR WHILE THE VIRUS MULTIPLIES, I THINK INFECTIVITY
REMAINS JUST THE SAME.........
The best available diagnostic test is HIV viral load
assay........T.........FROM AN OBSTETRIC POINT OF VIEW.....VIRAL
LOAD IS THE ONLY USEFUL WAY OF PREDICTING
DISEASE......BECAUSE SEROLOGY IS UNRELIABLE IN ITS
RESULTS DUE TO MATERNAL ANTIBODIES......WHICH MIGHT
TAKE UPTO 18MONTHS TO DIE DOWN...
Without any intervention during pregnancy the mother to child
transmission rate is 80% .........F.....ITS 15-45%......LATTER IN
UNTREATED CASES......
Mode of delivery by LSCS decreases the transmission to
baby......T.....CAN BRING THE TRANSMISS DOWN TO 2%
Short course of anti viral Rx during labor decreases transmission to
baby.........T
BF is safe ........F.......MOST IMP METHOD OF TRASMISS IF OTHER
PREVENTIVE MEASURES HAVE BEEN UNDERTAKEN....

Percutaneous mode of infection


H simplex........T
Rabies .......F.....BITE WITH SALIVA
Hepatitis B ........F......
Epstein barr virus .......T I THINK.......ECAUSE IT SAYS YOU CAN
GET IT FROM PATIENTS THROUGH "CASUAL CONTACT"......

Hep A mode of infection is through


Blood transfusion.....F
Shared needles .....F
Faeco oral route ......T
Insect bites ......F
Air borne ......F

Match the disease with their respective causative org


Bone marrow suppression – myco bacterium avium
Red cell aplasia – parvovirus

The foll are pre malignant


CIN 3 ........T
Vulval lichen sclerosis.......F BUT NOT SURE...
Endometrioses .......F
Fibro epithelial polyp ............T I THINK...
Atypical endometrial hyperplasia ??T

Bacterial vaginosis
Microscopic finding “clue cells”.........T
pH less than 4.5 .......F........HAS TO BE MORE THAN 5 FOR BV TO
OCCUR
best Rx with ampicillin ............F
sensitive to clindamycin ...........F.......I DIDNT FIND IT
ANYWHERE.....SO I WILL GO FOR F
may be due to gardnerella vaginalis........T.....

vaginal infecton
incidence of n gonorrhoea resistance to penicillin is 1% ......T BUT NOT
SURE 100%
incidence of n gonorrhoea resistance to 3 rd gen ceph 5% .....? FALSE
azithromycin DOC for trichomoniasis ,.......F....METRO...
crytococcus neoformans causes meningitis .........T.....ESP IN AIDS PTS

C trachomatis
Strains L123 cause LGV.........T
Reticulate body is the infectious form .......F.....RETICULATE I THE
VEGETATIVE NON INF INTRACELL. FORM....

Transferrin
Increased in pregnancy ........T
One third saturated with Fe .........T
Binds to 10 mg of iron per gm ???.....DUNNO....DIDNT FIND THIS
KIND OF DATA ANYWHERE....
Levels in neonate are low........FALSE.....

Antibodies
Are soluble proteins........I THINK ITS TRUE....THEY HAVE TO
BE.....BUT NO BOOK CONFIRMATION YET.....
Formed before 12 wks in fetus .........TRUE.....IGM FORMS AT 11
WEEKS....
ABO antigens are present in fetus........TRUE
Foll Hb chains are present in intrauterine life
Alpha 2 beta2 .......TRUE......HB A......PRESENT 8TH MONTH
ONWARS
Alpha 2 gamma 2 ........T......HB F......ALL THROUGH PREG AND
LATER TOO...
Gamma 4 ......F
Alpha 2 epsilon 2........F
Zeta 2 epsilon2 ........T.....HBGOWER 1.......PRESENT I THE FIRST
FEW WEEKS.....
ALSO THERE IS HBA2....ALPHA2 AND DELTA 2 UNITS...

Pulm embolism
Spiral CT scan is indicated in suspected cases.........F
DVT always causes pulm embolism ............F
X ray angiography is diagnostic ...............T
Spiral CT involves dignoses based on the perfusion copared to air in
lungs ...............F
ECG is reliable for diagnosis............F
IS SPIRAL CT AND VQ SCAN THE SAME THING????SORRY FOR
ASKING SUCH A BASICALLY DUMB QUESTION....

MRI
Involves administerind of radio contrast agents........T I THINK....
Protons give better imaging ?????T I THINK.........BECAUSE IT
DEENDS ON THE OSCILLATION OF CHARGED
PARTICLES......EPS THE HYDROGEN ATOM....
Damage to fetus is due to tissue heating.......F I THINK.....BECAUSE
MRI IS NOT AN IONISING RADIATION.....SO I DONT THINK
THERE IS ANY HEATING.....
Xray
Contraindicated in pregnancy...............F
Low rays are used for breast imaging ............????......LOW COMPARED
TO WHAT??7 TO 10 MRADS ARE USED..........WHICH IS MORE
THAN EG THE 2-3 MRADS FOR CHEST X RAY...........I DONT
THINK THATS LOW.......I WOULD SAY FALSE.....

Conventional USG
Increased freq gives higher resolution
Increased freq gives better attenuation
TVS uses higher freq compared to conventional USG
Doppler USG measures movement ........T
Gives accurate measurement of bld flow in fetal vessels........T
Doppler effect based on blood flow within vessels ...........T......THE
SOUND OF BLOOD FLOW WITHIN VESSELS....TO BE EXACT....
FOR THE FIRST THREE STEMS......I COULDNT FIND THE
ANSWER......BUT IT WAS SOMEWHERE IN THIS FORUM.........I
WILL POST IT AS SOON AS I COME ACROSS IT AGAIN.......

Calcification occurs in
Renal calculi ...........T
Hyaline degeneration of fibroid.........T.....CALCIFICATION OCURS IN
FIBROID........I DONT WHAT THE SIGNIFICANCE OF HYALINE
DEGENERATION IS IN TH QUESTION
Parathyroid adenomas ............T
Secondary bone deposits in prostratic CA ..........T
Foll originate from endothelial cells
Nitric oxide......T
Protein C ............F
Fibronectin ........T
Plasminogen activator................T

Thyroid
100-200 micro gms are trapped daily.......T
increases in size in pregnancy .........T I THINK.....ACTOVITY
INCREASES........SO DOES THE SIZE......BUT IM NOT SURE....
trapsinorganic iodine .......T
colloid is stored outside epithelial cells.........T
TBG is increased n pregnancy ...........T.
5% is bound to CBG ...........................TO TBG I THINK WAS
MEANT....F......T3 AND T4 ARE BOUND 75 AND 85 %

ketone bodies
formed from acetone..............T
formed from acetyl CoA ..........T
ormed during starvation .....T
not utilised by <a href="http://www.[obscene]a.com/tutorial/?q=brain
&s=0">brain </a> T

Mgso4
Potentiates action of non depolarising agents T I THINK
Check respiratory status T......PARALYSIS OF RESP MUSCLE IS THE
FIRST SIGN OF OVERDOSE
Causes brisk patellar reflexes T.........ANOTHER SIGN OF MG
EXCESS....
In toxicity Rx IV calcium sulphate .....F.....CA GLUCONATE...

CIN
CIN 3 does not involve breach in basement epithelial layer F....THATS
EXACTLY TH DISTINCTION
Arias stella reaction mimicks clear cell CA..........THIS ONE IS JUT
SOMEWHERE IN MY brain CELLS......DONT KNOW NOW....
Tissue biopsy is required for diagnosis ......FALSE....SMEAR CAN
ALSO SHOW CIN III CHNGES.......
Incidence is decreased in renal transplant patients?????......F I
THINK.....COULDNT FIND INFO ANYWHERE.......BUT MY GUESS
IS THAT IMMUNOSUPRESSION DURING RENAL TRANSPLANT
WOULD EXACERBATE MALIGNANCY RATHER THAN
SUPPRESS IT.....

GH
Stmulated by glucose infusion.........FALSE....
Increased in sleep ......T
Opposes insulin action F
Causes positive nitrogen balance F
Decreased in pregnancy F
GH STIM BY
LOW ENERGY CONDN
HYPOGLYCaenia
EXCERCISE
FASTING
HIGH AMINO ACIDS
HIGH PROT MEAL
ARGININE INFUSION
STRESS STIM
GLUCAGO
SLEEP

GH INHIB BY
GLUCOSE
CORTISOL
FFA
MEDROXYPROGEST
GH

ANABOLIC HORMONE......WANTS TO BULD AND


CONSTRUCT.....SO I THINK N2 BALANCE WOULD BE
NEGATIVE......
AND SHOULD BE HIGH IN PREG I THINK......

Foll form boundaries of ovarian fossa


Ureter T LATERAL TO OF
Ext iliac vn T ABOVE OF
Int iliac art T ABOVE THEN POST TO OF
Int pudendal art F.....WAY OFF BELOW THE LEVATOR
ANI....NOWHERE CLOSE...
Obliterated umb art..T LAT TO OVARY..
Penem antibiotics
Active in beta lactam ring T
Drug of choice for ps aeroginoa T

Vagina
Derived from mullerian ducts T
Seperated from anal canal by anal body....PINEAL BODY-T

Bladder
Pelvic splanchnic stimulation contracts the trigone F......ALL
TRUE....BUT NOTHING CONTRACTS THE TRIGONE!
Ext urethral sphincter innervated by pudendal nv....F....NO PUDENDAL
INVLVMT IN URETHERA......SUPP BY SOMATIC FIBS FROM S2-4
Mucosa of urethra is innervated by pudendal nv F
Transaction of spinal cord above S2 causes autonomous bladder T I
THINK........BECAUSE IT WOULD STILL WORK
AUTONOMOUSLY THROUGH REFLEX.......BUT NO SOMATIC
ACTION BECAUSE CEREBRAL CONTROL IS LOST....
Afferent innervation of glans of clitoris is by ilioinguinal nv ........F
AFFERRENT OF GLANS CLIT IS THROUGH PELVIC
SPLANCH......EFF IS THRU PUDENDAL N......AND BLOOD
SUPPLY IS THROUGH INT PUD ARE
PUD N ALSO SUPPS INF RECTAL NERVE AND A PERINEAL
NERVE....

Hypo k+ is found in
Prolonged vomiting T
In diabetis ketoacidosis T
Addison’s disease F HYPERKALEMIA HERE........CORTISOL IS
HYPOKALEMIC.......AND NO CORTISOL IN ADDISONS....
Foll are below inguinal ligament
Ilio inguinal nv T
Lateral branch of genito femoral nv T
Subcostal nv F
Femoral art T

Inf hypogastric (pelvic) plexus


Supplies pain fibres to body of ut T
Continues as vesical plexus T
Lies at base of braod lig T

Sciatic nv
S3&4 contribute towards it F L4 TO S3
Lies behind quadratus femoris F......IT LIES ON IRIFORMIS.....WIT NO
MORE USCLES ON TOP....QF IS A VERY DEEP MUSCLE
Tibial branch supplies short head of biceps femoris ......FASE......SHORT
HEAD IS BY COMMON PERONEAL.....LONG HEAD ISBY
TIBIAL.....
Injection on the outer & upper quadrant of the buttock may cause damage
to the nv ....FLASE........THIS IS THE SAFEST AREA
Undescended testis
More common in premature than term infants T
Commonly assoc with gonadal neoplasia T
Asoc with 1% of absent testis
More common on left side

Corpus luteum
Is major source of relaxin T RELAXIN, INHIBIN AND ACTIVIN
Contains endothelial cells T I THINK, IT HAS RICH BLOOD
SUPPLY.......SO ENDOTHELIAL CELLS TOO I SUPPOSE...
Hysterectomy prolonges the life period F I
THINK.....HYSTERECTOMY WOULD HAVE NO EFFECT.......OR IT
WOULD DECREASE THE LIFE......NO BOOK CONFIRMATION....
17 hydroxy prog is the main prog secreted F I
THINK.......PROGESTERONE IS THE MAIN
steroids prevent recruitment of new follicles T STEROIDS LOWER THE
LEVEL OF ESTROGEN RECEPTORS..

fertilisation
acrosome fuses with zona pellucida T
polar body has same karyotype as ovum T
fetilised egg moves slower than unfertilised egg in fallopian tube F
UNSURE.....BUT WHY SHOULD IT?
inner ell mass lysis deciduas during implantation F INNER CELL MASS
DOES NOTHING OUTSIDE.....IT BECOMES THE FETUS.....

histo pathologic examination of tissue of conception can diagnose


CIN 3 T
Cx incompetence F
Ectopic preg T
Gest Trophblastic Di T

Ring inversion of Y chromosome found the foll will be done


Maternal chromosome is tested F
Paternal chromosome is tested T
Scan to look for male genitalia T
Anomaly scan and amniocentesis done for fetus ?T
Do karyotype of all previous children of couple F
ALL ARE DEDUCTIONS......BECAUSE Y IS RELATED WITH
PATERNAL.....AND ASSOCIATED WITH EXPRESSION OF MALE
PHENOTYPE.......PLEASE DISCUSS

Right ureter
Below sigmoid coon F LEFT URETER
Supplied by vaginal art T
Lined throughout by urothelium T IF UROTHELIUM IS ANOTHER
NAME FOR TRANSITIONAL EPITH
Contain glands F.......BLADDER EPITH IS DEVOID OF GLANDS......I
THINK URETERS' IS ALSO THE SAME....

Oral absorption of drug depends


More availablibility in ionised form than non ionised F.....BECAUSE I
DIDNT FIND IT ANYWHERE THAT IONISED FORM INFLUENCED
ORAL ABS.....
Increased lipid solubility increases penetration in CNS T
Enters intestinal cells by simle diffusion T
Protein binding increases excretion F
Lipid form increases uptake T

Fetal circulation
Portal vn drains in IVC T
Ductus arteriosus takes blood to pulm art F AWAY FROM IT
R atria and L atria communicate thro’ Foramen Ovale T

Voiding in females
Occurs above rate of 20 ml/sec T MAX URINE FLOW RATE =20-40
ML/SEC
When vesical press is more than 45 mm H2O T VOID PRESS=45-70
MM
Initial contraction of trigone F........TRIGONE DOES NOTHING!!!

Enzyme activity is modified by


Dephosphorylation T
Allosteric esters T
Binding to plasma prot T
Proteolytic cleaving of inactive precursors T
Gene transcription ?F
ALL GUESSES......ANYONE KNOWS THE ANSWERS??
Functional cyst occurs in
Stein levinthal synd T
PID F
Multiple sclerosis F
Gest tropho di F
Rx with clomiphene T

Amniotic fluid
Hypotonic compared to fetal plasma ?T
Absored in lungs ?T
Unrelated to swallowing F
Increased until term in fetus F DECS TO TERM
Contains more HCO3 compared to fetal blood ?T

Action of complement
Increase in permeability T
Migration of polymorphs T

Genetics
South blot detects DNA F ITS ONLE SEROLOGICAL TEST
DNA is exact copy of RNA F
DNA is transcribed by RNA polymerase T
Ribosomes contain RNA T
Transcription occurs at any direction along the chain F ONLY 5' TO 3'
DNA replication is called transformation F
DNA has transcription restriction enzyme ?F DIDNT FIND ANYTHING
LIKE THAT ANYWHERE
Histones do not contain DNA T

Insulin
Binding to tyrosine receptor kinase increases activity T
Causes glucose transporter movement to plasma membrane T
Increases glycogen synthetase activity T
Increases fat deposition T
Increase glucose utilisation by CNS ?T

Heparin
T ½ is 1.5 hrs T
LMW heparin is administered 4 hrly F 8 TO 12
Given in pregnancy causes intracranial haemorrhage in fetus F DOESNT
CROSS PLAC COS ITS TOO BIG
Longer duration of action in S/C compared to IV administration T

Foll have imp natural reservoir other than human


Listeria F
Saolmonella typhimurium F
Brucella T FROM DAIRY AIMALS

Crohn’s disease
Non caseous granulomatous infection T
Forms deep fissures T
Forms crypt abscesses T
Assoc with mucosal polyps F......PSEUDOPOLYPS
Recognised to occur in vulva F......AT THE MOST....A FISTULA
MIGHT OPEN I THE VULVA......BUT THATS NOT CROHNS
OCCURING IN THE VULVA I THINK....
Assoc with intestinal wall thickening ? T......NOT A PROPER
STATEMENT......THERE IS INFLAMMATION.....THICKENING IS A
VERY LAYMAN TERM I THINK....

Absorption in kdny
Glucose occurs in loop of henle F ACTIVE ABSPN IN PCT WITH AA
Water is actively absorbed in proximal conv tubules F OPASSIVE
DIFFSN IN PCT WITH NA AND CL

Estrogen
17 beta estradiol is formed by aromatisation of testos T
increases LH receptors T
secreted by theca cells F GRANULOSA CELLS
granulosa cells secrete inhibin F CORP LUT DOES
humoral immunity is depressed in pregnancy F I THINK.....IF IT WERE
TRUE, RHESUS CONFLICT WOULDNT BE AN ISSUE
T & B cells are derived from bone marrow T

teratoma
usually benign T
can occur is assoc with germ cell tumors T
secretes hormones F....NOT USUALLY I THINK

during menstruation
platelet and fibrin plug is formed after 1st 20 hrs of bleeding F I
THINK.....PLATELET PLUG FORMN TAKES PLACE VERY QUICK
AND IN VERY SMALL TRAUMAS.....
hemostasis is thro’ estr mediated vaso constriction F
hemostasis is thro PG mediated spiral vessel constriction F PG ARE
CONSIDERED TO BE THE CAUSE.....BUT I DONT KNOW IF ITS
AT THE SPIRAL ARTS OR BELOW THEM.......BECAUSE THE
APIRAL ARTS ARE ACTUALLY SHED....ISNT IT...

During ovulation
Basal vacuoles occur in mid secretory phase T
Gland stromal mitosis maximum in mid secretory phase T
PG in follicle fluid during ovulation F DIDNT FIND THAT
ANYWHERE
Increase of estr and prog receptors in proliferaive phase T I THINK
LDL receptors are increased in proliferative phase ?F
Indomethacin inhibits ovulation F
Matrixmetalloproteinases(MMC) is inhibited by progestrones ???!!!
WOW NO CLUE
Cloiphene can cause deficient secretory phase F ITS ANTI
ESTROGEN......SO IT MAY SHORTEN PROLIF PHASE....

Glycogen
Synthesis occurs in muscle T
Is stored along with water F ITS POLYMERISED......AND TURNS TO
CRYSTALS ON STORING.....THIS IS THE REASON THERE IS NO
OSMOTIC HAVOC IN CELLS WITH IT

Carbohydrates
Yield energy ?17 J/Kg T
Major source of energy q=brain &s=0">brain </a> F I THINK ITS
GLUCOSE ACTULLY.....
Major component of diet T
Higher energy compared with fats F FATS HAVE MAX ENERGY

Foll decrease the efficacy of OCP’s


Isoniazid F
Carbamaepine T
Phenytoin T
Rifampicin T
ALSO PHENOBARB

Female pelvis compared to male


Sacrum more curved F LESS CURVED AND MORE MORE BACK
TILT
Greater sc notch is larger T
Sub pubic angle is more T
Distance betw pubic sym to acetabulum is less than the diam of
acetabulum F......ACETABULUM IS VERY SMALL...AND THE
DISTANCE IS GREATER

You might also like